वैश्विक समस्या बड़े शिकारियों की आबादी में गिरावट है। घटती जैव विविधता पंख घास की गिरावट का कारण

1. कारण जो जनसंख्या की स्थिरता का उल्लंघन करते हैं।

अत्यधिक खनन। प्रत्येक जनसंख्या को "ऊपर से" और "नीचे से" नियंत्रित किया जाता है। "नीचे से" इसे संसाधनों की मात्रा द्वारा नियंत्रित किया जाता है, और "ऊपर से" अगले ट्राफिक स्तर के जीवों द्वारा नियंत्रित किया जाता है। यदि किसी व्यक्ति द्वारा जनसंख्या के जैविक उत्पादन का एक निश्चित हिस्सा वापस ले लिया जाता है, तो यह अधिक गहन प्रजनन के कारण होने वाले नुकसान की भरपाई करता है। एमआरएल (अधिकतम स्वीकार्य उपज)।

उदाहरण: एल्क की शूटिंग की दर 15% है, और जंगली सूअर 30% है। हालांकि, अक्सर एक व्यक्ति इस मानदंड से अधिक हो जाता है और आबादी से "सुपर प्रॉफिट" प्राप्त करने की कोशिश करता है। यह आबादी को कमजोर कर सकता है।

स्थान बरबादी। चराई मिट्टी को संकुचित करती है और घास के मैदानों और मैदानों की प्रजातियों की संरचना को खराब करती है। रूस के यूरोपीय भाग में, स्टेपी घास स्टैंड की संरचना में पंख घास की आबादी (सुंदर, लेसिंगा, आम - पिननेट) दुर्लभ हो गई है। स्टेपीज़ की जुताई और कुंवारी भूमि के विकास के परिणामस्वरूप कई कीट आबादी गायब हो गई है। आबादी के आवास पर्यटकों और छुट्टियों द्वारा नष्ट कर दिए जाते हैं, और उपनगरीय क्षेत्र नागरिकों द्वारा नष्ट कर दिए जाते हैं। उच्च गति परिवहन द्वारा जलीय आवास नष्ट हो जाते हैं। इसके पारित होने के दौरान होने वाली लहर किशोर मछलियों को नष्ट कर देती है। मोटर बोट की टक्कर से मछलियां भी मर रही हैं।

नई प्रजातियों का परिचय। मनुष्य जानबूझकर दुनिया के विभिन्न क्षेत्रों में प्रजातियों का परिचय देता है। ये प्रजातियां अपनी आबादी को नष्ट करते हुए, देशी प्रजातियों को बाहर निकाल सकती हैं।

पर्यावरण प्रदूषण। कई पौधों और जानवरों की प्रजातियों की आबादी घनत्व में कम हो जाती है और यहां तक ​​कि कृषि और औद्योगिक प्रदूषण के प्रभाव में गायब हो जाती है। जलीय पारिस्थितिक तंत्र के निवासी इससे सबसे अधिक पीड़ित हैं।

2. पर्यावरण को प्रदूषण से बचाएं।

नुकसान कम करने के उपाय।

तनुकरण। यहां तक ​​​​कि उपचारित बहिःस्राव को भी 10 बार पतला किया जाना चाहिए, लेकिन यह विधि अक्षम है और केवल अस्थायी उपायों के रूप में ही संभव है।

सफाई। रूस में, यह मुख्य विधि है, हालांकि, उपचार के परिणामस्वरूप, बहुत अधिक केंद्रित अपशिष्ट उत्पन्न होता है, जिसे संग्रहीत भी करना पड़ता है।

पुरानी तकनीकों को नई कम-अपशिष्ट तकनीकों से बदलना। एक उद्योग का कचरा दूसरे उद्योग के लिए कच्चा माल बन जाता है।

उपचार की सुविधा।

एमपीसी - अधिकतम अनुमेय एकाग्रता।

एमपीई समय की प्रति यूनिट अधिकतम स्वीकार्य उत्सर्जन है जिस पर प्रदूषक की सांद्रता मैक से अधिक नहीं होगी।

यांत्रिक सफाई। तरल बहिस्राव जम जाता है, जबकि ठोस कण जमा हो जाते हैं। रेत और रेत-बजरी फिल्टर का उपयोग किया जाता है, जो हल्के निलंबित कणों को फंसाते हैं जो बसने वाले टैंकों में नहीं बसे हैं। कभी-कभी सेंट्रीफ्यूजेशन का उपयोग किया जाता है। तेल उत्पाद जो नाबदान में सामने आए हैं, वे यंत्रवत् रूप से अलग हो गए हैं। गैस उत्सर्जन को साफ करने के लिए, विशेष धूल-सेटिंग कक्ष और सेंट्रीफ्यूज (चक्रवात), कपड़े फिल्टर का उपयोग किया जाता है।

रासायनिक सफाई। घुलनशील यौगिकों को अघुलनशील में परिवर्तित करके, रसायनों के साथ बहिःस्राव का उपचार किया जाता है। ऑक्साइड और हाइड्रोजन सल्फाइड के उत्सर्जन को कम करने के लिए, क्षारीय वर्षा का उपयोग किया जाता है, जिसके माध्यम से गैसीय उत्सर्जन होता है, जिसके परिणामस्वरूप नमक और पानी होता है। अवशोषक-फिल्टर के रूप में, विशेष adsorbents का उपयोग किया जाता है: सक्रिय कार्बन, एलुमोगेल, सिलिका जेल, रेजिन - आयन एक्सचेंजर्स।

भौतिक और रासायनिक सफाई। इलेक्ट्रोलिसिस द्वारा यह शुद्धिकरण जटिल यौगिकों को सरल में परिवर्तित करता है और धातुओं, एसिड और अन्य अकार्बनिक यौगिकों को निकालता है। सबसे खतरनाक या मूल्यवान दूषित पदार्थों को अलग करने के लिए, जिनका उपयोग आगे की प्रक्रिया के लिए किया जाता है, आयन-एक्सचेंज रेजिन का उपयोग किया जाता है, जैसे स्पंज जो इन पदार्थों को अवशोषित करते हैं।

घरेलू कचरे के प्रसंस्करण में अग्नि विधि का उपयोग किया जाता है।

जैविक सफाई। विशेष रूप से डिजाइन किए गए पारिस्थितिक तंत्र में, प्रदूषक सूक्ष्म जीवों और छोटे जानवरों द्वारा विस्तारित या केंद्रित होते हैं। जीव भारी धातुओं और रेडियोधर्मी समस्थानिकों के साथ जमा और अवक्षेपित हो सकते हैं (डायटोपिक शैवाल इसे विशेष रूप से सफलतापूर्वक करते हैं)।

टिकट #9

1. जीवों के व्यवहार के प्रकार।

विभिन्न प्रकार के पौधे और जानवर जीवित रहने के तरीकों में भिन्न होते हैं - जीवों की रणनीतियाँ। जीवों को तीन समूहों में बांटा गया है, जिन्हें लाक्षणिक रूप से "शेर", "ऊंट" और "गीदड़" नाम दिया गया है।

जीवों के लाक्षणिक नाम

रहने की स्थिति

प्रतियोगिता का स्तर

कार्यात्मक और वास्तविक निचे का अनुपात

जीवों के उदाहरण

"शेर" अनुकूल परिस्थितियों में रहें मजबूत प्रतियोगी आमतौर पर कार्यात्मक और वास्तविक निशान मेल खाते हैं शेर, बाघ, हाथी, ओक।
"ऊंट" संसाधन-गरीब वातावरण में रहते हैं कोई प्रतियोगी नहीं आला स्तर मैच ऊंट, कैक्टस, पंख घास
"जैकल्स" वे "शेर" जैसे संसाधनों की प्रचुरता वाली परिस्थितियों में रहते हैं कमजोर प्रतिस्पर्धी क्षमता कार्यात्मक आला अधिक है, लेकिन ज्यादा नहीं मक्खी के लार्वा, सियार, खेत के पौधे।

इसके व्यावहारिक कार्यान्वयन के उद्देश्य से। तालिका 1.2.1। सीखने का भेद। छात्रों के बाहरी आंतरिक आत्म-विभेद उनके सीखने के स्तर के अनुसार (अलग-अलग जटिलता की समस्याओं को हल करके) विशेष स्कूल गणित के गहन अध्ययन के साथ कक्षाएं, शिक्षक विकास के स्तर को निर्धारित करता है और ...

अनुभव, अपनी बात व्यक्त करें। इसलिए, हमें विभिन्न उत्तर मिले जिनका सामान्यीकरण करना कठिन है। सामान्य डेटा तालिका संख्या 7 में दिया गया है। तालिका संख्या 7 कक्षा 1-2 में माता-पिता के साथ शिक्षक के काम की विशेषताएं। प्रश्न शब्द उत्तर विकल्प शहर के अनुसार उत्तरदाताओं की संख्या कुल क्लेपेडा सियाउलिया लिडा 13. क्या हैं...

स्कूली गणित की सामग्री में महारत हासिल करने के गुणात्मक रूप से नए स्तर पर। दूसरा अध्याय। 5-9 ग्रेड में समीकरणों को हल करने के शिक्षण के साधन के रूप में स्वतंत्र कार्य के उपयोग के लिए पद्धतिगत और शैक्षणिक नींव। § 1. ग्रेड 5-9 में समीकरणों को हल करने के शिक्षण में स्वतंत्र कार्य का संगठन। शिक्षण के पारंपरिक तरीके में, शिक्षक अक्सर छात्र को किसी वस्तु की स्थिति में रखता है...




सीखने की प्रक्रिया में, डेटाबेस जैसे सूचना संसाधनों के साथ काम करने के लिए पूरी तरह से नए दृष्टिकोण विकसित करना आवश्यक है। अध्याय 2. गणित में एकीकृत राज्य परीक्षा के लिए छात्रों को तैयार करने की प्रक्रिया में गणितीय समस्याओं के डेटाबेस का उपयोग करने की तकनीक 2.1 मॉडल का कार्यान्वयन छात्रों के मानसिक कार्यों के क्रमिक गठन के सिद्धांत के अनुसार, एक के वितरण की तैयारी एकीकृत ...

1. जीवमंडल, जीवमंडल की सीमाएं, जीवमंडल की वैश्विक समस्याएं।

जीवमंडल सबसे बड़ा पारिस्थितिकी तंत्र है। यह ग्रह का खोल है
जीवों का निवास है।

जीवमंडल का सिद्धांत वर्नाडस्की द्वारा बनाया गया था। उन्होंने साबित कर दिया कि 4 अरब वर्षों में
जीवित जीवों के अस्तित्व के कारण, उन्होंने बड़े परिवर्तन किए
हमारा ग्रह (ऑक्सीजन प्रकट होता है, तलछटी चट्टानें बनती हैं)।

जीवमंडल की सीमाएँ 20 किमी से थोड़ी अधिक हैं।

जमीन पर ऊंचाई 6 किमी है।

समुद्र में 11 किमी की गहराई पर।

जीवमंडल की वैश्विक समस्याएं:

1. ग्रीनहाउस प्रभाव।

वैज्ञानिकों के अनुसार 19वीं शताब्दी के अंत से लेकर वर्तमान तक का तापमान
हमारा ग्रह 1.2 डिग्री सेल्सियस बढ़ गया है। यह तेजी से विकास के कारण है
उद्योग। ग्लोबल वार्मिंग बड़े पैमाने पर पिघलने का कारण बन सकती है
ग्लेशियर और बढ़ते समुद्र के स्तर।

2. ओजोन छिद्र (ओजोन परत का ह्रास):

यह प्रक्रिया विशेष रूप से ध्रुवों पर तेजी से होती है, जहां
ओजोन छिद्र। खतरा यह है कि अल्ट्रा-वायलेट
विकिरण जीवों के लिए हानिकारक है। थकावट का मुख्य कारण
ओजोन परत फ्रीन्स का उपयोग है। 90 के दशक में हस्ताक्षर किए
लोकप्रिय समझौता, जिसके अनुसार फ्रीन्स का उत्पादन 2 . कम कर दिया गया था
बार।

2. शहरी पारिस्थितिक तंत्र की मुख्य विशेषता।

शहरी पारिस्थितिकी तंत्र विषमपोषी हैं। मुख्य ऊर्जा स्रोत
शहरों के बाहर स्थित है। यह तेल, गैस और कोयले का क्षेत्र है।
शहर पानी की एक बड़ी मात्रा की खपत करता है, और इसे प्रकृति में लौटाता है
प्रदूषित राज्य। पानी भारी धातुओं से संतृप्त है और
तेल के पदार्थ। शहरी पारिस्थितिक तंत्र में पौधों की मुख्य भूमिका नहीं है
ऊर्जा उत्पादन, और वातावरण की गैस संरचना का विनियमन।

उत्पादक: वन पार्क, पार्क और लॉन के पौधे।

उपभोक्ता: पक्षी, पालतू जानवर।

शहरी पारिस्थितिक तंत्र की मुख्य विशेषता यह है कि यह असंभव है
पारिस्थितिकी संतुलन।

1. बी कॉमनर के पारिस्थितिक नियम।

1. सब कुछ हर चीज से जुड़ा है।

जीवमंडल: किसी एक देश में पारिस्थितिक सुख नहीं हो सकता
(ग्रीनहाउस प्रभाव, ओजोन छिद्र)।

प्राकृतिक पारिस्थितिकी तंत्र: पारिस्थितिकी तंत्र संतुलन की स्थिति में है,
इसे विनाश या एक नई प्रजाति की शुरूआत से तोड़ा जा सकता है।

कृषि पारिस्थितिकी तंत्र: पशुओं की संख्या कृषि योग्य भूमि के क्षेत्रफल पर निर्भर करती है।
इनके सही अनुपात से जड़ी-बूटी नष्ट नहीं होती, उसका रख-रखाव होता है
प्रजनन क्षमता।

शहरी पारिस्थितिकी तंत्र: शहर पारिस्थितिक तंत्र हैं जिनमें संतुलन
टूटना होता है। पारिस्थितिकी तंत्र पर मानव प्रभाव को कम करने के लिए,
पर्यावरण प्रदूषण को कम करने की जरूरत है।

2. आपको हर चीज के लिए भुगतान करना होगा।

बायोस्फीयर: इंटरनेशनल सोसाइटी वैज्ञानिक परियोजनाओं को निधि देती है,
जलवायु की जैव विविधता के संरक्षण के लिए। गरीब देश
पर्यावरणीय समस्याओं को सुलझाने में अमीरों की मदद का आनंद लें।

प्राकृतिक पारिस्थितिकी तंत्र: लागत सेवाओं को बनाए रखने की ओर जाती है,
प्राकृतिक संसाधनों और सृजन के तर्कसंगत उपयोग को नियंत्रित करना
भंडार।

कृषि पारिस्थितिकी तंत्र: खर्च फ़ीड और उर्वरकों की खरीद पर जाता है, साथ ही साथ
वनों और जल निकायों का संरक्षण।

शहरी पारिस्थितिकी तंत्र: अपशिष्ट जल के निर्माण पर धन खर्च किया जाता है
संरचनाएं।

3. सब कुछ कहीं जाना है।

बायोस्फीयर: अंतरराष्ट्रीय समुदाय ने निर्यात पर प्रतिबंध पर एक कानून अपनाया
गरीब देशों से परमाणु कचरा।

प्राकृतिक पारिस्थितिकी तंत्र: प्राकृतिक पारिस्थितिकी तंत्र बनाने की जगह नहीं है
लैंडफिल।

कृषि पारिस्थितिकी तंत्र: एक उचित रूप से संगठित पारिस्थितिक प्रक्रिया के साथ, में
कृषि पारिस्थितिकी तंत्र में कोई अपशिष्ट नहीं है: खाद उर्वरक है।

शहरी पारिस्थितिकी तंत्र: दफन को सुरक्षित रखने के लिए सावधानी बरतनी चाहिए
जहरीला और रेडियोधर्मी कचरा।

4. प्रकृति सबसे अच्छी तरह जानती है:

प्राकृतिक पारिस्थितिकी तंत्र: आपको पारिस्थितिकी तंत्र से उतने ही संसाधन लेने होंगे जितने
यह स्वाभाविक रूप से कितना बहाल कर सकता है।

कृषि पारिस्थितिकी तंत्र: उन पौधों को उगाना आवश्यक है जिनके लिए अधिक
अनुकूल मिट्टी और जलवायु।

शहरी पारिस्थितिकी तंत्र: शहर में पारिस्थितिक स्थिति में सुधार करने के लिए,
हरित क्षेत्र बनाने के लिए पार्कों और उद्यानों का उपयोग करना आवश्यक है।

2. पारिस्थितिक संतुलन की सामान्य विशेषताएं।

पारिस्थितिक संतुलन एक ऐसी अवस्था है जिसमें संरचना और
उत्पादकता जैविक परिस्थितियों के अनुरूप है।

संतुलन की मुख्य विशेषता इसकी गतिशीलता है। दो भेद करें
गतिशीलता प्रकार:

1. पारितंत्र में उत्क्रमणीय परिवर्तन पारितंत्र में होने वाले परिवर्तन हैं
विभिन्न वर्षों और परिवर्तनों में जलवायु में उतार-चढ़ाव के साथ वसंत से वसंत तक के वर्ष
अपने जीवन चक्र की लय के संबंध में कुछ प्रजातियों की भूमिका। इस तरह के लोगों के साथ
परिवर्तन, पारिस्थितिकी तंत्र की प्रजातियों की संरचना संरक्षित है, और पारिस्थितिकी तंत्र
बाहरी और आंतरिक कारकों (भालू) में उतार-चढ़ाव को समायोजित करता है।

2. उत्तराधिकार - परिस्थितियाँ बदलने पर पारितंत्रों में लगातार परिवर्तन
पर्यावरण (आग के बाद वनों की कटाई)। धीरे-धीरे कुछ प्रजातियों की भूमिका
घटता है जबकि अन्य बढ़ता है। उत्तराधिकार को के रूप में कहा जा सकता है
आंतरिक और बाहरी कारक।

1. पारिस्थितिकी तंत्र की सामान्य विशेषताएं। पारिस्थितिकी तंत्र की संरचना।

"पारिस्थितिकी तंत्र" शब्द पहली बार 1935 में टेन्सली द्वारा गढ़ा गया था।

एक पारिस्थितिकी तंत्र जीवित जीवों के परस्पर क्रिया का एक संग्रह है और
पर्यावरण की स्थिति।

बायोगेकेनोसिस पौधों, जानवरों, सूक्ष्मजीवों का एक संग्रह है,
भूमि के सजातीय टुकड़े पर मिट्टी और वातावरण।

पारिस्थितिक तंत्र के बीच कोई स्पष्ट सीमा नहीं है, और एक पारिस्थितिकी तंत्र सुचारू रूप से संक्रमण करता है
दूसरे करने के लिए।

बड़े पारिस्थितिक तंत्र छोटे पारिस्थितिक तंत्रों से बने होते हैं।

पारिस्थितिकी तंत्र जितना छोटा होता है, उसके सदस्य उतने ही निकट से परस्पर क्रिया करते हैं।
जीव।

उभयचर

ऑर्चर्ड

किशमिश

पक्षी कीड़े

जीवाणु एककोशिकीय और बहुकोशिकीय

पारिस्थितिकी तंत्र के प्रकार:

पारिस्थितिकी तंत्र प्राकृतिक

(प्राकृतिक कारकों के प्रभाव में गठित, हालांकि एक व्यक्ति कर सकता है
उनके प्रभाव में हो): वन, टुंड्रा।

मानवजनित

(आर्थिक गतिविधि की प्रक्रिया में मनुष्य द्वारा निर्मित): फल
बगीचा, मैदान।

पारिस्थितिकी तंत्र

स्वपोषी

(एटोट्रॉफ़्स का वर्चस्व)

परपोषी

(हेटरोट्रॉफ़्स प्रबल होते हैं)

फोटोऑटोट्रॉफ़्स कीमोऑटोट्रॉफ़्स

(टुंड्रा, टैगा, (जैविक)

क्षेत्र) उपचार संयंत्र

और पारिस्थितिकी तंत्र

भूजल)।

2. स्टेपी पारिस्थितिक तंत्र।

स्टेपी पारिस्थितिक तंत्र के निर्माण में एक महत्वपूर्ण भूमिका जंगली झुंडों द्वारा निभाई गई थी
खोलना स्टेपी में, पौधों की प्रजातियां जो प्रतिरोधी हैं
जानवरों को खाने के बाद रौंदना और तेजी से बढ़ना (पंख घास,
fescue), साथ ही कड़वी जड़ी-बूटियाँ (वर्मवुड)।

1मी? ~ 80 पौधों की प्रजातियां।

मुख्य मिट्टी का प्रकार चेरनोज़म है। स्टेप्स भी ढलानों पर स्थित हैं।
पहाड़ी मिट्टी (उरल्स की ढलान, काकेशस की तलहटी बेल्ट), शाहबलूत (दक्षिणी)
साइबेरिया)।

साल दर साल, रूस में स्टेप्स कम और कम रहे, लेकिन पशुधन
पशुधन कम नहीं हुआ। इससे घास नष्ट हो गई
संरक्षित कदम। बहाली का सिलसिला शुरू हो गया है। स्टेपी की स्थिति
पारिस्थितिक तंत्र बिगड़ गया है: प्रजातियों की संरचना खराब हो गई है, प्रजातियों की संख्या घट गई है
फोर्ब्स और पंख घास। हरड़ में फ़ेसबुक और वर्मवुड अधिक होते हैं।
स्टेपीज़ की प्राथमिक जैविक उत्पादकता में कमी आई है, उनकी
जीव ग्राउंडहोग गायब हो गए हैं, और कम कृंतक और चील हैं जो उन पर फ़ीड करते हैं,
बाधाएं

स्टेपी पारिस्थितिक तंत्र में संतुलन बहाल करना दो तरह से संभव है:

1. पशुधन की संख्या कम करना।

2. चारागाह क्षमता बढ़ाना।

चरागाह क्षमता पशुधन की वह मात्रा है जिसे एक पर चराया जा सकता है
साल भर हेक्टेयर घास को नष्ट किए बिना।

घास के मिश्रण की जुताई और बुवाई करना एक मूलभूत सुधार है
फलियों के मिश्रण के साथ सूखा प्रतिरोधी घास।

1. पारिस्थितिकी की संरचना।

पर्यावरण विज्ञान

सामान्य पारिस्थितिकी एप्लाइड साइंसेज

बायोस्फेरिक पारिस्थितिकी वैश्विक परिवर्तनों का अध्ययन करती है जो पर होते हैं
आर्थिक गतिविधि के प्रभाव के परिणामस्वरूप हमारा ग्रह
प्राकृतिक घटनाओं के लिए आदमी।

वन पारिस्थितिकी इस बात का अध्ययन है कि कैसे वन संसाधन (लकड़ी,
खेल जानवर, जामुन, आदि) उनकी निरंतर बहाली के साथ और
परिदृश्य के जल शासन को बनाए रखने में वनों द्वारा निभाई गई भूमिका।

टुंड्रा की पारिस्थितिकी टुंड्रा में तर्कसंगत प्रकृति प्रबंधन के तरीकों का अध्ययन करती है और
वन-टुंड्रा - बारहसिंगा प्रजनन और शिकार; खनन पारिस्थितिकी तंत्र पर प्रभाव का अध्ययन करता है
तेल और गैस, हानिकारक प्रभाव को कम करने के तरीके विकसित करना
उद्योग।

समुद्र की पारिस्थितिकी मानव आर्थिक गतिविधि के प्रभाव का अध्ययन करती है
समुद्री पारिस्थितिक तंत्र, बहाल करने और बनाए रखने के तरीके विकसित करता है
समुद्री पारिस्थितिक तंत्र।

कृषि पारिस्थितिकी अध्ययन कैसे प्राप्त करें
मिट्टी और घास के संसाधनों की कमी के बिना कृषि उत्पाद, के साथ
पर्यावरण का संरक्षण और पर्यावरण के अनुकूल उत्पाद प्राप्त करना।

औद्योगिक पारिस्थितिकी औद्योगिक संयंत्रों से उत्सर्जन के प्रभाव का अध्ययन करती है
पर्यावरण पर और इस प्रभाव को कम करने की संभावना के माध्यम से
प्रौद्योगिकियों और उपचार सुविधाओं में सुधार।

शहरी पारिस्थितिकी मानव पर्यावरण में सुधार के तरीकों की खोज करती है
शहर मे।

चिकित्सा पारिस्थितिकी प्रदूषण से जुड़े मानव रोगों का अध्ययन करती है।
पर्यावरण, और उन्हें रोकने और इलाज के तरीके।

गणितीय पारिस्थितिकी मॉडल पारिस्थितिक प्रक्रियाएं, अर्थात।
प्रकृति में परिवर्तन जो तब होगा जब पर्यावरण में परिवर्तन होगा
स्थितियाँ।

रासायनिक पारिस्थितिकी पदार्थों के निर्धारण के तरीकों का विकास करती है -
वातावरण में प्रवेश करने वाले प्रदूषक, मिट्टी, पानी, भोजन,
गैसीय, तरल और ठोस कचरे के रासायनिक शुद्धिकरण के तरीके और
नई उत्पादन प्रौद्योगिकियां जिसमें कचरे की मात्रा
घटता है।

आर्थिक पारिस्थितिकी पारिस्थितिक तंत्र विकसित करती है
पर्यावरण प्रबंधन - संसाधन लागत अनुमान और आकार
प्रदूषण जुर्माना।

कानूनी पारिस्थितिकी के उद्देश्य से कानूनों की एक प्रणाली विकसित होती है
प्रकृति का संरक्षण।

2. पर्यावरण का औद्योगिक प्रदूषण।


प्रदूषण घटाओ






पर गिरा


बांध





बिजली के तार।

1. ऊर्जा का प्रवाह (ओडम के अनुसार) और उदाहरण के रूप में ऑक्सीजन का उपयोग करने वाले पदार्थों का संचलन।

एक उदाहरण के रूप में ऑक्सीजन का उपयोग कर परिसंचरण:

प्रकाश संश्लेषण के दौरान हरे पौधों द्वारा ऑक्सीजन छोड़ी जाती है, और अवशोषित हो जाती है
सांस लेते समय इसके सभी जीवित जीव। सभ्यता के आगमन से पहले, यह चक्र
भी संतुलित था। आज ऑक्सीजन का उपयोग दहन में किया जाता है
कार के इंजनों में ईंधन, ताप विद्युत संयंत्रों की भट्टी में, in
विमान और रॉकेट इंजन, आदि। यह एक अतिरिक्त लागत है
ऑक्सीजन अपने चक्र के संतुलन को बिगाड़ सकती है। जब तक जीवमंडल मुकाबला करता है
ऑक्सीजन चक्र में मानवीय हस्तक्षेप के साथ: इसके नुकसान की भरपाई की जाती है
हरे पौधे। एक और कमी के साथ

वन क्षेत्र और अधिक से अधिक ईंधन जलाना
वातावरण में ऑक्सीजन की कमी होने लगेगी।

CO2 => प्रकाश संश्लेषण O2

पौधे की सांस

जानवरों की सांस

ईंधन दहन

2. गैर-पारंपरिक ऊर्जा।

सौर ऊर्जा का उपयोग। इसके लिए कई विकल्प हैं
उपयोग। सौर ऊर्जा को आत्मसात करने के भौतिक तरीकों के साथ
गैल्वेनिक बैटरी या दर्पण की एक प्रणाली का उपयोग करें जिसे वे अवशोषित करते हैं
सौर ऊर्जा और इसे तापीय या विद्युत ऊर्जा में परिवर्तित करें।
जैविक विधियों से ऐसे जीव उगाए जाते हैं जो ठीक करते हैं
प्रकाश संश्लेषण के लिए सौर ऊर्जा।

पवन ऊर्जा का उपयोग। एक आधुनिक पवनचक्की एक जटिल उपकरण है।
पवन टरबाइन का नुकसान ब्लेड से होने वाला शोर है।
प्रोपेलर रोटेशन के दौरान। ध्वनि प्रदूषण इसे खतरनाक बनाता है
स्थापना क्षेत्र में लोगों की उपस्थिति।

ज्वारीय बिजली संयंत्र। (पीईएस) पीईएस एक बांध के साथ एक जल विद्युत संयंत्र की तरह है, लेकिन
"जलाशय" उच्च ज्वार पर भर जाता है, और इस दौरान पानी बहता है
कम ज्वार का समय। जैसे ही यह बहता है, पानी टरबाइन ब्लेड को घुमाता है।

भूतापीय स्रोतों का उपयोग, अर्थात्। पृथ्वी की गहराई की गर्मी। यह
गर्मी का उपयोग लगभग किसी भी क्षेत्र में किया जा सकता है, लेकिन लागत
भुगतान करें जहां गर्म पानी पृथ्वी की पपड़ी की सतह के करीब है।

1. दरिद्रता का खतरा और जैविक विविधता के संरक्षण के उपाय
ग्रह।

जैव विविधता में संयुक्त प्रजातियों की संख्या है
पारिस्थितिकी तंत्र।

पारिस्थितिकी तंत्र के विभिन्न ब्लॉकों की विविधता आपस में जुड़ी हुई है।

जैविक संतुलन पर्यावरण की स्थितियों और उसकी संरचना पर निर्भर करता है, और पर
पारिस्थितिकी तंत्र की संरचना।

उदाहरण: उष्ण कटिबंधीय वनों में लगभग 1000 प्रजातियों की विविधता है, और
समशीतोष्ण क्षेत्र के मिश्रित वन में लगभग 60 प्रजातियां हैं।

एक व्यक्ति पारिस्थितिक तंत्र की जैविक विविधता की रक्षा कैसे करता है?

रिजर्व बनाए जा रहे हैं। हालांकि, अगर रिजर्व के संगठन से पहले क्षेत्र
मध्यम रूप से उपयोग किया जाता है, तो उपयोग की पूर्ण समाप्ति हो सकती है
प्रजातियों की उपस्थिति के कारण विविधता में वृद्धि होती है, लेकिन कम भी होती है
उसके। कुछ प्रजातियों को संरक्षित करने के लिए, रिजर्व का हिस्सा मध्यम है
उपयोग किया गया

2. पर्यावरण प्रदूषण के मुख्य प्रकार और उनसे बचाव के तरीके।

प्रदूषण के प्रकार प्रदूषक प्रदूषण के स्रोत विधियाँ
प्रदूषण घटाओ

वातावरण का रासायनिक प्रदूषण। सल्फर डाइऑक्साइड, नाइट्रोजन ऑक्साइड और
कार्बन, क्लोरीन, अमोनिया, फिनोल और हाइड्रोजन सल्फाइड। सीएचपी (कोयला या तेल),
थर्मल पावर प्लांट, बॉयलर हाउस, घरेलू भट्टियां, वाहन, तेल रिफाइनरियां
उद्यम। वायु निस्पंदन और उपचार संयंत्र

जल निकायों का रासायनिक प्रदूषण तेल उत्पाद, नाइट्रोजन यौगिक, फिनोल और
अन्य औद्योगिक अपशिष्ट। तेल रिफाइनरियों; तेल उत्पाद,
पर गिरा

परिवहन। जल प्रदूषण को कम करने के लिए निर्माण करना आवश्यक है
बांध

मृदा रासायनिक प्रदूषण उर्वरक, कीटनाशक, भारी धातु।
इसे रोकने के लिए कृषि, उद्योग, परिवहन,
फैक्ट्रियों में डस्ट कलेक्टर बनाना जरूरी

ध्वनि प्रदूषण ___________ औद्योगिक उद्यम,
यातायात। कम आबादी वाले क्षेत्रों में सड़कों का विस्तार।

विकिरण प्रदूषण ___________ टीवी स्टेशन, बड़े रेडियो,
रडार उपकरण। हाई वोल्टेज के पास न रखें
बिजली के तार।

टिकट नंबर 7।

1. जीवों के क्षैतिज संबंध।

एक ही उष्णकटिबंधीय स्तर के जीवों के बीच संबंध को कहा जाता है
क्षैतिज।

विभिन्न उष्णकटिबंधीय स्तरों के जीवों के बीच संबंध को कहा जाता है
खड़ा।

क्षैतिज संबंध

इंट्रास्पेसिफिक (एक पैक में भेड़िये) इंटरस्पेसिफिक (जंगल में पेड़)

रिश्तों:

प्रतिस्पर्धा (हमेशा उस संसाधन के लिए जाती है जो कम आपूर्ति में होता है)

उदाहरण: 1) एक भेड़िया और एक लोमड़ी भोजन के लिए प्रतिस्पर्धा करते हैं।

2) पेड़ प्रकाश के लिए प्रतिस्पर्धा करते हैं।

आपसी सहायता:

पारस्परिक सहायता अस्थायी है।

उदाहरण: 1) एक झुंड में, एक भेड़िया शावकों को खिलाती है। जब शावक
बड़े होकर, वे भोजन के लिए प्रतिस्पर्धा करते हैं।

2) बमुश्किल एक छाया की जरूरत है। सन्टी उसके लिए एक छाया बनाता है। जब स्प्रूस बढ़ता है, तो वे
पोषक तत्वों के लिए प्रतिस्पर्धा।

2. रूस में मिट्टी, मिट्टी की विविधता, मिट्टी - रहने का आवास
जीव।

मिट्टी एग्रोइकोसिस्टम का मुख्य संसाधन है। मिट्टी की उर्वरता निर्भर करती है
धरण भंडार, पोषक तत्व सामग्री, संरचना,
नमी प्रदान करना।

मिट्टी की संरचना उस गांठ का आकार और आकार है जिसमें वह टूट जाती है
मृदा। सर्वश्रेष्ठ बनावट: बारीक चिपचिपा।

सबसे उपजाऊ मिट्टी चेरनोज़म हैं, वे स्टेपी के तहत बनाई गई थीं
वनस्पति जो बहुत उत्पादक है और जिसकी जड़ें गहरी हैं
व्यवस्था।

चेर्नोज़म्स:

ह्यूमस परत लगभग 1m - 6-10%

स्प्रूस जंगलों की मिट्टी के पास, धरण की एक पतली परत के नीचे एक सफेद धुलाई होती है
परत। यह भट्ठी की राख जैसा दिखता है और इसे पॉडज़ोलिक कहा जाता है, और मिट्टी स्वयं -
पॉडज़ोलिक, या पॉडज़ोल।

पर्णपाती जंगलों के नीचे, मिट्टी का निर्माण होता है जो कब्जा करती है
पॉडज़ोल और चेरनोज़म के बीच संक्रमणकालीन स्थिति और ग्रे कहा जाता है
वन।

ग्रे वन:

परत 3-4 सेमी - धरण - 24%

पॉडज़ोल और चेरनोज़म के साथ समानता से, उन्हें हल्के भूरे रंग में विभाजित किया जाता है,
भूरा और गहरा भूरा जंगल।

अर्ध-रेगिस्तानी क्षेत्र में, मिट्टी में भी चेरनोज़म की तुलना में कम ह्यूमस होता है, और
उन्हें चेस्टनट कहा जाता है।

रूस के स्टेपी क्षेत्रों में विशेष मिट्टी होती है जो लवण से संतृप्त होती है:
क्लोराइड, सल्फेट्स, सोडा। मिट्टी की पूरी मोटाई में लवण के साथ संतृप्त
उन्हें सोलोंचक कहा जाता है, और यदि बहुत सारे लवण हैं, तो सोलोंचक। यदि एक
खारा परत सतह से कुछ गहराई पर स्थित है (यह
शायद 5-10 और 30-50 सेमी), मिट्टी को सोलोनेट कहा जाता है।

विभिन्न प्राकृतिक क्षेत्रों में बनने वाली मिट्टी के अलावा (पॉडज़ोलिक से तक)
शाहबलूत), दलदली मिट्टी रूस में आम है, जो हो सकती है
देश के किसी भी हिस्से में मिलें। वे पीट में संक्रमण का प्रतिनिधित्व करते हैं,
जिसमें धरण का निर्माण होता है, और पीट, जिसमें सड़ी हुई परत होती है
पौधे के अवशेष।

मिट्टी में जीवन शांत है: मायसेलियम में उलझे पौधों की जड़ें अवशोषित करती हैं
उसमें घुले नमी और पोषक तत्व, नाइट्रोजन स्थिर करने वाले जीवाणु
वायुमंडलीय नाइट्रोजन को आत्मसात करें, मिट्टी के जानवरों की एक विशाल सेना
जीवित और विशेष रूप से मृत जड़ों पर फ़ीड करता है और एक दूसरे को खाता है,
सूक्ष्मजीव कार्बनिक पदार्थों को सरल कार्बनिक में विघटित करते हैं और
खनिज यौगिकों और उन्हें मिट्टी के घोल में लौटा दें।

1. कारण जो जनसंख्या की स्थिरता का उल्लंघन करते हैं।

अत्यधिक खनन। प्रत्येक जनसंख्या को "ऊपर से" नियंत्रित किया जाता है और
"नीचे से"। "नीचे से" यह संसाधनों की मात्रा को नियंत्रित करता है, और "ऊपर से"
अगले ट्राफिक स्तर में जीव। यदि एक निश्चित भाग
किसी व्यक्ति द्वारा जनसंख्या के जैविक उत्पादों को वापस ले लिया जाता है, तो यह क्षतिपूर्ति करता है
अधिक गहन प्रजनन के कारण नुकसान। एमआरएल (अधिकतम
स्वीकार्य उपज)।

उदाहरण: एल्क की शूटिंग की दर 15% है, और जंगली सूअर 30% है। हालांकि, अक्सर एक व्यक्ति
इस मानदंड से अधिक है और जनसंख्या से "अधिशेष लाभ" प्राप्त करने का प्रयास करता है। यह
जनसंख्या को कमजोर कर सकता है।

स्थान बरबादी। चराई मिट्टी को संकुचित करती है और प्रजातियों को खराब करती है
घास के मैदान और कदमों की संरचना। रूस के यूरोपीय भाग में, पंख घास की आबादी
(सुंदर, लेसिंगा, साधारण-पंख वाले) रचना में दुर्लभ हो गए हैं
स्टेपी घास खड़ा है। परिणामस्वरूप कई कीट आबादी गायब हो गई है
सीढि़यों की जुताई और कुंवारी भूमि का विकास। जनसंख्या आवासों को नष्ट करें
पर्यटक और छुट्टी मनाने वाले, और उपनगरीय शहरी निवासी। जलीय आवास
तेजी से परिवहन को नष्ट कर देता है। इसके पारित होने के दौरान होने वाली लहर टूट जाती है,
किशोर मछली को मारता है। मोटर बोट की टक्कर से मछलियां भी मर रही हैं।

नई प्रजातियों का परिचय। मनुष्य जानबूझकर प्रजातियों का परिचय देता है
दुनिया के विभिन्न क्षेत्रों। ये प्रजातियां देशी प्रजातियों को नष्ट कर सकती हैं, नष्ट कर सकती हैं
उनकी आबादी।

पर्यावरण प्रदूषण। कई पौधों और जानवरों की प्रजातियों की आबादी घट रही है
उनका घनत्व और यहां तक ​​कि कृषि के प्रभाव में गायब हो जाते हैं और
औद्योगिक प्रदूषण। रहवासियों को सबसे ज्यादा परेशानी
जलीय पारिस्थितिक तंत्र।

2. पर्यावरण को प्रदूषण से बचाएं।

नुकसान कम करने के उपाय।

तनुकरण। यहां तक ​​कि उपचारित बहिःस्राव को भी 10 बार पतला किया जाना चाहिए, लेकिन
यह विधि अप्रभावी है और केवल अस्थायी उपायों के रूप में ही संभव है।

सफाई। रूस में, यह मुख्य विधि है, हालांकि, सफाई के परिणामस्वरूप
बहुत सारा सांद्रित कचरा उत्पन्न होता है, जिसे होना भी चाहिए
रखना।

पुरानी तकनीकों को नई कम-अपशिष्ट तकनीकों से बदलना। एक की बर्बादी
उत्पादन दूसरे के लिए कच्चा माल बन जाता है।

उपचार की सुविधा।

एमपीसी - अधिकतम स्वीकार्य एकाग्रता।

एमपीई समय की प्रति यूनिट अधिकतम स्वीकार्य उत्सर्जन है जिस पर
प्रदूषक की सांद्रता एमपीसी से अधिक नहीं होगी।

यांत्रिक सफाई। तरल बहिःस्राव जम जाता है, जबकि ठोस खड़ा रहता है
कण। रेत और रेत-बजरी फिल्टर लागू करें, देरी से
हल्के निलंबित कण जो अवसादन टैंकों में नहीं बसे हैं। आवेदन करना
कभी-कभी सेंट्रीफ्यूजेशन। यांत्रिक रूप से अलग तेल उत्पाद जो सामने आए हैं
नाबदान में। गैस उत्सर्जन को साफ करने के लिए, विशेष
धूल अवसादन कक्ष और सेंट्रीफ्यूज (चक्रवात), कपड़ा फिल्टर।

रासायनिक सफाई। बहिःस्राव रसायनों से प्रभावित होता है
घुलनशील यौगिकों को अघुलनशील में परिवर्तित करना। प्रति
ऑक्साइड और हाइड्रोजन सल्फाइड के उत्सर्जन को कम करें, क्षारीय वर्षा लागू करें, के माध्यम से
जो गैस से भरपूर उत्सर्जन पास करता है, जिसके परिणामस्वरूप नमक होता है
और पानी। अवशोषक-फिल्टर विशेष का उपयोग करते हैं
adsorbents: सक्रिय कार्बन, एल्यूमीनियम जेल, सिलिका जेल, रेजिन - आयन एक्सचेंजर्स।


जटिल यौगिकों को सरल में परिवर्तित करते हैं और धातुओं, अम्लों आदि को निकालते हैं।
अकार्बनिक यौगिक। सबसे खतरनाक या मूल्यवान को उजागर करने के लिए
आगे की प्रक्रिया के लिए उपयोग किए जाने वाले दूषित पदार्थों का उपयोग किया जाता है
आयन-विनिमय रेजिन, स्पंज की तरह जो इन पदार्थों को अवशोषित करते हैं।

घरेलू कचरे के प्रसंस्करण में अग्नि विधि का उपयोग किया जाता है।

जैविक सफाई। विशेष रूप से डिजाइन किए गए पारिस्थितिक तंत्र में, प्रदूषक
सूक्ष्मजीवों और छोटे जानवरों द्वारा विस्तारित या केंद्रित।
जीव भारी धातुओं और रेडियोधर्मी को जमा और अवक्षेपित कर सकते हैं
आइसोटोप (विशेषकर डायटोप शैवाल इसे सफलतापूर्वक करते हैं)।

1. जीवों के व्यवहार के प्रकार।

विभिन्न प्रकार के पौधे और जानवर जीवित रहने के तरीकों में भिन्न होते हैं -
जीव रणनीतियाँ। जीवों को तीन समूहों में बांटा गया है, जो लाक्षणिक रूप से हैं
"शेर", "ऊंट" और "गीदड़" कहा जाता है।

जीवों के आलंकारिक नाम आवास प्रतियोगिता का स्तर
कार्यात्मक और वास्तविक निचे का सहसंबंध जीवों के उदाहरण

"शेर" अनुकूल परिस्थितियों में रहते हैं मजबूत प्रतियोगी आमतौर पर
कार्यात्मक और वास्तविक निचे शेर, बाघ, हाथी, ओक से मेल खाते हैं।

ऊंट संसाधन-गरीब वातावरण में रहते हैं कोई प्रतिस्पर्धी स्तर नहीं
निचे मैच ऊंट, कैक्टस, पंख घास

"गीदड़" संसाधनों की प्रचुरता वाले वातावरण में रहते हैं, जैसे "शेर" कमजोर
प्रतिस्पर्धात्मकता कार्यात्मक आला अधिक है, लेकिन ज्यादा नहीं लार्वा
मक्खियाँ, सियार, खेत के पौधे।

2. रेडियोधर्मी प्रदूषण।

दुर्घटनाओं या लापरवाह भंडारण के परिणामस्वरूप ऐसा संदूषण संभव है।
परमाणु आवेशों के विस्फोट के दौरान परमाणु ऊर्जा संयंत्रों में अपशिष्ट भूमिगत बनाने के लिए
भंडारण सुविधाओं या परमाणु हथियारों के उत्पादन में। विकिरण दूषित
सबसे बड़ी दुर्घटनाओं के स्थलों के आसपास विशाल प्रदेश - चेरनोबिल और
किश्तिम्स्काया। लैंडफिल में प्रवेश करने पर विकिरण संदूषण संभव है
रेडियोधर्मी समस्थानिक युक्त पुराने उपकरण। ऐसे उपकरण अक्सर होते हैं
उद्योग, वैज्ञानिक प्रयोगशालाओं, चिकित्सा में उपयोग किया जाता है।
जीवन के अंतिम उपकरणों या संपूर्ण उपकरणों से रेडियोधर्मी पदार्थ
विशेष भंडारण सुविधाओं में संग्रहित किया जाना चाहिए जहां उनका विकिरण नहीं लाता है
पर्यावरण और मनुष्यों को नुकसान।

टिकट #10

1. खाद्य श्रृंखला और खाद्य जाले। पारिस्थितिक पिरामिड।

पोषण की प्रक्रिया में जुड़े विभिन्न पोषी समूहों के जीव और
ऊर्जा हस्तांतरण खाद्य श्रृंखला बनाते हैं।

जड़ी बूटी - मृग - शेर।

पत्तियां - कैटरपिलर - गौरैया - चील।

पूरी श्रृंखला: पौधे, फाइटोफेज, जूफेज।

छोटी श्रृंखला: यदि लिंक में से कोई एक गुम है।

डेट्राइटल चेन: गिरी हुई पत्तियाँ - केंचुआ - गौरैया - परभक्षी
चिड़िया।

डेट्राइटल खाद्य श्रृंखलाएं डेट्रिटोफेज और डीकंपोजर से जुड़ी श्रृंखलाएं हैं,
मृत कार्बनिक पदार्थ का उपयोग करना।

डेट्रिटस श्रृंखला में भाग ले सकता है, जैसे एक कड़ी जो मृतकों को खाती है
कार्बनिक पदार्थ, साथ ही एक लिंक जो ज़ोफेज द्वारा खाया जाता है।

वास्तविक प्रकृति में, खाद्य श्रृंखलाएं नहीं बनती हैं, बल्कि खाद्य जाले बनते हैं।







बायोमास।



पारिस्थितिक तंत्र - फाइटोप्लांकटन।



दूसरे के लिए स्तर।

2. घास का मैदान पारिस्थितिकी तंत्र।

घास के मैदान मानव गतिविधियों का परिणाम हैं। घास का मैदान
पारिस्थितिक तंत्र मिट्टी पर बनते हैं जो स्टेपी की तुलना में बेहतर सिक्त होते हैं
चर्नोज़म्स यदि घास के मैदान का उपयोग बंद कर दिया जाए तो यह फिर से बढ़ जाएगा
वन।

मीडोज नमी से प्यार करने वाले पौधों का एक समुदाय है। वे आम हैं
स्टेपी ज़ोन के तराई भाग में वन क्षेत्र का समतल भाग, अल्पाइन में
पहाड़ों और नदी घाटियों की बेल्ट।

घास के मैदान के प्रकार पौधों के नाम

1. टैगा ज़ोन टोंका तुला घास, लाल फ़ेसबुक, सुगंधित स्पाइकलेट।

2. नदी के बाढ़ के मैदान मेदो फेस्क्यू, मेडो फॉक्सटेल, सोडी पाइक।

3. पर्णपाती वनों और वन-स्टेप का क्षेत्र। संकरी पत्ती वाला पुदीना

4. साइबेरिया के घास के मैदान रेंगने वाले सोफे घास और जौ

5. सुदूर पूर्व बड़े ईख घास के बाढ़ के मैदान Langsdorff

6. माउंटेन मीडोज जूसी फोर्ब्स

7. तराई के घास के मैदान सेज प्रबल होते हैं।

घास के मैदान का अर्थ:

चराई

घास के मैदान

कटाव से मिट्टी की सुरक्षा

घास के मैदानों की उत्पादकता बढ़ाने के तरीके:

1. आमूल-चूल सुधार - प्राकृतिक जड़ी-बूटियाँ पूरी तरह नष्ट हो जाती हैं, और
घास के मैदान को मूल्यवान चारा घास के साथ बोया जाता है और उर्वरक लगाए जाते हैं।

2. सतही सुधार - जड़ी-बूटी, जिसमें बहुत से बहुमूल्य चारे होते हैं
घास को संरक्षित किया जाता है, लेकिन कई प्रकार की फलियां अतिरिक्त रूप से बोई जाती हैं और
अनाज, और उर्वरक लागू करें।

टिकट #11

1. पारिस्थितिक आला। आला प्रकार।

सभी पर्यावरणीय कारकों की समग्रता जिसके भीतर यह संभव है
जनसंख्या के अस्तित्व को पारिस्थितिक आला कहा जाता है।

प्रत्येक प्रजाति के जीवों में दो निचे हो सकते हैं - मौलिक और
क्रियान्वित किया।

एक मौलिक स्थान वे पर्यावरणीय परिस्थितियां हैं जिनमें एक प्रजाति कर सकती है
मौजूद हैं और प्रतिस्पर्धा के अभाव में भी फलते-फूलते हैं।

एक एहसास आला मौलिक जगह का एक हिस्सा है जिसमें वह सक्षम है
प्रतिस्पर्धियों से अलग, यदि कोई हो।

वन क्षेत्र में संकरी-पतली घास (पतली बेंटग्रास) का प्रभुत्व है। वह है
खराब मिट्टी पर उग सकता है। अगर इसके बीज बगीचे में बोए जाते हैं और
उर्वरक, उपज 3 गुना बढ़ जाएगी। यदि आप झुकाव को मंजूरी देते हैं
घास का मैदान, तो यह घास से गायब हो जाएगा। प्रतियोगिता की प्रक्रिया में, इसे बाहर कर दिया जाएगा
चौड़ी पत्ती वाली घास (यूर्चिन टीम) जिस पर उग नहीं सकती
नष्ट हुई मिट्टी।

2. मानव पर्यावरण प्रदूषण का खतरा।

प्रदूषण एक प्रमुख स्वास्थ्य खतरा है
अपार्टमेंट। प्रदूषण का स्रोत हो सकता है:

सिंथेटिक रेजिन के जहरीले स्राव, जो गर्भवती हैं
चिपबोर्ड (फर्नीचर)।

रासायनिक फर्श कवरिंग का वाष्पीकरण (लिनोलियम, पीवीसी फिल्म)

गैस स्टोव और स्टोव में पूरी तरह से जली हुई गैस नहीं।

प्रत्येक मामले में, कम करने के लिए विशिष्ट उपाय किए जाने चाहिए
हानिकारक वायु प्रदूषकों की सांद्रता। फर्नीचर पेंट से ढका हुआ है और
वार्निश जो हानिकारक पदार्थों की रिहाई को कम करते हैं। लिनोलियम नहीं है
बेडरूम में इस्तेमाल किया।

गैस स्टोव के ऊपर एग्जॉस्ट डिवाइस लगाए गए हैं। ढेर सारी धूल
किताबें जमा करते हैं, इसलिए उन्हें नियमित रूप से खाली करने की जरूरत होती है, रखा जाता है
चमकता हुआ अलमारियों और अलमारियाँ में। अपार्टमेंट में अधिक गीली सफाई करें।

हवा और कुछ इनडोर पौधों को साफ करें। वे ऑक्सीजन छोड़ते हैं
फाइटोनसाइड्स का स्राव (उनके गुण रोगजनक को मारते हैं)
सूक्ष्मजीव) औषधीय हाउसप्लांट हैं (मुसब्बर,
कोलांचो)।

टिकट #12

1. जीवों के लंबवत संबंध।

"शिकारी-शिकार" संबंध में, शिकारी तुरंत शिकार को खा जाता है, क्योंकि
इसी पर वह रहता है। चूहे और भेड़िये लोमड़ियों और उल्लुओं द्वारा पकड़े जाते हैं, खरगोश -
लोमड़ियों और लिनेक्स।

सहजीवन पौधों और सूक्ष्मजीवों को बांधता है - बैक्टीरिया और कवक
(सहजीवन), विभिन्न प्रजातियों के कीड़े। सिम्बायोसिस व्यापक है
पौधों और कीट परागणकों के बीच। कवक के साथ पौधों का सहजीवन
वन पारिस्थितिक तंत्र के पारिस्थितिक संतुलन को बनाए रखने के लिए बहुत महत्वपूर्ण है।
मशरूम पेड़ों को फास्फोरस प्रदान करके उन्हें पोषण देने में मदद करते हैं। बिना मदद के
मशरूम, पेड़ के घोड़े फास्फोरस को खराब तरीके से अवशोषित करते हैं, और पेड़ खराब हो जाते हैं।

2. टुंड्रा का पारिस्थितिक तंत्र।

टुंड्रा एक वृक्षविहीन पारिस्थितिकी तंत्र है जो टैगा क्षेत्र के उत्तर में स्थित है।
वर्षा की मात्रा प्रति वर्ष 200 से 500 मिमी तक भिन्न होती है, लेकिन शाश्वत होने के कारण
पर्माफ्रॉस्ट, पानी सतह पर जम जाता है और जलभराव होता है।

टुंड्रा पारिस्थितिकी तंत्र जलभराव पारिस्थितिकी तंत्र हैं।

मिट्टी पीट और टुंड्रा-ग्ली हैं।

उत्पादकता कम है, लेकिन क्षेत्र की विशालता के कारण वे एक बड़ा देते हैं
माध्यमिक जैविक उत्पादों की मात्रा।

उत्पादक (बौने पेड़, झाड़ियाँ, काई, लाइकेन)।

उपभोक्ता (लीविंग, रेनडियर, आर्कटिक लोमड़ियों)

औद्योगिक कारणों से टुंड्रा में पारिस्थितिक संतुलन गड़बड़ा गया है
इन क्षेत्रों का विकास (तेल और गैस उत्पादन)।

इस वजह से ट्रैक किए गए वाहनों के कई मार्गों को भारी क्षति हुई है
वनस्पति आवरण नष्ट हो जाता है और मिट्टी उजागर हो जाती है।

साथ ही, जब तेल निकाला जाता है, तो पारिस्थितिकी तंत्र प्रदूषित होता है।
तेल के पदार्थ।

घरेलू बारहसिंगों के झुंड टुंड्रा को बहुत नुकसान पहुंचाते हैं, जो रोजाना
उसी क्षेत्र में चरना।

टिकट #13

1. पारिस्थितिकी तंत्र: घटक, बातचीत, स्थिरता।

एक पारिस्थितिकी तंत्र जीवित जीवों के परस्पर क्रिया का कोई भी समूह है
और पर्यावरण की स्थिति।

पारिस्थितिक तंत्र की संरचना में जीवित जीव और निर्जीव कारक शामिल हैं -
वातावरण, पानी, पोषक तत्व, प्रकाश और मृत कार्बनिक
पदार्थ अपरद है।

सभी जीवित जीवों को स्वपोषी (उत्पादक) और विषमपोषी में विभाजित किया गया है।
(उपभोक्ता)

स्वपोषी - अकार्बनिक से कार्बनिक पदार्थों का संश्लेषण करते हैं।

विषमपोषी - कार्बनिक पदार्थों का उपयोग करते हैं जिन्हें वे संश्लेषित करते हैं
स्वपोषी।

ऑटोट्रॉफ़्स को दो समूहों में विभाजित किया जाता है: फोटोऑटोट्रॉफ़्स और केमोऑटोट्रॉफ़्स।

फोटोऑटोट्रॉफ़्स कार्बनिक पदार्थों को संश्लेषित करने के लिए सूर्य का उपयोग करते हैं।
ऊर्जा (सभी हरे पौधे)।

केमोआटोट्रॉफ़ संश्लेषण के लिए रासायनिक ऊर्जा का उपयोग करते हैं (सल्फर बैक्टीरिया,
आयरन बैक्टीरिया)।

फाइटोफेज (शाकाहारी) ऐसे जानवर हैं जो पौधों को खाते हैं।

ज़ोफेज शिकारी होते हैं जो जानवरों को खाते हैं।

सिम्बियोट्रोफ़ बैक्टीरिया और कवक हैं जो जड़ों पर फ़ीड करते हैं।
पौधों का स्राव। सूक्ष्मजीव भी सहजीवी होते हैं।
(बैक्टीरिया, एककोशिकीय जंतु) जो पाचक में रहते हैं
जानवरों के पथ - फाइटोफेज, उन्हें भोजन पचाने में मदद करते हैं।

डेट्रिटोफेज - जीव जो मृत कार्बनिक पदार्थों पर फ़ीड करते हैं
(कौवे, चूहे, गोबर भृंग, केंचुए)।

डीकंपोजर - जीव, जो पारिस्थितिकी तंत्र में अपनी स्थिति के करीब हैं
डेट्रोफिग्स, यानी। वे मृत कार्बनिक पदार्थों पर भी भोजन करते हैं।
हालांकि, डीकंपोजर - बैक्टीरिया और कवक कार्बनिक पदार्थों को नष्ट कर देते हैं
खनिज यौगिक जो मिट्टी के घोल में और फिर से लौट आते हैं
पौधों द्वारा उपयोग किया जाता है।

2. दलदली पारिस्थितिकी तंत्र।

दलदल भूमि के ऐसे क्षेत्र होते हैं जहां मिट्टी स्थायी होती है या वर्ष के अधिकांश समय में होती है
जलभराव की स्थिति में हैं, और इसलिए अपरद का निर्माण होता है
पीट का रूप।

अन्य पारिस्थितिक तंत्रों के विपरीत, मृत कार्बनिक पदार्थ अंत तक सड़ते हैं, और
इसलिए ह्यूमस नहीं बनता है।

वनस्पति: घास, झाड़ियाँ और पेड़ जिन्हें के अनुकूल बनाया गया है
जलभराव वाली मिट्टी (ब्लैक एल्डर, सेज, ईख)।

दलदल के प्रकार:

झील के पारिस्थितिक तंत्र के अतिवृद्धि के उत्तराधिकार के दौरान निचले दलदल उत्पन्न होते हैं,
या दलदली भूमि। मुख्य पीट फॉर्मर्स विभिन्न सेज हैं,
ईख, कैटेल, ईख।

संक्रमणकालीन दलदल तराई दलदल की पीट परत की मोटाई
साल दर साल बढ़ता है (40-50 सेमी)। पौधे भोजन के लिए स्विच करते हैं
पीट का खाता, जो जीवों द्वारा खनिज किया जाता है। पीट की परत जितनी शक्तिशाली होगी,
इसकी ऊपरी परत जितनी खराब होगी। इसके तहत पारिस्थितिक उत्तराधिकार का कारण बनता है
आंतरिक कारकों की कार्रवाई। वनस्पति बदल रही है। इसलिए
संक्रमणकालीन दलदल बनते हैं। संक्रमणकालीन दलदल पर घड़ी बढ़ती है
ट्राइफोलिएट, डाउनी बर्च, मार्श कैला, स्फाग्नम मॉस।

अपलैंड दलदल। यदि पीट जमा में वृद्धि जारी है, तो
पीट की ऊपरी परत में खनिजों की मात्रा सम हो जाती है
कम। एक समय आता है जब संक्रमणकालीन दलदल के पौधे नहीं कर सकते
बढ़ना। इस प्रकार उभरे हुए दलदल बनते हैं। वनस्पति: पाइन, क्रैनबेरी,
जंगली मेंहदी, मार्श मर्टल, पॉडबेल, लाइकेन।

दलदल का अर्थ. 1. शरद ऋतु-वसंत की अवधि में, दलदल पानी जमा करते हैं, और में
शुष्क समय धीरे-धीरे इसे धाराओं को दें, जो बदले में फ़ीड करते हैं
नदियाँ।

2. दलदल कृषि पारिस्थितिक तंत्र के आदेश हैं। में बहना
उनके पानी में उर्वरक, कीटनाशक और दलदल से बहने वाली धारा शामिल है
पूरी तरह से साफ।

टिकट #14

1. पारिस्थितिकी तंत्र की जैविक उत्पादकता।

पारिस्थितिक तंत्र उत्पादकता कार्बनिक की मात्रा द्वारा मापी जाती है
पदार्थ जो प्रति इकाई समय प्रति इकाई क्षेत्र में निर्मित होता है,
जैविक उत्पादकता कहते हैं। इकाइयों
उत्पादकता: जी/एम? प्रति दिन, किग्रा / मी? प्रति वर्ष, टी/किमी? साल में।

प्राथमिक जैविक उत्पाद हैं जो उत्पादकों द्वारा बनाए जाते हैं,
और उपभोक्ताओं द्वारा बनाए गए द्वितीयक जैविक उत्पाद और
डीकंपोजर

प्राथमिक उत्पादन में विभाजित है: सकल - यह कुल राशि है
कार्बनिक पदार्थ बनाया, और शुद्ध - यह वही है जो बाद में रहता है
श्वसन और जड़ स्राव पर व्यय।

पारिस्थितिक तंत्र को उनकी उत्पादकता के अनुसार चार वर्गों में बांटा गया है:

अत्यधिक उच्च जैविक उत्पादकता वाले पारिस्थितिक तंत्र - 2 किग्रा/मीटर से अधिक? में
साल। इनमें वोल्गा, डॉन और यूराल के डेल्टा में रीड बेड शामिल हैं।

उच्च उत्पादकता वाले पारिस्थितिक तंत्र - 1-2 किग्रा/मी? साल में। यह लिंडन-ओक है
जंगल, झील पर कैटेल या नरकट के घने जंगल, मकई की फसलें।

औसत जैविक उत्पादकता के पारिस्थितिक तंत्र - 0.25-1 किग्रा/मी? साल में। प्रति
इसमें देवदार, सन्टी वन, घास के मैदान और मैदान शामिल हैं।

कम जैविक उत्पादकता वाले पारिस्थितिक तंत्र - 0.25 किग्रा/मीटर से कम? साल में।

ये आर्कटिक रेगिस्तान, टुंड्रा, अधिकांश समुद्री पारिस्थितिक तंत्र हैं।

पृथ्वी के पारितंत्र की औसत उत्पादकता 0.3 किग्रा/मीटर है? प्रति वर्ष, अर्थात्।
मध्यम और निम्न-उत्पादक पारिस्थितिकी प्रणालियों में पृथ्वी का प्रभुत्व है।

एक पोषी स्तर से दूसरे पोषी स्तर पर जाने पर 90% नष्ट हो जाता है
ऊर्जा।

2. किसी व्यक्ति को पीने का पानी उपलब्ध कराने की समस्या।

बिल्कुल हानिकारक पदार्थ नहीं हैं। कम खुराक पर कोई भी संदूषक
व्यावहारिक रूप से हानिरहित। पानी में हमेशा भारी धातुएं होती हैं। कोई भी मिट्टी
या चट्टान में प्राकृतिक रेडियोधर्मिता है। भी साथ
वातावरण में नाइट्रोजन उत्सर्जित करने वाले आस-पास के उद्यमों की अनुपस्थिति,
इसमें से कुछ वर्षा जल में पाया जाता है। वह में दिखाई देता है
बिजली के निर्वहन का परिणाम, जिसमें अमोनिया बनता है।

हानिकारक प्रदूषकों से वातावरण, जल या मिट्टी की शुद्धि है
उन मूल्यों के प्रति उनकी एकाग्रता में कमी, जिन पर वे बन जाते हैं
हानिरहित। ऐसे थ्रेशोल्ड मान MPC कहलाते हैं - अधिकतम
स्वीकार्य सांद्रता।

जल प्रदूषण को प्रभावी ढंग से नियंत्रित करने के लिए सभी की एमपीसी जानना आवश्यक है
प्रमुख प्रदूषक।

प्रदूषण के प्रत्येक स्रोत के लिए, एमपीई स्थापित है - अधिकतम
समय की प्रति यूनिट स्वीकार्य रिलीज जिस पर एकाग्रता
पानी में प्रदूषक एमपीसी से अधिक नहीं होगा। सफाई के लिए
उपभोक्ता को आपूर्ति किया जाने वाला पेयजल, निलंबन के अवसादन का उपयोग करें,
निस्पंदन, वातन (ऑक्सीजन संवर्धन), जैव रासायनिक प्रक्रियाएं और
क्लोरीनीकरण

प्रत्येक व्यक्ति को प्रतिदिन 2-2.5 लीटर पानी की आवश्यकता होती है। जो पानी आता है
हमारे अपार्टमेंट में, क्लोरीन अवशेष और कई अन्य दूर हैं
उपयोगी सामग्री। उन्हें हटाने के लिए, आपको नल के पानी की आवश्यकता है।
आयन एक्सचेंज फिल्टर से गुजरें।

टिकट #15

पारिस्थितिक पिरामिड।

विभिन्न पोषी स्तरों के जीवों का बायोमास समान नहीं होता है। जमीन में
पारिस्थितिक तंत्र में पोषी स्तर में वृद्धि के साथ, यह घटता है, क्योंकि
एक पोषी स्तर से दूसरे पोषी स्तर में जाने पर ऊर्जा का ह्रास होता है।
विभिन्न पोषी स्तरों के जीवों के बायोमास के अनुपात को दर्शाया गया है
ग्राफिक रूप से बायोमास पिरामिड के रूप में।

स्थलीय पारिस्थितिकी तंत्र। जलीय पारिस्थितिकी तंत्र।

प्रत्येक पोषी स्तर के जीवों के बायोमास को इस प्रकार दर्शाया जाता है:
एक आयत जिसकी लंबाई या क्षेत्रफल संख्या के समानुपाती होता है
बायोमास।

बढ़ते पोषी स्तरों के साथ स्थलीय पारितंत्रों में, बायोमास स्टॉक
घट जाती है, और समुद्री में - बढ़ जाती है। इनमें मुख्य उत्पादक
पारिस्थितिक तंत्र - फाइटोप्लांकटन।

बायोमास के पिरामिडों के अलावा, संख्याओं के पिरामिड भी हैं। उसी का निर्माण करें
ऊर्जा पिरामिड जो एक ट्राफिक से इसके संक्रमण को दर्शाते हैं
दूसरे के लिए स्तर।

2. मीठे पानी का पारिस्थितिकी तंत्र।

मीठे पानी का पारिस्थितिकी तंत्र है नदियाँ, झीलें, तालाब, नदियाँ, अस्थायी
जलकुंड।

जलीय पारिस्थितिकी तंत्र की संरचना की सामान्य योजना दूसरों की तरह ही होती है
पारिस्थितिकी तंत्र

उत्पादकों को पौधों के दो बड़े समूहों में बांटा गया है - मैक्रोफाइट्स (में .)
मुख्य रूप से फूल वाले पौधे) और माइक्रोफाइट्स (सायनोबैक्टीरिया, हरा और)
डायटम)।

मैक्रोफाइट्स:

तैरते हुए पौधे (डकवीड, टेलोरेज़, आदि)

संलग्न जलीय पौधे (पीला पानी लिली, सफेद पानी लिली के साथ
पत्तेदार प्लेटें)।

संलग्न अर्ध-जलीय पौधे (छाता सुसाक, एरोहेड, बुलरुश)
झील, आदि)

माइक्रोफाइट्स पानी में स्वतंत्र रूप से तैरते हैं, शाम को चलते हैं
सतह के करीब, जहां अधिक प्रकाश होता है, और समान रूप से फैलता है
दिन के दौरान पूरे पानी के स्तंभ में।

उपभोक्ताओं का प्रतिनिधित्व प्लवक (एककोशिकीय जंतु, कृमि,
बैक्टीरिया, मछली) और बेन्थोस (बेंटिक जीव, और
नीचे से जुड़े पौधे)।

उपभोक्ताओं में जलपक्षी, ऊदबिलाव, मेंढक भी शामिल हैं।
कछुए

डीकंपोजर बैक्टीरिया हैं।

टिकट #16

1. एक पारिस्थितिकी तंत्र में जनसंख्या: संरचना, बहुतायत, घनत्व।

एक सजातीय क्षेत्र में एक ही प्रजाति के जीव जनसंख्या बनाते हैं।

अधिकांश आबादी में प्रतिस्पर्धा के कारण, एक निश्चित
घनत्व।

जनसंख्या घनत्व प्रति इकाई क्षेत्र में व्यक्तियों की संख्या है। अपर
जनसंख्या घनत्व की सीमा सबसे अधिक कमी की मात्रा द्वारा निर्धारित की जाती है
संसाधन।

जीवों का एक विशेष समूह पौधे हैं - वार्षिक। उनका घनत्व
आबादी लगभग अनिश्चित काल तक बढ़ सकती है। के लिए संभव है
इस तथ्य के कारण कि घनत्व में वृद्धि के साथ, व्यक्ति का आकार घट जाता है।

जनसंख्या विषम हैं।

असमान व्यक्तियों वाली जनसंख्या अधिक स्थिर होती है। ऐसा
पाले की स्थिति में आबादी के फसलों के नष्ट होने की संभावना कम होती है (उदाहरण के लिए,
फूलों का केवल एक हिस्सा जमा देता है, बाकी को रूप में संरक्षित किया जा सकता है
कलियाँ)।

मुकाबला।

जनसंख्या घनत्व स्थिर नहीं है और साल-दर-साल इसमें उतार-चढ़ाव होता है क्योंकि
व्यक्तिगत वर्षों में सबसे दुर्लभ संसाधन की मात्रा स्थिर नहीं है।

शिकार जनसंख्या घनत्व का प्रकोप जनसंख्या घनत्व के प्रकोप का कारण बनता है
शिकारियों क्योंकि शिकारियों को खाने वाले शिकारियों के पास पर्याप्त भोजन होता है।
अतः भोजन के अभाव में इनकी मृत्यु नहीं होती है। और खासकर
आबादी जीवित रहती है। आखिरकार, दुर्लभ संसाधन का स्तर जितना अधिक होगा,
उच्च जनसंख्या घनत्व, अर्थात्। जितने अधिक शिकार, उतने अधिक शिकारी।

गिलहरी की आबादी का घनत्व भोजन की मात्रा पर निर्भर करता है, अर्थात। मैं फ़िन
जंगल में कई शंकु, मशरूम और एकोर्न आदि उगेंगे, तो गिलहरियों की आबादी
वृद्धि होगी। गिलहरी अपनी संतानों को पुन: उत्पन्न करने और खिलाने में सक्षम होगी।

ऐसे कारण जो जनसंख्या की स्थिरता का उल्लंघन करते हैं।

अत्यधिक खनन।

आवास विनाश (पर्यटक आग लगाते हैं, नदियों को प्रदूषित करते हैं)।

एक नई प्रजाति का परिचय। नई प्रजातियां देशी प्रजातियों को नष्ट कर सकती हैं, नष्ट कर सकती हैं
उनकी आबादी।

पर्यावरण प्रदूषण। जलीय पर्यावरण के निवासी इससे सबसे अधिक पीड़ित हैं।
पारिस्थितिकी तंत्र

2. पारिस्थितिक संतुलन में मानवीय हस्तक्षेप के परिणाम।

पारिस्थितिक संतुलन में मानवीय हस्तक्षेप कभी-कभी सबसे अधिक कारण बनते हैं
पारिस्थितिक तंत्र की गंभीर गड़बड़ी।

यहाँ कुछ उदाहरण हैं:

एक शिकारी के पारिस्थितिकी तंत्र से बहिष्करण के परिणाम। - अमेरिका में रक्षा करने के लिए
हिरण भेड़ियों की सामूहिक शूटिंग का आयोजन किया गया। जल्द ही हिरण
गुणा किया जिसने सभी चरागाहों को रौंद डाला। यह भुखमरी और मौत का कारण बना
जानवरों। नतीजतन, हिरणों की आबादी नहीं बढ़ी, बल्कि घट गई।
मुझे भेड़ियों को फिर से पेश करना पड़ा।

एक ऐसी प्रजाति के पारिस्थितिकी तंत्र में शामिल होने के परिणाम जिसके लिए कोई डीकंपोजर नहीं है। पर
गायों को ऑस्ट्रेलिया लाया गया था, लेकिन वे डीकंपोजर जो रहते थे
ऑस्ट्रेलिया, केवल कंगारू खाद को संसाधित करता है। इससे कमी आई
उपयोगी चारागाह क्षेत्र। मुझे गोबर भृंग लाने थे।

पौधों की प्रजातियों के पारिस्थितिकी तंत्र में शामिल करने के परिणाम जो नहीं करते हैं
फाइटोफेज। कांटेदार नाशपाती कैक्टस को ऑस्ट्रेलिया में पेश किया गया था। वह बहुत तेज हो गया
गुणा करें और विशाल प्रदेशों पर कब्जा करें। मुझे एक तितली लेनी थी
कैक्टस की आग।

"काउ डिस्टेंपर" का वायरस अफ्रीका लाया गया था। पहले मरना शुरू किया
घरेलू और फिर जंगली मवेशी। इससे पौधे का संचयन हुआ
मलबे के रूप में जनता, आग अधिक बार हो गई, लकड़ी का प्रतिस्थापन
जड़ी बूटियों और झाड़ियों के साथ वनस्पति। केवल संतुलन बहाल
एक टीका विकसित होने के बाद।

टिकट #17

1. पारिस्थितिक उत्तराधिकार। आंतरिक और के कारण उत्तराधिकार
बाह्य कारक।

पारिस्थितिक उत्तराधिकार एक पारिस्थितिकी तंत्र का क्रमिक परिवर्तन है जिसके अंतर्गत
आंतरिक या बाहरी परिस्थितियों से प्रभावित।

आंतरिक कारकों के प्रभाव में (यदि मिट्टी में बीज हैं या उनके
पड़ोसी क्षेत्रों से वितरण) सड़क के किनारे ऊंचे हो गए हैं, बहाल हो गए हैं
मनुष्य द्वारा नष्ट किए गए प्राकृतिक पारिस्थितिक तंत्र।

इव्नोस्ट। समय के साथ स्केल लाइकेन की एक पतली परत के बजाय
झाड़ी लाइकेन और काई, घास दिखाई देते हैं, और फिर बस जाते हैं
झाड़ियाँ और यहाँ तक कि पेड़ भी।

सभी अपशिष्ट चट्टानों या राख के ढेरों पर स्व-अतिवृद्धि नहीं होती है।
यदि पौधे की वृद्धि के लिए परिस्थितियाँ अनुकूल नहीं हैं, तो डंप का अतिवृद्धि
व्यक्ति मदद करता है। यह डंप की सतह को मिट्टी की एक परत के साथ कवर करता है जिसकी मोटाई
10-20 सेमी और घास, झाड़ियों और पेड़ों के बीज बोता है। ऐसा
गतिविधियों को सुधार कहा जाता है।

बाहरी कारक अक्सर मानवीय गतिविधियों से जुड़े होते हैं। उदाहरण के लिए:

झीलों के पारितंत्रों में, यदि उन्हें खेतों से उर्वरक मिलता है या
औद्योगिक और घरेलू अपशिष्टों से कार्बनिक पदार्थ, रिसाव
उत्तराधिकार। जलीय पारिस्थितिक तंत्र की उत्पादकता और संरचना को बदलने की प्रक्रिया
पोषक तत्वों की अधिक मात्रा के प्रभाव में कहा जाता है
सुपोषण वहीं जलीय पौधे जो केवल में उगते हैं
साफ पानी (हाईलैंडर उभयचर, साल्विनिया तैरता हुआ), और बढ़ता है
प्रदूषित जल पौधे - हॉर्नवॉर्ट, डकवीड।

यदि उत्तराधिकार उत्पन्न करने वाले बाह्य कारकों का प्रभाव समाप्त हो जाता है, तो
आमतौर पर पारिस्थितिकी तंत्र की आत्म-पुनर्प्राप्ति की प्रक्रिया शुरू होती है - उत्तराधिकार,
आंतरिक कारकों के कारण।

2. स्कूली बच्चे व्यावहारिक पर्यावरण में कैसे भाग ले सकते हैं
गतिविधियां।

पर्यावरण के संरक्षण के लिए लड़ रहे सामाजिक आंदोलन। अक्सर
पर्यावरण गैर-सरकारी संगठन कहा जाता है (संक्षिप्त)
इको-एनजीओ)। रूस में पहले इको-एनजीओ छात्र सुरक्षा दल थे
प्रकृति (डीओपी।), जो बीसवीं शताब्दी के शुरुआती 60 के दशक में दिखाई दी। निगरानी रखने वालों
(जैसा कि डीओपी के सदस्यों ने खुद को बुलाया।) रिजर्व के शासन को बनाए रखने में लगे हुए थे
और प्रकृति भंडार, अवैध शिकार के खिलाफ लड़ाई, आदि। तारीख तक
डीओपी आंदोलन छात्र प्रकृति संरक्षण टीमों का एक नेटवर्क है जो मौजूद है
रूस के 40 से अधिक शहरों में। छात्र भी भाग ले सकते हैं
ये गतिविधियाँ।

वार्षिक "मार्च ऑफ़ पार्क्स" अभियान न केवल अतिरिक्त धन लाता है
भंडार और राष्ट्रीय उद्यानों को बनाए रखने के लिए, लेकिन यह भी आकर्षित करता है
पर्यावरण के मुद्दों पर सामान्य ध्यान। इस आयोजन में
छात्र भाग लेते हैं। वे . को समर्पित अपने चित्र प्रदर्शित करते हैं
प्रकृति संरक्षण का विषय।

स्कूली बच्चों को पता होना चाहिए कि पशु आश्रय, बिल,
घोंसले, आदि

जंगल में आकर आप उन पौधों को नहीं फाड़ सकते जो लाल किताब में सूचीबद्ध हैं,
उदाहरण के लिए घाटी की लिली। जंगल में, पर्यटक, पार्किंग स्थल छोड़कर, अवश्य करें
आग बुझानी चाहिए, भले ही बमुश्किल सुलग रही हो
फायरब्रांड और कोयले। हो सके तो भर दें। मुख्य
जंगल की आग का कारण खराब तरीके से बुझाई गई अलाव है।

टिकट #18

1. पर्यावरणीय कारक: वर्गीकरण, सीमित कारक।

पर्यावरणीय कारकों को अजैविक में विभाजित किया गया है - ये निर्जीव कारक हैं।
प्रकृति (तापमान, आर्द्रता), जैविक - ये जीवन के प्रभाव के रूप हैं
एक दूसरे पर जीव, और मानवजनित।

पारिस्थितिक तंत्र की जैविक उत्पादकता जीवमंडल के जीवन का आधार है और
इसके हिस्से के रूप में व्यक्ति। उत्पादकता अत्यधिक निर्भर है
एक संसाधन जो दुर्लभ या प्रचुर मात्रा में है
(उदाहरण के लिए, मिट्टी का जलभराव या उच्च हवा का तापमान)। ऐसा
संसाधन को सीमित (अर्थात सीमित) कारक कहा जाता है। पर
स्टेपी की स्थिति और विशेष रूप से रूस के अर्ध-रेगिस्तानी क्षेत्र (कैस्पियन)
तराई) फसल वर्षा की मात्रा और वन क्षेत्र में सीमित है,
जहाँ पर्याप्त नमी हो, - मिट्टी में पोषक तत्वों की मात्रा। पर
टुंड्रा क्षेत्र में और पहाड़ी क्षेत्रों में, फसल गर्मी की मात्रा से सीमित होती है।

2. कृषि पारिस्थितिकी तंत्र की सामान्य विशेषताएं। मिट्टी है मुख्य संसाधन
कृषि पारिस्थितिकी तंत्र।

कृषि भूमि या कृषि-पारिस्थितिकी तंत्र मानवजनित हैं (अर्थात।
मानव निर्मित) पारिस्थितिक तंत्र। मनुष्य अपनी संरचना निर्धारित करता है और
उत्पादकता: वह भूमि के कुछ भाग को जोतता है और बोता है
कृषि फसलें, साइट पर घास के मैदान और चारागाह बनाती हैं
जंगल, खेत जानवरों को पालते हैं।

कृषि पारिस्थितिक तंत्र स्वपोषी हैं: उनकी ऊर्जा का मुख्य स्रोत सूर्य है।
अतिरिक्त (मानवजनित) ऊर्जा सौर ऊर्जा के 1% से अधिक नहीं होती है
ऊर्जा।

कृषि पारिस्थितिकी तंत्र में उत्पादकों की खेती की जाती है पौधे, घास का मैदान घास और

मिश्रण। खरपतवार भी उत्पादक हैं।

कृषि पारिस्थितिकी तंत्र में अपघटक मुख्यतः जीवाणु होते हैं।

कृषि पारिस्थितिकी तंत्र का मुख्य संसाधन मिट्टी है। इसकी उर्वरता स्टॉक पर निर्भर करती है
कार्बनिक पदार्थ - ह्यूमस, पौधों के लिए उपलब्ध सामग्री
पोषक तत्व, संरचना, नमी की उपलब्धता, प्रतिक्रियाएं
मिट्टी का घोल, उसमें जहरीले नमक आयनों की सामग्री।

ह्यूमस उर्वरता का भंडार है। यह पौधों के अवशेषों से बनता है और
जानवरों। इसका संचय ह्यूमस परत की मोटाई और प्रतिशत पर निर्भर करता है
इसमें ह्यूमस सामग्री। चेरनोज़म मिट्टी की धरण परत की मोटाई 1 . है
मी, और धरण सामग्री 6-10% है; वन मिट्टी की धरण परत की मोटाई -
10-30 सेमी, और धरण सामग्री 2-4% है।

सूक्ष्मजीवों का एक अन्य समूह ह्यूमस से मिट्टी के घोल में छोड़ता है
पोषक तत्व। इनमें मैक्रोन्यूट्रिएंट्स (नाइट्रोजन, फास्फोरस,
पोटेशियम), जो बड़ी मात्रा में पौधों के लिए आवश्यक हैं, और
ट्रेस तत्व (मैंगनीज, बोरॉन, तांबा, जस्ता, आदि) जो पौधों को चाहिए
थोड़ा। सभी पोषक तत्व मिट्टी के घोल से अवशोषित होते हैं
पौधे की जड़ें और फसल के साथ मिट्टी से हटा दिया। अतः यदि मिट्टी
उर्वरक (खाद, खनिज उर्वरक) न लगाएं, फिर तत्वों का भंडार
यह भोजन से बाहर चला जाता है।

मिट्टी की संरचना उन गुच्छों का आकार और आकार है जिनसे यह
तोड़ता है। सबसे अच्छी संरचना बारीक ढेलेदार है।

टिकट #19

1. समय के साथ पारिस्थितिकी तंत्र को बदलना।

जीवन और आंदोलन का अटूट संबंध है। इसलिए जिंदा है
जीवित, कि सभी प्रकार के
प्रक्रियाएं - पहली नज़र में अगोचर, लेकिन जल्दी या बाद में
दृश्य परिवर्तन। उदाहरण के लिए, सेकंड और मिनटों के भीतर,
सूक्ष्मजीवों का कोशिका विभाजन, प्रकाश संश्लेषण की प्रक्रिया चल रही है, बदल रही है
माइक्रॉक्लाइमेट (तापमान, आर्द्रता, पारिस्थितिकी तंत्र के अंदर रोशनी)।

इसके परिणामस्वरूप पौधे और जानवर घंटों के भीतर मर सकते हैं
प्राकृतिक आपदाएं, भोजन की तलाश में जंगली जानवरों और शिकारियों की आवाजाही और
बहुत अधिक।

छोटे कृन्तकों के लिए अक्सर कुछ दिन या सप्ताह पर्याप्त होते हैं
सर्दियों के बाद, वे प्रजनन करते हैं, उनकी संख्या दसियों और सैकड़ों तक बढ़ाते हैं
बार, और फिर बड़े पैमाने पर पलायन किया। इसी तरह मनाया जाता है
उदाहरण के लिए, टुंड्रा में, जहां नींबू पानी, तेजी से गुणा करने और उत्पादन करने में सक्षम
एक छोटी उत्तरी गर्मियों के लिए, कई पीढ़ियाँ सक्रिय रूप से बसती हैं -
वे चौड़ी नदियों में तैरते हैं, कस्बों और शहरों में "जीवित लहरों" में प्रवेश करते हैं।

कुछ पौधे (उनमें से कई शुरुआती फूल वाले, जिन्हें आमतौर पर कहा जाता है
"स्नोड्रॉप्स") अक्सर जल्दी दिखने में एक महीने का समय लेता है
वसंत में, खिलते हैं और बीज पैदा करते हैं या भूमिगत अंगों में भंडार जमा करते हैं
अगले वसंत तक।

कीड़ों की कई प्रजातियों का विकास एक समान तरीके से होता है: in
जंगल और दलदल अचानक बड़ी संख्या में खून चूसने वाले दिखाई देते हैं
कीड़े, लेकिन कुछ ठंडी रातों के बाद वे भी अप्रत्याशित रूप से
गायब, केवल अगले साल फिर से प्रकट होने के लिए।

बेशक, दीर्घकालिक परिवर्तन हैं: मौसमी, दीर्घकालिक,
सदियों पुराना।

2. कृषि प्रदूषण।

कृषि प्रदूषण, औद्योगिक प्रदूषण की तरह, हानिकारक है
पारिस्थितिकी तंत्र यह कई प्रकार में आता है।

1. पशुधन फार्मों और चारागाहों से जल प्रदूषण
परिसरों

खेतों में लगाने के लिए गलत तरीके से तैयार की गई खाद, पानी में गिरना,
प्राकृतिक पारिस्थितिक तंत्र को नुकसान पहुंचा सकता है, अर्थात। वे इससे मर जाते हैं
इसलिए, मिट्टी के बैक्टीरिया ने मिट्टी की उर्वरता को कम कर दिया।

खाद से वातावरण और जल निकायों को प्रदूषित न करने के लिए, वे निर्माण करते हैं
कंक्रीट की दीवारों के साथ खाद का भंडारण।

2. कीटनाशकों और उनके अवशेषों से प्रदूषण।

इससे मृदा की जीवित जनसंख्या का ह्रास होता है - की संख्या
मिट्टी के जानवर, शैवाल, सूक्ष्मजीव। कीटनाशकों के अवशेष
मिट्टी, वातावरण, भोजन में मिलें।

मिट्टी पर लागू कीटनाशकों की मात्रा को विनियमित करना आवश्यक है।

3. भारी धातुओं से मृदा प्रदूषण।

इस प्रकार का प्रदूषण फॉस्फेट उर्वरकों के प्रयोग से जुड़ा है, जो
भारी धातुओं की अशुद्धियाँ होती हैं। वे नकारात्मक रूप से प्रभावित करते हैं
मानव स्वास्थ्य (विशेषकर गुर्दे पर)।

उर्वरकों को उन कारखानों में अच्छी तरह से साफ किया जाना चाहिए जहां वे हैं
उर्वरकों के बैचों में उनकी सामग्री का उत्पादन, और लगातार निगरानी करना
और मिट्टी में।

4. नाइट्रेट से मिट्टी और भोजन का दूषित होना।

एक बार मानव शरीर में भोजन के साथ, वे सामान्य को बाधित करते हैं
संचार प्रणाली का काम, कैंसर के ट्यूमर का कारण बन सकता है।

सब्जियों में नाइट्रेट के संचय को रोकने के लिए, उपयोग को सीमित करना आवश्यक है
खनिज नाइट्रोजन उर्वरक और उनका एक साथ उपयोग करें
जैविक खाद। आप उच्च के साथ फसल नहीं उगा सकते
छायादार स्थानों में नाइट्रेट जमा करने की क्षमता।

टिकट #20

1. पारिस्थितिकी XXI सदी का विज्ञान है।

"पारिस्थितिकी" शब्द पहली बार 1866 में हेकेल द्वारा पेश किया गया था।

पारिस्थितिकी जीवों और पर्यावरणीय परिस्थितियों के बीच संबंधों का विज्ञान है
एक वास।

पारिस्थितिक विज्ञानी जिन्होंने पारिस्थितिकी में महत्वपूर्ण योगदान दिया है:

कार्ल लिनिअस। व्यावहारिक उपयोग के लिए एक सुविधाजनक बनाया गया
पौधों और जानवरों की प्रजातियों का वर्गीकरण और व्यवस्थित जानकारी के बारे में
विभिन्न प्रजातियों की रहने की स्थिति।

जीन बैप्टिस्ट लैमार्क। उन्होंने यह विचार व्यक्त किया कि जीवित और निर्जीव सब कुछ
हमारा ग्रह एक संपूर्ण है - जीवमंडल, और चेतावनी दी
प्रकृति पर मानव प्रभाव के संभावित परिणामों के बारे में मानवता।

थॉमस माल्थस। गणितीय रूप से संख्या में वृद्धि के पैटर्न का वर्णन किया
एक ही प्रजाति के जीवों और, लैमार्क का अनुसरण करते हुए, संभावित गंभीर . का पूर्वानुमान दिया
मानव आर्थिक गतिविधि के परिणाम, यदि इसकी संख्या
सीमा के बिना वृद्धि होगी और अधिक जनसंख्या होगी।

चार्ल्स डार्विन। माल्थस के विचारों के आधार पर उन्होंने प्राकृतिक के सिद्धांत की रचना की
चयन, जो कमजोरों के विलुप्त होने की कीमत पर अधिक जनसंख्या को समाप्त करता है
व्यक्तियों।

में और। वर्नाडस्की ने लैमार्क के विचारों को विकसित किया और का एक समग्र सिद्धांत बनाया
जीवमंडल, जीवों द्वारा निभाई गई भूवैज्ञानिक भूमिका को दर्शाता है
ग्रह का परिवर्तन।

वी.एन. सुकाचेव। सजातीय स्थलीय पारिस्थितिक तंत्र के सिद्धांत का निर्माण किया -
बायोगेकेनोज।

पारिस्थितिकी का मुख्य उद्देश्य एक पारिस्थितिकी तंत्र है - जीवन का एक सेट
जीव और उनका पर्यावरण।

2. मानव स्वास्थ्य के संरक्षण की वैश्विक समस्या।

वैश्विक कारणों से देश में पारिस्थितिक स्थिति बिगड़ रही है
प्रक्रियाएं।

ग्रीनहाउस प्रभाव। जलवायु वार्मिंग। पश्चिमी हवाएँ रूस लाती हैं
यूरोप से औद्योगिक उत्सर्जन से प्रदूषित वायु। वैज्ञानिकों के अनुसार
उन्नीसवीं सदी के अंत और वर्तमान तक, हमारे ग्रह पर तापमान
1.2 डिग्री सेल्सियस की वृद्धि हुई। जलवायु वार्मिंग के कारण हो सकता है
ग्लेशियरों का तीव्र पिघलना और महासागरों का बढ़ता स्तर।

ओजोन परत की कमी। एकाग्रता में धीरे-धीरे वृद्धि होती है
वातावरण में कार्बन डाइऑक्साइड (कार्बन डाइऑक्साइड)। छेद दिखाई देते हैं
ओजोन स्क्रीन जो जानलेवा पराबैंगनी को रोकती है
किरणें। यह प्रक्रिया विशेष रूप से ध्रुवों पर तेजी से होती है, जहां पहले से ही
ओजोन छिद्र दिखाई दिए। ओजोन रिक्तीकरण का मुख्य कारण
फ्रीन्स का उपयोग है। 1990 के दशक में, एक अंतरराष्ट्रीय
एक समझौता जिसके तहत फ्रीन्स का उत्पादन 2 गुना कम कर दिया गया था।

इसमें वर्षों की कड़ी मेहनत, बड़े निवेश और सुविचारित प्रयास लगते हैं
इन पर्यावरणीय समस्याओं को हल करने के लिए कार्यक्रम। इसीलिए
रूसी सरकार एक विशेष दीर्घकालिक कार्यक्रम विकसित कर रही है
"रूस की पारिस्थितिक सुरक्षा"।

टिकट #21

1. जैविक प्रेरण।

जीवित जीव पर्यावरण की स्थिति से निकटता से संबंधित हैं। और इसलिए परिवर्तनों के बारे में
ये स्थितियां - प्रदूषण, मिट्टी की नमी में वृद्धि या कमी, इसकी
लवणता, जलवायु परिवर्तन, आदि को अक्सर प्रतिक्रिया द्वारा आंका जा सकता है
व्यक्तिगत जीव और उनकी आबादी या पारिस्थितिक तंत्र की संरचना द्वारा।

जीवों की स्थिति और पारिस्थितिक तंत्र की प्रजातियों की संरचना के अनुसार पर्यावरण का आकलन
जैविक संकेत कहा जाता है।

विभिन्न जैविक संकेतक हैं।

कुछ प्रदूषकों की उपस्थिति का अंदाजा बाहरी संकेतों से लगाया जा सकता है
पौधे और पशु। उदाहरण के लिए, लिंडन के पत्तों पर काले धब्बे का दिखना
इस तथ्य के बारे में बात करें कि सर्दियों में, चौकीदारों को अत्यधिक दूर ले जाया जाता था
बर्फ के पिघलने में तेजी लाने के लिए उस पर नमक छिड़कना।

कुछ कारकों के प्रभाव को पत्तियों के आकार की ख़ासियत से आंका जा सकता है।
या पौधे की ऊंचाई।

पौधे हैं - कृषि योग्य भूमि पर मिट्टी की स्थिति के संकेतक। अगर टकसाल दिखाई दिया
खेत, जिसका अर्थ है कि पानी कृषि योग्य भूमि पर स्थिर हो जाता है।

पौधों की संरचना के अनुसार - खरपतवार, आवेदन की तीव्रता का आकलन करना संभव है
शाकनाशी

वायुमंडलीय प्रदूषण के सूक्ष्म संकेतक - कुछ प्रकार के काई और
लाइकेन

विशेष जीवित उपकरण भी हैं - ब्रियोमीटर - छोटे बक्से के साथ
कुछ प्रकार के काई।

बेशक, जैविक संकेतक विस्तृत विश्लेषणों की जगह नहीं लेंगे। हालांकि
कम, कई मामलों में पर्यावरणीय कारकों के प्रभाव का आकलन
जैव संकेत विधियाँ बहुत उपयोगी हैं।

2. कृषि में ऊर्जा की बचत।

कृषि-पारिस्थितिकी विज्ञान किससे उच्च पैदावार प्राप्त करने के तरीके विकसित करता है
मानवजनित ऊर्जा की लागत को कम करना (पर्यावरण पर कोई प्रभाव)
मानव गतिविधियों से जुड़ा पर्यावरण)।

कृषि में ऊर्जा की बचत के मुख्य तरीके:

फसलों का उचित स्थान। क्या बेहतर फिट
एक पौधे के लिए बढ़ती परिस्थितियाँ, वह जितना अधिक उत्पादक होता है और उसकी लागत उतनी ही कम होती है
एक फसल प्राप्त करना।

बढ़ी हुई प्रतिस्पर्धी शक्ति वाली किस्मों का चयन और
सूखा प्रतिरोध, रोगों और कीड़ों का प्रतिरोध -
कीट सूखा प्रतिरोधी किस्मों को पानी देने की आवश्यकता नहीं है, वे किस्में जो रक्षा करती हैं
खुद को फंगल रोगों या कीटों से, उपयोग की आवश्यकता नहीं है
कीटनाशकों, इसलिए पानी की आपूर्ति के लिए ऊर्जा लागत की बचत,
कीटनाशकों का उपयोग।

मिश्रित मिट्टी। इनमें विभिन्न पारिस्थितिक निचे के पौधे शामिल हैं।
- कमोबेश सूखा प्रतिरोधी, गहरी जड़ प्रणाली के साथ और
सतही रूप से कठोर, उच्च और निम्न। ऐसी फसलें हैं
प्रकाश और मिट्टी के संसाधनों का अधिक पूर्ण उपयोग, अधिक उत्पादक और कम
मातम से भरा हुआ है, क्योंकि उनके पास निपटान के लिए कम खाली जगह है
खरपतवार का पौधा।

पशुपालन की ऊर्जा दक्षता में सुधार। उत्पादन के लिए
ऊर्जा खर्च की जाती है, और इसलिए उन प्रजातियों को वरीयता दी जाती है
खेत के जानवर और वे नस्लें जो उत्पादन के लिए हैं
उत्पादन की इकाइयाँ कम फ़ीड की खपत करती हैं।

टिकट #22

1. रूस में पर्यावरणीय समस्याएं।

आंतों, जंगलों और मिट्टी का क्षरण। इसका कारण वनों का अत्यधिक उपयोग और
धरती। इस समस्या को हल करने के लिए, आपको उपयोग करना बंद करना होगा और
पुनर्जन्म का मौका दें।

नदी और समुद्री क्षेत्रों का प्रदूषण। वजह है कचरे का निस्तारण
समुद्र की मिट्टी। रिलीज को साफ और वैध बनाना आवश्यक है।

शहरों में उद्योग का अत्यधिक संकेंद्रण, अप्रचलित
उद्यमों में उपकरणों के कारण भारी उत्सर्जन हुआ
प्रदूषकों की मात्रा।

परमाणु ऊर्जा संयंत्र और भंडारण परियोजनाओं की अपर्याप्त विश्वसनीयता
खर्च किए गए रेडियोधर्मी ईंधन ने भयानक आपदाएँ पैदा कीं।

गैस और तेल पाइपलाइनों का गलत तरीके से बिछाना। नतीजतन
लाख हेक्टेयर टुंड्रा और वन टुंड्रा नष्ट हो गए हैं।

रूस और सभी मानव जाति के लिए समस्याएं:

ओजोन परत की कमी। ओजोन रिक्तीकरण का मुख्य कारण
फ्रीन्स का उपयोग है। 1988 में, एक अंतरराष्ट्रीय
एक समझौता जिसने 2000 तक फ्रीऑन उत्पादन में कटौती की
50% द्वारा।

ग्रीनहाउस प्रभाव। यह उद्योग के तेजी से विकास के कारण है।
अनिवार्य रूप से, समुद्र के स्तर में वृद्धि और 2 से 4 . तक जलवायु वार्मिंग
डिग्री। नतीजतन, ग्लेशियर पिघलना शुरू हो जाएंगे, जिससे
बाढ़, जिससे भूमि के बड़े क्षेत्रों में बाढ़ आ जाएगी।

रूसी सरकार एक विशेष दीर्घकालिक कार्यक्रम विकसित कर रही है
इन समस्याओं को खत्म करने के लिए "रूस की पर्यावरण सुरक्षा"।

2. रासायनिक प्रदूषण से होने वाले नुकसान को कम करने के तरीके। इलाज
संरचनाएं।

सबसे आम प्रदूषण - रासायनिक। तीन मुख्य हैं
उनके नुकसान को कम करने के तरीके।

से होने वाले नुकसान को कम करने के लिए प्रदूषण एक अप्रभावी तरीका है
प्रदूषण, केवल एक अस्थायी उपाय के रूप में स्वीकार्य।

शुद्धिकरण पर्यावरण उत्सर्जन को कम करने का मुख्य तरीका है
रूस आज।

पुरानी तकनीकों को नई कम-अपशिष्ट तकनीकों से बदलना। विकास करना
विशेष आर्थिक तंत्र। एक उत्पादन से अपशिष्ट
दूसरों के लिए कच्चा माल बन जाते हैं।

रूस में पर्यावरण की स्थिति में कुछ सुधार प्राप्त किया गया है
मुख्य रूप से उपचार सुविधाओं के काम में सुधार के कारण।

निम्नलिखित प्रकार की उपचार सुविधाएं हैं:

यांत्रिक सफाई। यह सफाई रेत का उपयोग करती है और
रेत और बजरी फिल्टर, जो निलंबित कणों को बनाए रखते हैं जो अंदर नहीं बसे हैं
बसने वाले टैंक। तेल उत्पादों को यंत्रवत् रूप से अलग किया जाता है।

रासायनिक सफाई। इस उपचार के दौरान बहिःस्राव रसायन से प्रभावित होते हैं
घुलनशील यौगिकों को अघुलनशील में परिवर्तित करने वाले पदार्थ।

भौतिक और रासायनिक सफाई। इलेक्ट्रोलिसिस द्वारा यह शुद्धिकरण परिवर्तित होता है
जटिल यौगिकों को सरल में परिवर्तित करते हैं और धातु, अम्ल आदि निकालते हैं।

जैविक सफाई। इस दौरान सफाई विशेष रूप से बनाई गई
पारिस्थितिक तंत्र, प्रदूषक सूक्ष्मजीवों द्वारा नष्ट हो जाते हैं और छोटे
जानवरों। जैविक उपचार विधियां अत्यंत महत्वपूर्ण हैं क्योंकि
प्रदूषकों का एक महत्वपूर्ण अनुपात जिसे फ़िल्टर नहीं किया जा सकता है,
इलेक्ट्रोलिसिस द्वारा हटाया नहीं जा सकता - ये पानी में घुलने वाले कार्बनिक पदार्थ हैं
पदार्थ।

टिकट #23

1. पारिस्थितिक तंत्र के प्रकार।

पारिस्थितिक तंत्र बहुत विविध हैं। उनकी रचना कई कारकों पर निर्भर करती है,
मुख्य रूप से जलवायु, भूवैज्ञानिक स्थितियों और मानव प्रभाव पर। वे हैं
स्वपोषी हो सकता है यदि मुख्य भूमिका स्वपोषी जीवों द्वारा निभाई जाए
- उत्पादक, या विषमपोषी यदि पारिस्थितिकी तंत्र में कोई उत्पादक नहीं हैं, या
उनकी भूमिका नगण्य है।

पारिस्थितिक तंत्र प्राकृतिक या मानव निर्मित हो सकते हैं -
मानवजनित।

प्राकृतिक (प्राकृतिक) पारिस्थितिक तंत्र प्राकृतिक के प्रभाव में बनते हैं
कारक, हालांकि एक व्यक्ति उन्हें प्रभावित कर सकता है।

मानव द्वारा मानवजनित (कृत्रिम) पारितंत्रों का निर्माण इस प्रक्रिया में किया जाता है
आर्थिक गतिविधि। उनके उदाहरण: कृषि परिदृश्य के साथ
फसलों और पशुओं के झुंड, शहर, वन वृक्षारोपण, आदि।

प्राकृतिक और मानवजनित दोनों पारिस्थितिक तंत्र अपने स्रोत में भिन्न हैं
ऊर्जा जो उन्हें जीवित रखती है।

स्वपोषी पारितंत्र ऊर्जा आत्मनिर्भर हैं और
फोटोऑटोट्रॉफ़िक में विभाजित - किसके कारण सौर ऊर्जा की खपत होती है
निर्माता - फोटोऑटोट्रॉफ़्स, और केमोऑटोट्रॉफ़िक - का उपयोग कर
उत्पादकों के कारण रासायनिक ऊर्जा - कीमोआटोट्रॉफ़्स।

विषमपोषी पारिस्थितिक तंत्र उन्हें प्राप्त होने वाली रासायनिक ऊर्जा का उपयोग करते हैं
कार्बनिक पदार्थ या मानव निर्मित कार्बन के साथ मिलकर
ऊर्जा उपकरण। उदाहरण: समुद्र की गहराई का एक पारिस्थितिकी तंत्र, जहां
सूरज की रोशनी नहीं पहुँचती।

विषमपोषी मानवजनित पारिस्थितिक तंत्र में भी शामिल हैं:

1.जैविक उपचार सुविधाएं,

2.केंचुआ प्रजनन कारखाने,

3. शैंपेन के बागान,

4. शहरों में मछली तालाब।

2. मानव रोग के लिए जोखिम कारक।

बीसवीं सदी में, मानव जाति ने पूरी तरह से महसूस किया कि कई बीमारियां
वायु प्रदूषण, खराब जल आपूर्ति,
खराब गुणवत्ता वाले उत्पाद।

कोई भी वायु प्रदूषण लोगों के स्वास्थ्य को प्रभावित करता है। उदाहरण के लिए, में
रूस में, औद्योगिक उद्यमों और घरेलू बॉयलर हाउसों को फेंक दिया जाता है
वातावरण 19 मिलियन टन, और वाहन - अन्य 11 मिलियन टन प्रदूषक
पदार्थ। इसका मतलब है कि रूस में हर शहरवासी के लिए
लगभग 275 किलोग्राम हानिकारक पदार्थ होते हैं। शहरी आबादी के बीच,
विशेष रूप से बच्चों में, वृद्धि के कारण होने वाली बीमारियों का एक उच्च अनुपात है
वायु प्रदुषण।

भारी मात्रा में प्रदूषक पर्यावरण में छोड़े जाते हैं
तकनीकी सहायता प्रणालियों में दुर्घटनाओं या विफलताओं के परिणामस्वरूप पर्यावरण।

नाइट्रोजन उर्वरकों के कृषि क्षेत्रों से धुलाई काफी बढ़ जाती है
पानी में हानिरहित नाइट्रेट की सामग्री, जिसे परिवर्तित किया जा सकता है
नाइट्राइट्स लेकिन वे वास्तव में खतरनाक हैं: रक्त में एक बार, नाइट्राइट्स
हीमोग्लोबिन (पूरे शरीर में ऑक्सीजन ले जाने वाला प्रोटीन) और
जिससे रक्त की मुख्य कार्य करने की क्षमता में तेजी से कमी आती है।
कार्य - अंगों और ऊतकों को ऑक्सीजन की आपूर्ति करना।

बारिश और धूल के साथ वातावरण में छोड़ा गया औद्योगिक उत्सर्जन
पृथ्वी की सतह पर लौटें, धीरे-धीरे मिट्टी में जमा हो रही हैं।
स्वास्थ्य के लिए हानिकारक पदार्थ - आर्सेनिक, लेड, मरकरी, कैडमियम, जिंक,
क्रोमियम, निकल, तांबा, कोबाल्ट - भूजल के साथ प्रवेश कर सकते हैं
स्थानीय पेयजल स्रोत। लेकिन सबसे खतरनाक संक्रमण
भोजन में मिट्टी से दूषित पदार्थ।

धातुकर्म संयंत्रों से उत्सर्जन के क्षेत्र में लोगों के लिए,
सीसा विशेष रूप से खतरनाक है। यह जठरांत्र के माध्यम से शरीर में प्रवेश करता है
पथ या फेफड़े, रक्तप्रवाह में प्रवेश करते हैं और इसे पूरे शरीर में फैलाते हैं,
हड्डियों, मांसपेशियों, यकृत, गुर्दे, हृदय, लसीका में जमा होना
नोड्स। सीसा विषाक्तता, प्रारंभिक अवस्था में भी, सिर को प्रभावित करती है।
मस्तिष्क, जिसके परिणामस्वरूप बच्चों में बुद्धि की कमी होती है, बिगड़ा हुआ
आंदोलनों, श्रवण और स्मृति का समन्वय बिगड़ जाता है।

टिकट #24।

जनसंख्या-विशिष्ट सुरक्षा का स्तर।

इस स्तर पर, संरक्षण की वस्तुएं विशिष्ट पौधों की प्रजातियां हैं या
आबादी में रहने वाले जानवर।

सुरक्षा को व्यवस्थित करने के लिए, वे वस्तुओं की पहचान करते हैं और Red . बनाते हैं
पुस्तकें। पहली रेड बुक 1966 (5 खंड) में छपी।

आरएसएफएसआर (पौधों) की लाल किताब 1988 (533 प्रजातियों) में प्रकाशित हुई थी।

आरएसएफएसआर (जानवरों) की लाल किताब 1985 (247 प्रजातियां) में प्रकाशित हुई थी।

जनसंख्या-प्रजाति के स्तर पर संरक्षण किसके द्वारा किया जाता है:

फूल वाले पौधों (वेनेरेन, स्लिपर) के संग्रह पर प्रतिबंध।

दुर्लभ औषधीय जड़ी बूटियों के संग्रह पर प्रतिबंध (वेलेरियन ऑफिसिनैलिस)।

पक्षियों और जानवरों की दुर्लभ प्रजातियों (क्रेन, हंस, बाघ) के शिकार पर प्रतिबंध।

कुछ प्रकार की मछलियों (स्टर्जन), तितलियों और भृंगों को पकड़ने पर प्रतिबंध।

अत्यधिक शिकार का निषेध (आर्कटिक लोमड़ी, सेबल)।

आवास विनाश निषिद्ध है।

नई प्रजातियों (सबसे खूबसूरत पंख वाली घास) की शुरूआत पर प्रतिबंध।

प्रदूषण निषेध

समस्या को हल करने के तरीके:

मानव नियंत्रण में प्रजनन प्रजातियां।

जानवरों को चिड़ियाघरों, पौधों - वनस्पति उद्यानों में पाला जाता है। अस्तित्व
दुर्लभ प्रजातियों का प्रजनन।

2. जीन बैंकों का निर्माण।

जार पौधों के बीज और जमे हुए दोनों को स्टोर कर सकते हैं
ऊतक संस्कृतियों या रोगाणु कोशिकाओं (जमे हुए शुक्राणु को स्टोर करने की अधिक संभावना),
जिससे जंतु और पौधे प्राप्त किए जा सकते हैं।

2. ऊर्जा के विकास की संभावनाएं।

आज उद्योग और कृषि के लिए ऊर्जा के प्रमुख स्रोत
अर्थव्यवस्था - कोयला, तेल, गैस के संपूर्ण संसाधन। ऊर्जा से प्राप्त होती है
परमाणु ऊर्जा संयंत्र, हाइड्रोइलेक्ट्रिक पावर स्टेशन, ज्वारीय भू-तापीय, सौर और अन्य स्टेशन।

1. पारंपरिक थर्मल पावर इंजीनियरिंग का विकास (कोयले पर, जिसके भंडार
काफी बडा)। लगभग पारिस्थितिक रूप से प्राप्त करने की संभावना
जलविद्युत ऊर्जा संयंत्रों से स्वच्छ ऊर्जा शक्तिशाली फिल्टर का उपयोग करके जो देरी करते हैं
वायुमंडलीय प्रदूषक, और भूमिगत कोयला गैसीकरण से। गलती -
वातावरण और अस्थायी प्रकृति में कार्बन डाइऑक्साइड की एक बड़ी मात्रा
यह रणनीति। कोयला भंडार 100-150 साल तक चलेगा।

2. परमाणु ऊर्जा का विकास।

सुरक्षा में सुधार और नए प्रकार के रिएक्टरों की उपस्थिति के साथ
प्रतिरोधी में कचरे के गहन निपटान की सुरक्षा सुनिश्चित करना
चट्टानों का विनाश। परमाणु ऊर्जा की संभावनाएं निर्भर करेंगी
नए प्रकार के परमाणु रिएक्टर कितने सुरक्षित होंगे और
प्रतिक्रियाशील कचरे के निपटान के तरीके।

लाभ - वातावरण में कार्बन डाइऑक्साइड उत्सर्जन में कमी।

3. गैर-पारंपरिक ऊर्जा स्रोत - सूर्य, हवा, महासागर
ज्वार, पृथ्वी की गहराई की गर्मी। भविष्य विज्ञानी - पर्यावरणविद मानते हैं कि 2030 -
2050 तक, गैर-पारंपरिक ऊर्जा स्रोत मुख्य होंगे।

टिकट #25

1. ग्रह की जनसंख्या को विनियमित करने की समस्याएं।

आज, विश्व की जनसंख्या 2030 तक 5 बिलियन लोगों को पार कर गई है
ग्रह पर लोगों की संख्या दोगुनी होने की उम्मीद है। हम पहले से ही जानते हैं कि दो शताब्दियां
बैक अर्थशास्त्री टी। माल्थस ने खतरनाक पर्यावरणीय परिणामों की भविष्यवाणी की
ग्रह की अधिक जनसंख्या। आज वे काफी स्पष्ट हैं।

तीव्र जनसंख्या वृद्धि वाले क्षेत्रों में, जीवन प्रत्याशा
लोगों की संख्या घटकर 45 - 50 वर्ष हो गई, शिशु मृत्यु दर 200 मौतों तक पहुंच गई
प्रति हजार जन्म। उन क्षेत्रों में जहां जनसंख्या वृद्धि रुक ​​गई है और
एक या दो बच्चों वाले परिवार प्रमुख हैं, औसत जीवन प्रत्याशा
70 वर्ष है, और शिशु मृत्यु दर प्रति हजार जन्म नहीं है
5-10 से अधिक लोग।

अत्यधिक जनसंख्या प्रकृति पर दबाव डालती है। यह गरीबी और भूख की ओर ले जाता है।

वैज्ञानिक इस समस्या को हल करने के लिए एक योजना पेश करते हैं:

"प्रकृति के बिना एक दुनिया"

"एक परिवार - एक बच्चा"।

दोनों चरम अस्वीकार्य हैं। पारिस्थितिक विज्ञानी-यथार्थवादी इसे संभव मानते हैं
8 अरब लोगों के स्तर पर विश्व जनसंख्या का स्थिरीकरण।

"प्रकृति के बिना एक दुनिया बेजान है, एक व्यक्ति सभी का प्रबंधन करने में सक्षम नहीं है"
सबसे शक्तिशाली कंप्यूटरों का उपयोग करते हुए भी बायोस्फेरिक प्रक्रियाएं।
उसे प्रकृति को नहीं बदलना चाहिए, बल्कि उसमें सहयोग करना चाहिए।

"एक परिवार - एक बच्चा" योजना का पालन करने के लिए सभी को बाध्य करें
असंभव है, और हर कोई स्वेच्छा से इसे स्वीकार नहीं करना चाहेगा।
जनसंख्या वृद्धि को सीमित करने के लिए
जिन देशों में आज पहले से ही भीड़ है, वहां शिक्षा का स्तर बढ़ाना जरूरी है
जनसंख्या और जनसांख्यिकीय रूप से स्थिर देशों की जीवन स्थितियों में सुधार करना
आव्रजन नीति को कड़ा करना चाहिए, अर्थात। निवासियों के प्रवेश को प्रतिबंधित करें
जनसांख्यिकीय रूप से वंचित देशों से।

2. रूस के वन - एक राष्ट्रीय खजाना, वनों की कटाई।

वन हमारी मातृभूमि की दौलत है। रूस में, 42% क्षेत्र पर वनों का कब्जा है।

वन मानव आर्थिक गतिविधि का एक महत्वपूर्ण उद्देश्य हैं। अंतर करना
प्राथमिक और माध्यमिक वन प्रबंधन। वनों में मुख्य वन प्रबंधन के साथ
लकड़ी की कटाई की जा रही है। भंडार के मामले में रूस दुनिया में पहले स्थान पर है
लकड़ी। एक साइड इफेक्ट के साथ - मशरूम और जामुन, औषधीय पौधे। जंगल
रूस नट, जामुन (रसभरी, ब्लैकबेरी, स्ट्रॉबेरी, ब्लूबेरी,
ब्लूबेरी, लिंगोनबेरी, क्रैनबेरी, आदि), मशरूम, पाइन नट्स, जो
मानव स्वास्थ्य के लिए लाभकारी गुणों का एक दुर्लभ परिसर है।

औषधीय पौधों की 600 प्रजातियों में से जो इस क्षेत्र में काटी जाती हैं
रूस, एक तिहाई वन प्रजातियां हैं, जिनमें 10 पेड़ प्रजातियां शामिल हैं

एक वयस्क लिंडन का पेड़ 70 किलो शहद "देता है"। लिंडन के जंगलों में समृद्ध
सुदूर पूर्व और बश्कोर्तोस्तान। इन वनों को संरक्षित और गुणा करने की आवश्यकता है।

वनों में सबसे समृद्ध क्षेत्र साइबेरिया है।

माध्यमिक वन प्रबंधन में मूस, जंगली सूअर, खरगोश का शिकार करना शामिल है।
मार्टेंस, कॉलम, सपेराकैली, हेज़ल ग्राउज़ और अन्य खेल जानवर।

वन एक मनोरंजक भूमिका निभाते हैं - यह शहरी और ग्रामीण के लिए विश्राम का स्थान है
आबादी। ऑक्सीजन से भरपूर और हीलिंग स्रावों से भरपूर
पौधे वन - स्वास्थ्य का स्रोत

वन हमारी संपत्ति हैं और इन्हें संरक्षित किया जाना चाहिए। इसलिए, काटते समय
बड़े पेड़ छोड़ो। उनके बीज से ठीक हो सकेंगे
वांछित प्रजातियों की प्रधानता वाला वन। जहां कोई संभावना नहीं है
प्राकृतिक तरीके से जंगल की बहाली सुनिश्चित करने के लिए पौधरोपण करें
मूल्यवान नस्लों।

नदियों के किनारे जंगलों को बहाल करना जरूरी है।

पर्वतीय ढलानों पर वनों में चरने से मृदा अपरदन (क्षरण) होता है। के लिये
स्थिति को ठीक करते हुए जंगलों में चरना बंद कर दिया गया है।

जलाशयों या प्रमुख सड़कों का निर्माण करते समय पेड़ लगाए जाते हैं,
जो अधिक नमी (चिनार, एल्डर, विलो) से कम पीड़ित हैं, ताकि
बाढ़ से होने वाले नुकसान को कम करें।

सब्जियों के कच्चे माल की कटाई और जंगलों में शिकार का आयोजन इस प्रकार किया जाता है कि
आबादी की पुन: उत्पन्न करने की क्षमता को कमजोर करना, अर्थात से अधिक नहीं
फसल का अधिकतम स्वीकार्य हिस्सा।

लॉगिंग के नियमों के अधीन और एंथिल की सुरक्षा सुनिश्चित करना
जंगल में कीटनाशक लगाने की जरूरत नहीं है।

घरेलू कचरे से कूड़े को रोकने के लिए वन पार्क स्थापित करें
इसके संग्रह के लिए कंटेनर, अटे पड़े जंगलों को साफ करें।

असंगठित पर्यटन वन पारिस्थितिकी तंत्र को बहुत नुकसान पहुंचा सकता है।
वनों की संपत्ति को संरक्षित और पुनर्स्थापित करने के लिए उनकी तर्कसंगत अनुमति होगी
उपयोग।

टिकट #26

1. पर्यावरण संरक्षण में अंतर्राष्ट्रीय सहयोग।

पर्यावरण के मुद्दों पर अंतर्राष्ट्रीय सहयोग
यूनेस्को की अध्यक्षता में। 1972 में, उन्होंने अंतर सरकारी विकसित किया
संयुक्त राष्ट्र पर्यावरण कार्यक्रम। यह विकास में सहायता करता है
पर्यावरण शिक्षा। रिकॉर्ड रखता है और प्राकृतिक के संरक्षण का आयोजन करता है
विश्व धरोहर के रूप में वर्गीकृत वस्तुओं।

प्रकृति और प्राकृतिक संसाधनों के संरक्षण के लिए अंतर्राष्ट्रीय संघ (आईयूसीएन)। उसका गोला
गतिविधियाँ - प्राकृतिक पारिस्थितिक तंत्र का संरक्षण, दुर्लभ और का संरक्षण
पौधों और जानवरों की लुप्तप्राय प्रजातियों के साथ-साथ प्राकृतिक स्मारक,
प्रकृति भंडार और राष्ट्रीय उद्यानों का संगठन। पारिस्थितिक
शिक्षा।

विश्व स्वास्थ्य संगठन (डब्ल्यूएचओ)। गतिविधि का क्षेत्र -
पर्यावरण की स्वच्छता और महामारी विज्ञान निगरानी का संगठन
वातावरण। स्वच्छता और स्वच्छ परीक्षा और गुणवत्ता मूल्यांकन
वातावरण।

अंतर्राष्ट्रीय परमाणु ऊर्जा एजेंसी (आईएईए)। गतिविधि का क्षेत्र
- परमाणु ऊर्जा संयंत्रों के निर्माण और संचालन के लिए नियमों का विकास। मानक तय करना
विकिरण सुरक्षा। रेडियोधर्मी सामग्री के संपर्क का आकलन
पर्यावरण पर।

अंतर्राष्ट्रीय समुद्री संगठन (आईएमओ)। अंतर्राष्ट्रीय का विकास
समुद्र को प्रदूषण से बचाने के लिए कन्वेंशन।

संयुक्त राष्ट्र के खाद्य और कृषि संगठन (एफएओ)। वृत्त
गतिविधियाँ - कृषि में पर्यावरणीय समस्याओं का समाधान।
भूमि, जल संसाधन, वन, वन्य जीवन का संरक्षण और उपयोग,
महासागर के जैविक संसाधन।

2. मृदा: पारिस्थितिक तंत्र में पारिस्थितिक कार्य।

पारिस्थितिक तंत्र जीवित जीवों और आवासों से बने होते हैं जो प्रदान करते हैं
उनके संसाधन - ऊर्जा, पानी, पोषक तत्व। लेकिन पारिस्थितिकी तंत्र है
एक कारक जिसे जीवित जीवों या पर्यावरण के लिए जिम्मेदार नहीं ठहराया जा सकता है
एक वास। यह मिट्टी है। पृथ्वी के विभिन्न क्षेत्रों में मिट्टी की मोटाई है
कुछ सेमी से 2 मी.

मिट्टी का मुख्य पदार्थ ह्यूमस है। ह्यूमस की रासायनिक संरचना बहुत होती है
उलझा हुआ। इसमें फिनोल और गहरे रंग के कार्बनिक अम्ल होते हैं और
कार्बनिक के अपघटन (आर्द्रीकरण) की प्रक्रिया के परिणामस्वरूप गठित
पौधों और मिट्टी के जानवरों के जड़ अवशेषों के पदार्थ। धरण
मिट्टी के कुल कार्बनिक पदार्थ का 98% भाग बनाता है (बाकी जीवित है
जड़ें, मिट्टी के जानवर और असिंचित मृत अवशेष
जीव)।

साथ ही कार्बनिक पदार्थों के आर्द्रीकरण की प्रक्रिया के साथ,
ह्यूमस मिनरलाइजेशन। संरचना में शामिल सूक्ष्मजीवों के प्रभाव में
ह्यूमस कॉम्प्लेक्स कार्बनिक यौगिकों को सुलभ रूपों में नष्ट कर दिया जाता है
पौधे। प्राकृतिक मिट्टी में, आर्द्रीकरण की प्रक्रियाओं के बीच और
खनिजकरण, एक संतुलन है, अर्थात। इतना ह्यूमस बनता है
कितना नष्ट हो जाता है।

विभिन्न मिट्टी में अलग-अलग गुण होते हैं: उनमें पोषक तत्वों की एक अलग आपूर्ति होती है
पदार्थ, मिट्टी के घोल की एक अलग प्रतिक्रिया है, विभिन्न भौतिक
गुण (ढीले, घने), विभिन्न नमी शासन।

मिट्टी के प्रभावित होने पर मिट्टी के निर्माण की प्राकृतिक प्रक्रिया बाधित होती है
मानव।

टिकट #27

1. मास्को के उदाहरण पर वातावरण का औद्योगिक प्रदूषण।

औद्योगिक क्रांति की शुरुआत के साथ, वायु प्रदूषण में नाटकीय रूप से वृद्धि हुई।

मास्को कोई अपवाद नहीं है। राजधानी क्षेत्र ने एक पूरा गुच्छा एकत्र किया है
पर्यावरण के लिए खतरनाक उद्योग - परमाणु ऊर्जा से लेकर . तक
रसायन उद्योग। कुल मिलाकर, से अधिक हैं
2 हजार उद्यम, 4 नागरिक उड्डयन हवाई अड्डे, 14 रेलवे
राजमार्ग, कई राजमार्ग। केवल घरेलू नालियां
जलाशयों को भारी नुकसान पहुंचाते हैं। राजधानी एक जल प्रदूषण चैंपियन है।
अपशिष्ट जल की मात्रा (2367 मिलियन क्यूबिक मीटर प्रति वर्ष) के मामले में, यह . में पहले स्थान पर है
रूस। मास्को में सबसे तीव्र पर्यावरणीय समस्या रासायनिक है
पर्यावरण प्रदूषण। वातावरण में हानिकारक उत्सर्जन में से अधिक शामिल हैं
100 प्रकार के प्रदूषक। मास्को को में शामिल किया गया है
वायु प्रदूषण के उच्चतम स्तर वाले शहरों की सूची। बुनियादी
इसका अपराधी मोटर परिवहन है (शहर में सभी उत्सर्जन का 90% तक)। पकड़ा गया
भारी धातुएं 80-100 किमी के दायरे में हवा में जमा हो जाती हैं। मिट्टी विशेष रूप से
कैडमियम से अत्यधिक प्रदूषित, उदाहरण के लिए, संग्रहालय-संपदा "गोर्की" के क्षेत्र में
लेनिन्स्की के अनुसार, इसकी सांद्रता प्राकृतिक मूल्य से 70-100 गुना अधिक है।

उद्यमों द्वारा रेडियोधर्मी कचरे के प्रबंधन के नियमों का उल्लंघन
(उदाहरण के लिए, उन्हें शहर के डंपों में हटाना) इस तथ्य की ओर जाता है कि हर साल
राजधानी में रेडियोधर्मी संदूषण के 60-70 नए स्रोत बन रहे हैं।

2. संरक्षित क्षेत्र और पर्यावरण की गुणवत्ता बनाए रखने में उनकी भूमिका
वातावरण।

आबादी की रक्षा करने का सबसे विश्वसनीय तरीका है कि उन्हें समग्र रूप से संरक्षित किया जाए
पारिस्थितिक तंत्र जिसमें पारिस्थितिक संतुलन बनाए रखा जाता है। इसके लिए
विभिन्न प्रकार के विशेष रूप से संरक्षित क्षेत्र (पीए) बनाएं।

भंडार। यह पीए का मुख्य प्रकार है, जो सबसे विश्वसनीय सुरक्षा प्रदान करता है।
प्रकार। आज दुनिया में दो हजार से अधिक भंडार हैं, रूस में - 70।
सबसे महत्वपूर्ण भंडार बायोस्फेरिक हैं। बायोस्फीयर रिजर्व
दुनिया भर में समान रूप से वितरित, और प्रत्येक कुछ का प्रतिनिधित्व करता है
प्राकृतिक नज़ारा। वे वहां बनाए जाते हैं जहां प्रकृति ने अपना अस्तित्व नहीं खोया है
मूल लक्षण। मानकों के रूप में बायोस्फीयर रिजर्व में अवलोकन
प्रकृति में तैयार किए गए एकल अंतरराष्ट्रीय कार्यक्रम के अनुसार किया जाता है
यूनेस्को। यह वैज्ञानिकों द्वारा प्राप्त परिणामों की तुलना को सुनिश्चित करता है
विभिन्न देश। हमारे देश में दुनिया में लगभग 300 बायोस्फीयर रिजर्व हैं
उनमें से 11 (कोकेशियान, सिखोट-एलिन, आदि) हैं।

मनुष्य द्वारा अशांत क्षेत्रों में भी भंडार बनाए जाते हैं। हालांकि, नहीं
पारिस्थितिक तंत्र हमेशा अपनी मूल स्थिति में लौट आते हैं, क्योंकि आबादी
पौधों और जानवरों की कुछ प्रजातियों को बहाल नहीं किया जाता है। रिजर्व में
उपयोग या तो पूरी तरह से बंद कर दिया गया है या किया गया है, लेकिन इस उद्देश्य के लिए नहीं
संरक्षण के लिए इन जमीनों से मुनाफा भी वसूलते हैं।

राष्ट्रीय उद्यान। राष्ट्रीय उद्यानों में मुख्य बात यह है कि के लिए स्थितियां बनाना
अच्छी तरह से संरक्षित प्रकृति की गोद में लोगों का संगठित मनोरंजन। पर
पर्यटकों का उचित संगठन, प्रकृति के साथ संचार से लाभान्वित होना और
स्वास्थ्य को मजबूत करना, पारिस्थितिकी तंत्र को नुकसान न पहुंचाएं।

भंडार। वे वसूली के लिए एक निश्चित अवधि के लिए आयोजित किए जाते हैं।
खेल जानवरों के पशुधन और औषधीय पौधों की आबादी।
पुनर्स्थापित प्रजातियों का उपयोग करने की अनुमति नहीं है। रिजर्व में
खेल जानवर सामान्य के लिए स्थितियां बनाते हैं
मूस जैसे बड़े जानवरों की पुनरुत्पादन आबादी, या
सावधान पक्षी जैसे कि ब्लैक ग्राउज़ या सपेराकैली।

एक विशेष प्रकार के भंडार - वन उद्यान। वे बाढ़ के मैदानों के जंगलों में बनाए गए हैं: काट लें
पेड़ और झाड़ियाँ जिनका कोई संसाधन मूल्य नहीं है, और उनके स्थान पर स्वयं
मूल्यवान प्रजातियां बढ़ती हैं (नागफनी, वाइबर्नम, पक्षी चेरी या कुत्ता गुलाब)।

टिकट #28

1. एग्रोसेपोसिस और एग्रोइकोसिस्टम।

कृषि-पारिस्थितिकी तंत्र या कृषि भूमि मानवजनित हैं (अर्थात।
मानव निर्मित) पारिस्थितिक तंत्र।

कृषि पारिस्थितिकी तंत्र में तीन मुख्य ट्राफिक समूहों के जीव होते हैं:
उत्पादक, उपभोक्ता और अपघटक।

कृषि पारिस्थितिकी तंत्र में उत्पादकों की खेती की जाती है पौधे, घास का मैदान घास और
चरागाह, बगीचों के पेड़, वन वृक्षारोपण और प्राकृतिक वन इसके में शामिल हैं
मिश्रण। खेती वाले पौधों के उत्पादक और साथी मातम हैं।

कृषि पारिस्थितिकी तंत्र में अपघटक मुख्यतः जीवाणु होते हैं। वे समर्थन करते हैं
मिट्टी की उर्वरता, पौधों के अवशेषों को ह्यूमस और ह्यूमस में बदलना और लागू करना
खाद के खेतों पर - सरल कार्बनिक और खनिज पदार्थों में,
पौधों के लिए उपलब्ध है। डीकंपोजर में न केवल कम करने वाले एजेंट हैं
मिट्टी की उर्वरता, बल्कि इसके विध्वंसक भी।

मनुष्य कृषि पारिस्थितिकी तंत्र की सभी जीवित आबादी, प्रजातियों का हिस्सा नियंत्रित नहीं करता है
उसमें प्रवेश करता है और उसकी इच्छा के विरुद्ध रहता है। ऐसी जीवित प्रजातियां
स्वतःस्फूर्त कहा जाता है। इनमें कीट, खरपतवार के पौधे, और
उपयोगी जानवर: पक्षी, शिकारी कीड़े, आदि।

कृषि पारिस्थितिकी तंत्र में मनुष्य स्वयं एक फाइटोफेज उपभोक्ता है (पौधों पर फ़ीड करता है) और
ज़ोफैगस (मांस खाता है और दूध पीता है)। हालांकि, कृषि पारिस्थितिकी तंत्र में इसकी भूमिका अभी भी है
अधिक, क्योंकि वह अपनी रुचियों के आधार पर संरचना बनाता है और
कृषि पारिस्थितिकी तंत्र की संरचना और इसके पोषी तत्वों को प्रभावित करने के क्रम में
सबसे बड़ा प्राथमिक और द्वितीयक उत्पादन प्राप्त करें: वह हल करता है
भूमि का हिस्सा और कृषि फसलों को बोता है, घास के मैदान बनाता है
जंगलों के स्थान पर चरागाह, खेत जानवरों को पालता है।

2. पारिस्थितिक निगरानी की समस्याएं।

पारिस्थितिक तंत्र में होने वाली प्रक्रियाओं की निरंतर निगरानी
पर्यावरण निगरानी कहा जाता है। जमीन ही नहीं है
निगरानी, ​​विशेष उपकरणों का उपयोग करते समय वे निगरानी करते हैं
पानी, हवा, मिट्टी या हानिकारक भोजन में एकाग्रता
पदार्थ, लेकिन एयरोस्पेस भी। इस मामले में, में परिवर्तन
पारिस्थितिक तंत्र, उपग्रहों और विमानों पर उपकरणों का उपयोग करके मूल्यांकन किया गया,
जो जंगलों या फसलों की स्थिति, मिट्टी के कटाव की मात्रा को ध्यान में रखते हैं,
वातावरण में प्रदूषकों की सामग्री।

निगरानी के आंकड़ों के आधार पर आगे की भविष्यवाणी
देखे गए लक्षणों में परिवर्तन और सुधार के लिए निर्णय लेना
पर्यावरण की स्थिति - नई उपचार सुविधाओं का निर्माण
उद्यम जो वातावरण और पानी को प्रदूषित करते हैं, कटिंग सिस्टम बदलते हैं
वन और पौधे नए, मृदा-सुरक्षात्मक फसल चक्र शुरू किए जा रहे हैं, आदि।

निगरानी अक्सर क्षेत्रीय और गणतांत्रिक समितियों द्वारा की जाती है:
जल मौसम विज्ञान सेवा।

एक विशेष प्रकार की निगरानी जैविक (बायोमोनिटरिंग) है। पर
बायोमॉनिटरिंग पारिस्थितिक तंत्र के वनस्पतियों और जीवों की स्थिति की निगरानी करता है,
संरक्षण की आवश्यकता वाली प्रजातियों की पहचान की जाती है, जिन्हें "लाल" में शामिल किया गया है
पुस्तकें।" प्रदूषण के स्तर का आकलन करने के लिए बायोमोनिटरिंग का भी उपयोग किया जाता है
उनके जीवों की स्थिति के अनुसार पर्यावरण।

टिकट #29

1. ऊर्जा की बचत और संसाधन की बचत।

कम से कम ऊर्जा-गहन प्रौद्योगिकी का चयन करके ऊर्जा की बचत की जानी चाहिए
उत्पादन योजनाएं।

कन्वर्टर्स में स्टील गलाने के आधुनिक तरीकों के साथ, 2
पारंपरिक खुली चूल्हा भट्टियों की तुलना में कई गुना कम ऊर्जा।

यदि धातु को विभिन्न विद्युत चाप भट्टियों में साइट पर संसाधित किया जाता है
धातुकर्म संयंत्रों में इसके परिवहन के बिना क्षमता।

कृषि में, आप उच्च पैदावार बढ़ा सकते हैं और मांस प्राप्त कर सकते हैं और
कम ईंधन वाला दूध, उर्वरक, कीटनाशक आदि।

इंजन में अधिक उन्नत ईंधन दहन प्रणाली वाली कार और
जब ईंधन में विशेष पदार्थ मिलाते हैं जो इसके दहन में सुधार करते हैं,
दोगुना खर्च कर सकते हैं।

रोजमर्रा की जिंदगी में, एक 18 वाट का फ्लोरोसेंट लैंप उतनी ही रोशनी देता है जितना
75 वाट का गरमागरम बल्ब कितना है।

संसाधन की बचत।

संसाधनों की बचत। मैकेनिकल इंजीनियरिंग में, धातु की एक बड़ी मात्रा में चला जाता है
चिप्स, उच्च धातु की खपत, जिसे कम किया जाना चाहिए। उत्पादन में
तेल उत्पादों, तेल शोधन की गहराई बढ़ाने के लिए आवश्यक है (की उपज)
उत्पाद)। पानी बचाने की जरूरत है। इसे साफ और इस्तेमाल किया जा सकता है
बार-बार।

संसाधनों का एकीकृत उपयोग। महत्वपूर्ण संसाधन बचत हो सकती है
प्राप्त करें यदि समान संसाधन अलग-अलग साझा किए जाते हैं
अर्थव्यवस्था के क्षेत्र (उदाहरण के लिए: राख और लावा से - टीपीपी कचरा - आप कर सकते हैं
निर्माण सामग्री प्राप्त करें और दोहरी जीत प्राप्त करें)।

माध्यमिक कच्चे माल का प्रसंस्करण। स्क्रैप धातु, बेकार कागज, कांच,
प्लास्टिक, कपड़े - सब कुछ पुनर्नवीनीकरण किया जाना चाहिए। यह अनुमति देता है
ऊर्जा संरक्षण हेतु।

2. सुरक्षा प्रणाली ओओटी।

पीए (विशेष रूप से संरक्षित क्षेत्र) प्रकृति के भंडार, अभयारण्य हैं,
राष्ट्रीय उद्यान, प्राकृतिक स्मारक।

रिजर्व मुख्य प्रकार के पीए हैं (रूस में 70 रिजर्व हैं)।

बरगुज़िंस्की नेचर रिजर्व उत्तर-पूर्वी तट पर स्थित है
1916 में स्थापित बैकाल। संरक्षित: बरगुज़िन सेबल, मारन,
बैकाल सील।

तैमिर रिजर्व - क्रास्नोयार्स्क क्षेत्र, 1979 में स्थापित।
द्वारा संरक्षित: लाल गले वाले औक, हंस, पेरेग्रीन बाज़।

अस्त्रखान नेचर रिजर्व - 1919 में स्थापित। संरक्षित: कमल, बगुले,
जलकाग, जलपक्षी।

बायोस्फीयर रिजर्व बनाए जाते हैं जहां प्रकृति ने अपना खोया नहीं है
प्राचीन विशेषताएं (रूस में - 11)।

Prioksko-Terrasny - नदी घाटी में मास्को क्षेत्र (सेरपुखोव) के दक्षिण में
ठीक है। 1945 में स्थापित (1978 से बायोस्फेरिक)। पाइन, सन्टी
जंगल, ऊदबिलाव यहाँ केंद्रीय बाइसन नर्सरी है।

सिखोट-एलिन रिजर्व - प्रिमोर्स्की टेरिटरी (1935), बायोस्फेरिक विथ
1978. देवदार-व्यापक वन, टैगा, बाघ और हिमालयन
सहना।

भंडार के कार्य:

वनस्पतियों, जीवों और पारिस्थितिक तंत्र की सुरक्षा सुनिश्चित करनी चाहिए।

रिजर्व वैज्ञानिक संस्थान हैं जहां विभिन्न के जीवविज्ञानी और पारिस्थितिकीविद हैं
प्रोफाइल, पारिस्थितिकी तंत्र और उनके घटकों की स्थिति का विस्तार से अध्ययन
आबादी। ये जीवित प्रयोगशालाएं हैं जो मूल्यवान सामग्री प्रदान करती हैं जो मदद करती हैं
आबादी और पारिस्थितिक तंत्र की स्थिरता को बनाए रखना
संरक्षित।

भंडार दुर्लभ और . की आबादी के घनत्व की बहाली के लिए केंद्र के रूप में कार्य करते हैं
लुप्तप्राय जानवरों और पौधों की प्रजातियां। उनमें से कुछ निकाले जा सकते हैं
पुनर्वास

राष्ट्रीय उद्यान। कार्य: संगठित मनोरंजन के लिए परिस्थितियों का निर्माण
अच्छी तरह से संरक्षित प्रकृति की गोद में लोग। रूस में कुल 22
राष्ट्रीय उद्यान।

प्रकृति के स्मारक (9 हजार)।

1. निर्जीव प्रकृति के स्मारक (चट्टानें, गुफाएं)।

2. वन्यजीवों के स्मारक (पौधों और जानवरों को राहत देते हैं,
दिए गए क्षेत्र में सामान्य)।

3. संयुक्त स्मारक (तालाब, झील)।

भंडार। पशुधन को बहाल करने के लिए एक निश्चित अवधि के लिए व्यवस्थित करें
खेल जानवर या औषधीय जड़ी बूटियों की आबादी। ठीक करके नए जैसा बनाया गया
प्रकार की अनुमति नहीं है।

एक विशेष प्रकार का रिजर्व - वन उद्यान। वे बाढ़ के जंगलों में बनाए जाते हैं:
उन पेड़ों और झाड़ियों को काटें जिनका कोई संसाधन मूल्य नहीं है, और उन पर
यहां मूल्यवान प्रजातियां उगती हैं।

मूस की सुरक्षा के लिए, रिजर्व के लिए 18 हजार हेक्टेयर का एक क्षेत्र आवंटित किया गया है, और इसके लिए
खेल पक्षियों का संरक्षण मात्र 2 हजार हेक्टेयर है। क्यों? मूस - बड़ा
जानवरों। उन्हें पुन: पेश करने के लिए, उन्हें अधिक संसाधनों की आवश्यकता होती है।
पक्षियों से भी अमीर।

I पारिस्थितिकी तंत्र को आपूर्ति की जाने वाली ऊर्जा की कुल मात्रा है।

ऊर्जा का कुछ भाग परावर्तित होता है, और कुछ भाग ऊष्मा के रूप में नष्ट हो जाता है।

आर - सांस लेने की लागत।

Nu - असम्बद्ध ऊर्जा।

ना - ऊर्जा भंडारण।

पंजाब - सकल उत्पादन।

पीआर - शुद्ध उत्पादन।

जैविक उत्पादन जैविक पदार्थ की मात्रा है,
जो प्रति इकाई समय प्रति इकाई क्षेत्र (जीआर/एम?, किग्रा/एम?)

जैविक उत्पाद

प्राथमिक (सकल) माध्यमिक (शुद्ध)

सकल उत्पादन - यह वह उत्पाद है जो पौधे बनाते हैं
प्रकाश संश्लेषण की प्रक्रिया।

शुद्ध उत्पादन ऊर्जा का वह भाग है जो खर्च करने के बाद भी रहता है
सांस लेना।

पृथ्वी के पारितंत्रों की औसत उत्पादकता 0.3 किग्रा/मीटर से अधिक नहीं होती है। पर
एक स्तर से दूसरे स्तर पर ऊर्जा का संक्रमण, लगभग 90% खो जाता है
ऊर्जा, इसलिए माध्यमिक उत्पादन प्राथमिक से 20-50 गुना कम है।

एक टाइपो मिला? CTRL+Enter चुनें और दबाएँ

10 मई 2004

जनसंख्या विषम हैं।

असमान व्यक्तियों वाली जनसंख्या अधिक स्थिर होती है। इस तरह की आबादी में पाले की स्थिति में फसलों को खोने का जोखिम कम होता है (उदाहरण के लिए, फूलों का केवल एक हिस्सा जम जाता है, बाकी को कलियों के रूप में संरक्षित किया जा सकता है)।

मुकाबला।

जनसंख्या घनत्व स्थिर नहीं है और साल-दर-साल इसमें उतार-चढ़ाव होता है क्योंकि व्यक्तिगत वर्षों में सबसे दुर्लभ संसाधन की मात्रा स्थिर नहीं होती है।

शिकार जनसंख्या घनत्व का प्रकोप शिकारी जनसंख्या घनत्व के प्रकोप का कारण बनता है क्योंकि शिकार शिकारियों के पास खाने के लिए पर्याप्त है। अतः भोजन के अभाव में इनकी मृत्यु नहीं होती है। और अधिकांश आबादी जीवित रहती है। आखिरकार, दुर्लभ संसाधन का स्तर जितना अधिक होगा, जनसंख्या घनत्व उतना ही अधिक होगा, अर्थात। जितने अधिक शिकार, उतने अधिक शिकारी।

गिलहरी की आबादी का घनत्व भोजन की मात्रा पर निर्भर करता है, अर्थात। यदि जंगल में बहुत सारे शंकु, मशरूम और बलूत का फल आदि उगते हैं, तो गिलहरी की आबादी में वृद्धि होगी। गिलहरी अपनी संतानों को पुन: उत्पन्न करने और खिलाने में सक्षम होगी।

ऐसे कारण जो जनसंख्या की स्थिरता का उल्लंघन करते हैं।

अत्यधिक खनन। आवास विनाश (पर्यटक आग लगाते हैं, नदियों को प्रदूषित करते हैं)।

एक नई प्रजाति का परिचय। नई प्रजातियां अपनी आबादी को नष्ट करते हुए देशी प्रजातियों को बाहर कर सकती हैं।

पर्यावरण प्रदूषण। जलीय पारिस्थितिक तंत्र के निवासी इससे सबसे अधिक पीड़ित हैं।

8. आवास के रूप में मिट्टी की पारिस्थितिक विशेषताएं।

मिट्टी का वातावरण: मध्यवर्ती एम / वाई हवा और पानी। मोटाई कई मीटर है। मिट्टी में, वे स्रावित करते हैं 3 चरण: कठिन, तरलतथा गैसीय. जीवित जीवों के साथ मिट्टी सबसे अधिक संतृप्त है। और कभी कभी आवंटित चौथा चरण - लाइव. मिट्टी में गर्मी की कमी, नमी की कमी या अधिकता होती है।

मृदा आवास विशेषताएं:

पौधों के मरने वाले भागों (जड़ों, गिरती पत्तियों), मृत जानवरों और उनके चयापचय उत्पादों के कारण कार्बनिक पदार्थ नियमित रूप से मिट्टी में प्रवेश करते हैं; मिट्टी छिद्रों, छोटी और बड़ी गुहाओं से भर जाती है; मिट्टी के प्रकार के आधार पर, ऐसी रिक्तियों की मात्रा 20 से 70% तक भिन्न हो सकती है;

मिट्टी के छिद्रों में हवा ऑक्सीजन में खराब है, कार्बन डाइऑक्साइड से संतृप्त है और जल वाष्प में समृद्ध है;

सतह पर और मिट्टी के ऊपरी हिस्से में तापमान में तेज उतार-चढ़ाव देखा जाता है, जो गहराई से चिकना हो जाता है।

5. विशेष रूप से संरक्षित प्राकृतिक क्षेत्रों के प्रकार और कार्य। लाल किताबें।

संरक्षित क्षेत्रों (भंडार) की आवश्यकता है ताकि पौधों और जानवरों की दुर्लभ प्रजातियां उनके जीवन को खतरे में डाले बिना कार्य कर सकें। अपने अस्तित्व की शुरुआत से ही, मानव जाति प्रत्यक्ष या अप्रत्यक्ष रूप से अपने पर्यावरण को प्रभावित करती है, आसपास के जानवरों और पौधों की प्रजातियों को खतरे में डालती है। प्रकृति को मनुष्य और जीवन पर उसके अतिक्रमण से बचाने के लिए भंडार की आवश्यकता होती है।

आबादी की रक्षा करने का सबसे विश्वसनीय तरीका पूरे पारिस्थितिक तंत्र के हिस्से के रूप में उनकी रक्षा करना है जिसमें पारिस्थितिक संतुलन बनाए रखा जाता है। इसके लिए विभिन्न प्रकार के विशेष रूप से संरक्षित क्षेत्र (पीए) बनाए जाते हैं।

भंडार। यह पीए का मुख्य प्रकार है, जो सबसे मज़बूती से प्रजातियों की सुरक्षा सुनिश्चित करता है। आज दुनिया में दो हजार से अधिक भंडार हैं, रूस में - 70। सबसे महत्वपूर्ण भंडार बायोस्फेरिक हैं। बायोस्फीयर रिजर्व दुनिया भर में समान रूप से वितरित किए जाते हैं, और प्रत्येक किसी न किसी प्रकार के प्राकृतिक परिदृश्य का प्रतिनिधित्व करता है। वे वहां बनाए जाते हैं जहां प्रकृति ने अपनी मूल विशेषताओं को नहीं खोया है। प्रकृति के मानकों के रूप में बायोस्फीयर रिजर्व में अवलोकन यूनेस्को द्वारा तैयार किए गए एकल अंतरराष्ट्रीय कार्यक्रम के अनुसार किए जाते हैं। यह विभिन्न देशों में वैज्ञानिकों द्वारा प्राप्त परिणामों की तुलना सुनिश्चित करता है। दुनिया में लगभग 300 बायोस्फीयर रिजर्व हैं, हमारे देश में उनमें से 11 (कोकेशियान, सिखोट-एलिन, आदि) हैं।

मनुष्य द्वारा अशांत क्षेत्रों में भी भंडार बनाए जाते हैं। हालांकि, पारिस्थितिक तंत्र हमेशा अपनी मूल स्थिति में नहीं लौटते हैं, क्योंकि कुछ पौधों और जानवरों की प्रजातियों की आबादी बहाल नहीं होती है। भंडार में, उपयोग या तो पूरी तरह से बंद कर दिया जाता है, या किया जाता है, लेकिन इन भूमि से लाभ निकालने के उद्देश्य से नहीं, बल्कि सुरक्षा के लिए।

राष्ट्रीय उद्यान। राष्ट्रीय उद्यानों में मुख्य बात अच्छी तरह से संरक्षित प्रकृति की गोद में लोगों के संगठित मनोरंजन के लिए परिस्थितियों का निर्माण है। उचित संगठन से पर्यटक प्रकृति के साथ संचार और स्वास्थ्य को मजबूत करते हुए पारिस्थितिकी तंत्र को नुकसान नहीं पहुंचाते हैं।

भंडार। वे एक निश्चित अवधि के लिए खेल जानवरों के पशुधन और औषधीय पौधों की आबादी को बहाल करने के लिए आयोजित किए जाते हैं। पुनर्स्थापित प्रजातियों का उपयोग करने की अनुमति नहीं है। वन्यजीव अभयारण्यों में, बड़े जानवरों की आबादी के सामान्य प्रजनन के लिए स्थितियां बनाई जाती हैं, जैसे कि मूस, या सतर्क पक्षी, जैसे कि ब्लैक ग्राउज़ या कैपरकैली।

एक विशेष प्रकार के भंडार - वन उद्यान। वे बाढ़ के जंगलों में बनाए जाते हैं: जिन पेड़ों और झाड़ियों का कोई संसाधन मूल्य नहीं होता है, उन्हें काट दिया जाता है, और उनके स्थान पर मूल्यवान प्रजातियां बढ़ती हैं (नागफनी, वाइबर्नम, पक्षी चेरी या जंगली गुलाब)।

इस स्तर पर, संरक्षण की वस्तुएं आबादी में रहने वाले विशिष्ट पौधे या पशु प्रजातियां हैं।

सुरक्षा को व्यवस्थित करने के लिए, वस्तुओं की पहचान की जाती है और लाल पुस्तकें बनाई जाती हैं। पहली रेड बुक 1966 (5 खंड) में छपी।

आरएसएफएसआर (पौधों) की लाल किताब 1988 (533 प्रजातियों) में प्रकाशित हुई थी।

आरएसएफएसआर (जानवरों) की लाल किताब 1985 (247 प्रजातियां) में प्रकाशित हुई थी।

जनसंख्या-प्रजाति के स्तर पर संरक्षण किसके द्वारा किया जाता है:

फूल वाले पौधों (वेनेरेन, स्लिपर) के संग्रह पर प्रतिबंध।

दुर्लभ औषधीय जड़ी बूटियों के संग्रह पर प्रतिबंध (वेलेरियन ऑफिसिनैलिस)।

पक्षियों और जानवरों की दुर्लभ प्रजातियों (क्रेन, हंस, बाघ) के शिकार पर प्रतिबंध।

कुछ प्रकार की मछलियों (स्टर्जन), तितलियों और भृंगों को पकड़ने पर प्रतिबंध।

अत्यधिक शिकार का निषेध (आर्कटिक लोमड़ी, सेबल)।

आवास विनाश निषिद्ध है।

नई प्रजातियों (सबसे खूबसूरत पंख वाली घास) की शुरूआत पर प्रतिबंध।

प्रदूषण निषेध

समस्या को हल करने के तरीके:

मानव नियंत्रण में प्रजनन प्रजातियां।

जानवरों को चिड़ियाघरों में, पौधों को वनस्पति उद्यानों में पाला जाता है। दुर्लभ प्रजातियां प्रजनन कर रही हैं।

2. जीन बैंकों का निर्माण।

बैंक पौधों के बीज और जमे हुए ऊतक संस्कृतियों या रोगाणु कोशिकाओं (अधिक बार वे जमे हुए शुक्राणु को स्टोर करते हैं) दोनों को स्टोर कर सकते हैं, जिससे जानवरों और पौधों को प्राप्त किया जा सकता है।

9. आवास के रूप में जीवों की पारिस्थितिक विशेषताएं।

जैविक पर्यावरण की अपनी विशेषताएं हैं:

प्रकाश और वायुमंडलीय हवा की कमी;

लगभग स्थिर तापमान;

उच्च आर्द्रता;

पोषक तत्वों की प्रचुरता;

मेजबान प्रतिक्रिया।

6. भूमि-वायु पर्यावरण के लिए पौधों और जानवरों का अनुकूलन।

प्राकृतिक वास।

पर्यावरण को प्राकृतिक निकायों और परिघटनाओं के रूप में समझा जाता है जिनके साथ जीव प्रत्यक्ष या अप्रत्यक्ष संबंध में हैं। पर्यावरण के व्यक्तिगत तत्व जिनसे जीव अनुकूली प्रतिक्रियाओं के साथ प्रतिक्रिया करते हैं, कारक कहलाते हैं। पर्यावरणीय कारक पर्यावरण का कोई भी तत्व है जिसके लिए जीव अनुकूली प्रतिक्रियाओं या अनुकूलन के साथ प्रतिक्रिया करते हैं। सभी कारकों को 3 श्रेणियों में विभाजित किया गया है: 1) अजैविक - निर्जीव प्रकृति के कारक (वायुमंडलीय, जलवायु, मिट्टी) 2) जैविक - वन्य जीवन के कारक (अपशिष्ट उत्पाद); 3) मानवजनित - मानव गतिविधि के कारक (प्रदूषण, मछली पकड़ना, बांधों का निर्माण)। क्रिया की आवृत्ति और दिशा के अनुसार कारकों का वर्गीकरण।

आवंटित करें: सख्ती से समय-समय पर संचालन (दिन के समय में परिवर्तन, वर्ष के मौसम, उतार और प्रवाह); सख्त आवधिकता के बिना संचालन, लेकिन समय-समय पर आवर्ती। (मौसम की घटनाएं, भूकंप)। अगला समूह दिशात्मक कार्रवाई के कारक हैं, वे आमतौर पर एक दिशा में बदलते हैं (जलवायु का गर्म होना या ठंडा होना, जलाशयों का अतिवृद्धि, प्रदेशों का दलदल)। और अंतिम समूह अनिश्चित क्रिया (मानवजनित कारक) के कारक हैं।

जमीन - हवा: यह वातावरण सबसे विविध है। यह कम वायु घनत्व, बड़े तापमान में उतार-चढ़ाव और उच्च वायुमंडलीय गतिशीलता की विशेषता है। सीमित कारक गर्मी और नमी की कमी या अधिकता। तापमान में बड़ा उतार-चढ़ाव, ऑक्सीजन की अच्छी आपूर्ति याव। दिखने का एक अच्छा मकसद। निरंतर शरीर के तापमान वाले जीव। इस वातावरण में जीवों के लिए, 3 अनुकूलन तंत्र विशिष्ट हैं: शारीरिक, रासायनिक, व्यवहार. भौतिक- त्वचा, शरीर में वसा, पानी का वाष्पीकरण (जानवरों में पसीना, पौधों में वाष्पोत्सर्जन)। रासायनिक- गहन विनिमय इन-इन। व्यवहार- पसंदीदा पदों के जीवों द्वारा पसंद: सूरज या अंधेरे स्थानों के लिए खुला, विभिन्न प्रकार के आश्रय। बर्गमैन का नियम- जानवर जितना बड़ा होता है, इसका मतलब है कि वह उत्तर में रहता है, छोटे वाले उष्ण कटिबंध में।

7. जलीय पर्यावरण की पारिस्थितिक विशेषताएं।

जल पर्यावरण: सबसे सजातीय। यह अंतरिक्ष में थोड़ा बदलता है, व्यक्तिगत पारिस्थितिक तंत्र द्वारा कोई स्पष्ट सीमाएँ m / y नहीं होती हैं। सीमित कारक ऑक्सीजन. तापमान में वृद्धि, कार्बनिक पदार्थों के साथ संवर्धन और कमजोर मिश्रण के साथ, पानी में ऑक्सीजन की मात्रा कम हो जाती है। दूसरा सीमित कारक है रोशनी. गहराई के साथ रोशनी तेजी से घटती है। भारी प्रदूषित स्थानों में प्रकाश 50-60 मीटर की गहराई तक प्रवेश कर सकता है - केवल कुछ सेंटीमीटर। पानी में कुछ गर्म रक्त वाले जीव होते हैं। यह 2 कारणों6 छोटे तापमान में उतार-चढ़ाव और ऑक्सीजन की कमी का परिणाम है। जलीय पर्यावरण के निवासियों के शरीर का तापमान परिवर्तनशील होता है। पानी के कई निवासी शरीर के सभी अंगों के माध्यम से ऑक्सीजन का उपभोग करते हैं। अक्सर, श्वास को एक निस्पंदन प्रकार के पोषण के साथ जोड़ा जाता है, जिसमें बड़ी मात्रा में पानी शरीर से होकर गुजरता है। ऑक्सीजन की कमी की अवधि के दौरान कुछ जीव निलंबित एनीमेशन (पदार्थों के आदान-प्रदान की लगभग पूर्ण समाप्ति) तक, अपनी महत्वपूर्ण गतिविधि को धीमा करने में सक्षम हैं। प्रकाश की कमी या उसकी अनुपस्थिति की स्थितियों में, जीव अभिविन्यास के लिए ध्वनि का उपयोग करते हैं। विभिन्न बाधाओं का पता लगाने के लिए, परावर्तित ध्वनि का उपयोग इकोलोकेशन के प्रकार द्वारा किया जाता है। पानी की गहराई पर, कई जीवों का अपना स्वयं का प्रकाश होता है।

14. आबादी की गतिशीलता।

जनसंख्या की गतिशीलता समय के साथ जनसंख्या में परिवर्तन है। यदि जनसंख्या लंबे समय तक नहीं बदलती है, तो कहा जाता है कि यह एक राज्य में है समस्थिति. नामों की संख्या में तीव्र परिवर्तन की अवधि। जनसंख्या तरंगें, जीवन तरंगें कभी-कभी वे खाद्य कारकों से जुड़ी होती हैं, कभी मौसम के साथ, कभी सौर गतिविधि से। बहुतायत में अचानक परिवर्तन आमतौर पर जनसंख्या के जीवन के लिए नकारात्मक परिणाम होते हैं: उच्च बहुतायत में, भोजन की कमी के परिणामस्वरूप सभी व्यक्तियों के कमजोर होने के कारण, बड़े पैमाने पर रोग संभव हैं; कम बहुतायत में, सीमा से अधिक होने के कारण इसके न्यूनतम मूल्य। पशु साम्राज्य में, एक तीव्र अंतःविषय संघर्ष का परिणाम प्रकट होता है। आबादी में व्यक्तियों की अधिक भीड़ के साथ नरभक्षण (अपनी तरह का खाना) के रूप में, संख्याओं का विनियमन तंत्र याव हो सकता है। तनाव की घटनाएँ। वे स्तनधारियों की सबसे विशेषता हैं। तनाव में, कुछ व्यक्ति प्रजनन करने की अपनी क्षमता को कम या खो देते हैं। मजबूत व्यक्ति तनाव के प्रति कम संवेदनशील होते हैं। होमोस्टैसिस के कारक के रूप में प्रवासन प्रकट हुआ। 2 प्रकार में। उनमें से पहला भीड़भाड़ के दौरान आबादी से व्यक्तियों के बड़े पैमाने पर पलायन को संदर्भित करता है। यह प्रोटीन के लिए विशिष्ट है। आबादी छोड़ने वाले व्यक्ति आमतौर पर वापस नहीं आते हैं। और कुछ चलते-चलते मर जाते हैं। दूसरे प्रकार का प्रवासन व्यक्तियों के कुछ हिस्सों के कम जनसंख्या घनत्व के साथ अन्य आबादी के लिए अधिक क्रमिक (शांत) प्रस्थान से जुड़ा है।

11. जनसंख्या का लिंग और आयु संरचना।

जनसंख्या की आयु संरचना।इस प्रकार की संरचना जनसंख्या में विभिन्न आयु के व्यक्तियों के अनुपात से जुड़ी होती है। एक ही उम्र के व्यक्तियों को आमतौर पर सहवास, यानी आयु समूहों में जोड़ा जाता है।

पौधों की आबादी की आयु संरचना का विस्तार से वर्णन किया गया है। यह निम्नलिखित आयु (जीवों के आयु समूहों) को अलग करता है (टी.ए. रोबॉटनोव के अनुसार):

अव्यक्त अवधि - बीज की स्थिति;

प्रीजेनरेटिव अवधि (अंकुर, किशोर पौधे, अपरिपक्व पौधे और कुंवारी पौधे के राज्य शामिल हैं);

जनन अवधि (आमतौर पर तीन उप-अवधि में विभाजित - युवा, परिपक्व और पुराने जनन व्यक्ति);

पोस्ट-जेनरेटिव अवधि (एक उप-पौधे की अवस्था, एक बूढ़ा पौधा और मरने का चरण शामिल है)।

बेशक, यह कैलेंडर और जैविक युग के बीच संबंधों की समस्या को उठाता है। एक निश्चित आयु राज्य से संबंधित कुछ रूपात्मक (उदाहरण के लिए, एक जटिल पत्ती के विच्छेदन की डिग्री) और शारीरिक (उदाहरण के लिए, संतान देने की क्षमता) संकेतों की गंभीरता से निर्धारित होता है। इस प्रकार सबसे पहले व्यक्ति की जैविक आयु निश्चित होती है। पारिस्थितिक विज्ञानी के लिए जैविक युग का अधिक महत्व है, क्योंकि यह जनसंख्या प्रक्रियाओं में व्यक्ति की भूमिका निर्धारित करता है। इसी समय, एक नियम के रूप में, जैविक और कैलेंडर युग के बीच एक संबंध है।

पशु आबादी में, विभिन्न आयु चरणों को भी प्रतिष्ठित किया जा सकता है। उदाहरण के लिए, पूरी तरह से कायापलट के साथ विकसित होने वाले कीड़े अंडे, लार्वा, प्यूपा, इमागो (वयस्क कीट) के चरणों से गुजरते हैं। अन्य जानवरों (बिना कायापलट के विकसित) में, विभिन्न आयु राज्यों को भी प्रतिष्ठित किया जा सकता है, हालांकि उनके बीच की सीमाएं इतनी स्पष्ट नहीं हो सकती हैं।

किसी जनसंख्या की आयु संरचना (या, जैसा कि वे कहते हैं, आयु स्पेक्ट्रम) की प्रकृति किसी दी गई जनसंख्या की तथाकथित उत्तरजीविता वक्र विशेषता के प्रकार पर निर्भर करती है। उत्तरजीविता वक्र विभिन्न आयु समूहों में मृत्यु दर को दर्शाता है। इसलिए, यदि मृत्यु दर व्यक्तियों की आयु पर निर्भर नहीं करती है, तो उत्तरजीविता वक्र एक घटती हुई रेखा है (देखें आकृति, प्रकार I)। अर्थात् इस प्रकार में व्यक्तियों की मृत्यु समान रूप से होती है, मृत्यु दर जीवन भर स्थिर रहती है। ऐसा उत्तरजीविता वक्र उन प्रजातियों की विशेषता है जिनका विकास बिना कायापलट के होता है और जन्म लेने वाली संतानों की पर्याप्त स्थिरता होती है। इस प्रकार को आमतौर पर हाइड्रा का प्रकार कहा जाता है - यह एक सीधी रेखा के पास एक उत्तरजीविता वक्र की विशेषता है।

उन प्रजातियों में जिनके लिए मृत्यु दर में बाहरी कारकों की भूमिका छोटी होती है, उत्तरजीविता वक्र को एक निश्चित आयु तक मामूली कमी की विशेषता होती है, जिसके बाद प्राकृतिक (शारीरिक) मृत्यु दर के कारण तेज गिरावट होती है। इस प्रकार के करीब एक उत्तरजीविता वक्र मनुष्यों की विशेषता है (हालाँकि मानव उत्तरजीविता वक्र कुछ हद तक चपटा है और इस प्रकार I और II प्रकार के बीच कहीं है)। इस प्रकार को ड्रोसोफिला का प्रकार कहा जाता है: यह वह प्रकार है जिसे ड्रोसोफिला प्रयोगशाला स्थितियों में प्रदर्शित करता है (शिकारियों द्वारा नहीं खाया जाता है)।

कई प्रजातियों को ओटोजेनी के शुरुआती चरणों में उच्च मृत्यु दर की विशेषता है। ऐसी प्रजातियों में, जीवित रहने की अवस्था कम उम्र के क्षेत्र में तेज गिरावट की विशेषता है। जो व्यक्ति "गंभीर" उम्र से बच गए हैं, वे कम मृत्यु दर प्रदर्शित करते हैं और महान उम्र तक जीते हैं। प्रकार को सीप का प्रकार कहा जाता है। चित्र में III टाइप करें।

उत्तरजीविता वक्रों का अध्ययन पारिस्थितिकी विज्ञानी के लिए बहुत रुचि का है। यह आपको यह निर्धारित करने की अनुमति देता है कि किस उम्र में एक विशेष प्रजाति सबसे कमजोर है। यदि उन कारणों की कार्रवाई जो जन्म दर या मृत्यु दर को बदल सकती है, सबसे कमजोर स्तर पर आती है, तो जनसंख्या के बाद के विकास पर उनका प्रभाव सबसे बड़ा होगा। शिकार या कीट नियंत्रण का आयोजन करते समय इस पैटर्न को ध्यान में रखा जाना चाहिए।

जनसंख्या की यौन संरचना. बेशक, हम आबादी की यौन संरचना के बारे में तभी बात कर सकते हैं जब हम एक द्विलिंगी (उभयलिंगी) प्रजाति के बारे में बात कर रहे हों। जनसंख्या में व्यक्तियों की आनुवंशिक विविधता को बनाए रखने में उभयलिंगीता एक बड़ी भूमिका निभाती है। जनसंख्या स्थिरता के लिए आनुवंशिकी के महत्व पर अगले पाठ में विस्तार से चर्चा की जाएगी। अब हम ध्यान दें कि यौन संरचना, यानी लिंगानुपात, जनसंख्या के प्रजनन और उसकी स्थिरता से सीधे संबंधित है।

यह एक जनसंख्या में प्राथमिक, द्वितीयक और तृतीयक लिंगानुपात में अंतर करने की प्रथा है। प्राथमिक लिंग अनुपात आनुवंशिक तंत्र द्वारा निर्धारित किया जाता है - लिंग गुणसूत्रों के विचलन की एकरूपता। उदाहरण के लिए, मनुष्यों में, XY गुणसूत्र पुरुष लिंग के विकास को निर्धारित करते हैं, और XX - महिला। इस मामले में, प्राथमिक लिंगानुपात 1:1 है, यानी समान रूप से संभावना है।

द्वितीयक लिंगानुपात जन्म के समय (नवजात शिशुओं के बीच) लिंगानुपात है। यह कई कारणों से प्राथमिक से काफी भिन्न हो सकता है: X- या Y-गुणसूत्र को ले जाने वाले शुक्राणु के लिए अंडों की चयनात्मकता, ऐसे शुक्राणुओं की निषेचन की असमान क्षमता और विभिन्न बाहरी कारक। उदाहरण के लिए, प्राणी विज्ञानियों ने सरीसृपों में द्वितीयक लिंगानुपात पर तापमान के प्रभाव का वर्णन किया है। एक समान पैटर्न कुछ कीड़ों की विशेषता है। तो, चींटियों में, 20 सी से ऊपर के तापमान पर निषेचन सुनिश्चित किया जाता है, और कम तापमान पर, असंक्रमित अंडे रखे जाते हैं। नर बाद वाले से निकलते हैं, और मादा निषेचित से।

तृतीयक लिंगानुपात वयस्क जानवरों के बीच लिंगों का अनुपात है।

12. जनसंख्या की स्थानिक संरचना।

जनसंख्या की स्थानिक संरचना. जनसंख्या की स्थानिक संरचना अंतरिक्ष में व्यक्तियों के वितरण की प्रकृति को दर्शाती है।

अंतरिक्ष में व्यक्तियों के वितरण के तीन मुख्य प्रकार हैं:

वर्दी (व्यक्ति समान रूप से अंतरिक्ष में, एक दूसरे से समान दूरी पर होते हैं), प्रकार को समान वितरण भी कहा जाता है;

सामूहिक, या मोज़ेक (अर्थात, "धब्बेदार", व्यक्तियों को अलग-अलग समूहों में रखा जाता है);

यादृच्छिक, या फैलाना (व्यक्तियों को अंतरिक्ष में बेतरतीब ढंग से वितरित किया जाता है)।

यदि आप आँकड़ों के अनुकूल हैं, तो इस प्रकार की स्थानिक संरचना के बीच के अंतर को निम्नानुसार वर्णित किया जा सकता है। समान क्षेत्रों में व्यक्तियों की संख्या गिनते हुए, निश्चित संख्या में नमूने लें। यदि नमूनों में व्यक्तियों की संख्या का विचरण शून्य हो जाता है, तो हम एक समान वितरण के साथ काम कर रहे हैं। यदि विचरण अंकगणित माध्य के करीब है, तो यह एक यादृच्छिक वितरण है। यदि विचरण अंकगणित माध्य से बहुत बड़ा है, तो हम व्यक्तियों के सामूहिक वितरण की बात कर सकते हैं।

समान वितरण प्रकृति में दुर्लभ है और अक्सर तीव्र अंतर-विशिष्ट प्रतिस्पर्धा (जैसे, उदाहरण के लिए, शिकारी मछली में) के कारण होता है।

यादृच्छिक वितरण केवल एक सजातीय वातावरण में और केवल उन प्रजातियों में देखा जा सकता है जो एक साथ समूह करने की कोई प्रवृत्ति नहीं दिखाते हैं। समान वितरण के पाठ्यपुस्तक उदाहरण के रूप में, आटे में ट्रिबोलियम बीटल का वितरण आमतौर पर दिया जाता है।

समूहीकरण बहुत अधिक सामान्य है। यह सूक्ष्म पर्यावरण की विशेषताओं या जानवरों के व्यवहार की विशेषताओं के साथ जुड़ा हुआ है।

स्थानिक संरचना महान पारिस्थितिक महत्व की है। सबसे पहले, एक निश्चित प्रकार के क्षेत्र का उपयोग जनसंख्या को पर्यावरणीय संसाधनों का कुशलतापूर्वक उपयोग करने और अंतर-विशिष्ट प्रतिस्पर्धा को कम करने की अनुमति देता है। पर्यावरण का उपयोग करने की दक्षता और जनसंख्या के प्रतिनिधियों के बीच प्रतिस्पर्धा में कमी ने इसे इस पारिस्थितिकी तंत्र में रहने वाली अन्य प्रजातियों के संबंध में अपनी स्थिति को मजबूत करने की अनुमति दी है।

जनसंख्या की स्थानिक संरचना का एक अन्य महत्वपूर्ण मूल्य यह है कि यह जनसंख्या के भीतर व्यक्तियों की परस्पर क्रिया को सुनिश्चित करता है। इंट्रापॉपुलेशन संपर्कों के एक निश्चित स्तर के बिना, जनसंख्या अपनी प्रजातियों के कार्यों (प्रजनन, पुनर्वास) और पारिस्थितिकी तंत्र में भागीदारी से जुड़े कार्यों (पदार्थों के चक्र में भागीदारी, जैविक उत्पादों का निर्माण, और इसी तरह) दोनों को करने में सक्षम नहीं होगी। )

13. आबादी की नैतिक (व्यवहारिक) संरचना।

नैतिक (व्यवहारिक) संरचना. यह आबादी में व्यक्तियों के सह-अस्तित्व के विभिन्न रूपों को दर्शाता है। एकांत जीवन शैली को सबसे पहले अलग किया जाना चाहिए, हालांकि प्रकृति में जीवों का पूरी तरह से एकान्त अस्तित्व नहीं है, क्योंकि इस मामले में प्रजनन असंभव होगा। पारिवारिक जीवन शैली - माता-पिता और संतानों के बीच संबंध मजबूत होते हैं, जानवरों का क्षेत्रीय व्यवहार स्पष्ट रूप से प्रकट होने लगता है। विभिन्न संकेतों, चिह्नों, धमकियों आदि के माध्यम से संतानों के पालन-पोषण के लिए पर्याप्त स्थान पर कब्जा सुनिश्चित किया जाता है।

झुंड जानवरों का एक अस्थायी संघ है जो क्रियाओं के जैविक रूप से उपयोगी संगठन को प्रदर्शित करता है (दुश्मनों से खुद को बचाने के लिए, भोजन प्राप्त करने के लिए, प्रवास करने के लिए, आदि)। स्कूली शिक्षा मछली और पक्षियों के बीच सबसे व्यापक है, हालांकि यह स्तनधारियों में भी पाई जाती है (उदाहरण के लिए, कुत्तों में)।

एक झुंड जानवरों का एक दीर्घकालिक या स्थायी संघ है जिसमें एक प्रजाति के जीवन के सभी मुख्य कार्य किए जाते हैं: भोजन प्राप्त करना, शिकारियों से सुरक्षा, प्रवास, प्रजनन और युवा जानवरों को पालना।

प्रभुत्व संबंध झुंडों में समूह व्यवहार का आधार बनते हैं। एक अस्थायी या अपेक्षाकृत स्थायी नेता की उपस्थिति, जो झुंड के व्यक्तियों द्वारा अनुकरण की जाती है, विशेषता है।

झुंड का सक्रिय नेतृत्व (विशेष संकेत या धमकी) नेताओं द्वारा किया जाता है। एक पदानुक्रमित रूप से संगठित झुंड को आंदोलन के एक नियमित क्रम, दुश्मनों से बचाव करते समय कुछ स्थिति, आराम के स्थानों में स्थान आदि की विशेषता होती है (चित्र। 4.3)।

एक कॉलोनी लंबे समय तक या प्रजनन के मौसम के लिए गतिहीन जानवरों का एक समूह है। व्यक्तियों के बीच संबंधों की जटिलता के संदर्भ में, उपनिवेश बहुत विविध हैं, सबसे जटिल संबंध सामाजिक कीड़ों (दीमक, चींटियों, मधुमक्खियों, ततैया, आदि) के लिए बस्तियों में बनते हैं, जो एक बहुत विस्तारित परिवार के आधार पर उत्पन्न होते हैं। कॉलोनियों के सदस्य लगातार आपस में सूचनाओं का आदान-प्रदान कर रहे हैं।

15. समुदाय: अवधारणा और संरचना (विशिष्ट, स्थानिक, पारिस्थितिक)

समुदाय- एक निश्चित क्षेत्र पर कब्जा करने वाली परस्पर क्रिया करने वाली आबादी का एक समूह, पारिस्थितिकी तंत्र का एक जीवित घटक। समुदाय विभिन्न पोषी स्तरों, ऊर्जा प्रवाह और इसके माध्यम से पोषक चक्रण के साथ एक गतिशील इकाई के रूप में कार्य करता है।

सामुदायिक संरचना समय के साथ धीरे-धीरे निर्मित होती है। एक उदाहरण हाल ही में बने ज्वालामुखी द्वीप पर उजागर चट्टान के जीवों द्वारा उपनिवेशीकरण है। पेड़ और झाड़ियाँ नंगे चट्टान पर नहीं उग सकते, क्योंकि उनके लिए मिट्टी की आवश्यकता नहीं होती है। हालांकि, शैवाल और लाइकेन ऐसे प्रदेशों में अलग-अलग तरीकों से प्रवेश करते हैं और उनका उपनिवेश बनाते हैं अग्रणी समुदाय.

अंतिम समुदाय - स्थिर, आत्म-नवीकरणीय और पर्यावरण के साथ संतुलन में - कहा जाता है। चरमोत्कर्ष समुदाय।एक उदाहरण पर्णपाती वन है।

16. समुदायों में जीवों के बीच संबंधों के प्रकार।

17. पारिस्थितिकी तंत्र और बायोगेकेनोसिस की अवधारणाएं। पारिस्थितिक तंत्र संरचना।

नीचे पारिस्थितिकी तंत्र खुद-इन पारिस्थितिकी प्रणालियों - पदार्थों के संचलन को अंजाम देने, बाहरी प्रभावों का विरोध करने, जैविक उत्पादों का उत्पादन करने की क्षमता। का आवंटन सूक्ष्म पारिस्थितिक तंत्र(एक छोटा तालाब, सड़न के चरण में एक पेड़ का तना, एक मछलीघर, एक पोखर, जब तक वे मौजूद हैं और उनमें जीवित जीव हैं जो इन-इन के चक्र को लागू करने में सक्षम हैं।); मध्य पारिस्थितिक तंत्र(जंगल, तालाब, नदी); मैक्रोइकोसिस्टम(महासागर, महाद्वीप, प्राकृतिक क्षेत्र) और वैश्विकपारिस्थितिकी तंत्र - समग्र रूप से जीवमंडल।

पारिस्थितिक तंत्र में 2 ब्लॉक शामिल हैं:पहले में विभिन्न प्रजातियों के परस्पर जुड़े जीव होते हैं, जिसे उन्होंने कहा। बायोकेनोसिस(के. मोबियस), दूसरा ब्लॉक आवास है - बायोटोपया पारिस्थितिकी. प्रत्येक बायोकेनोसिस में कई प्रजातियां होती हैं, लेकिन प्रजातियां इसमें अलग-अलग व्यक्तियों के रूप में नहीं, बल्कि आबादी के रूप में शामिल होती हैं। आबादी- यह एक प्रजाति के व्यक्तियों का स्व-प्रजनन वाला हिस्सा है, कुछ हद तक एक ही प्रजाति के व्यक्तियों से अलग; - एक ही क्षेत्र में रहने वाले सभी जीवित जीवों की समग्रता है; - यह प्रजातियों का एक अपेक्षाकृत अलग-थलग हिस्सा है (एक ही प्रजाति के व्यक्तियों से मिलकर बनता है), एक निश्चित स्थान पर कब्जा कर लेता है और आत्म-नियमन में सक्षम होता है और व्यक्तियों की इष्टतम संख्या (एक निश्चित क्षेत्र (सीमा) में रहने वाले) को बनाए रखता है। क्षेत्रप्रजातियों का निवास स्थान है। सीमा को आबादी में विभाजित किया गया है। समुदायपौधों, जीवों का एक समुदाय है। नीचे पारिस्थितिकी तंत्र किसी भी प्रणाली के रूप में समझा जाता है जिसमें जीवित प्राणियों और उनके पर्यावरण शामिल होते हैं, जो एक एकल कार्यात्मक पूरे में एकजुट होते हैं। मुख्य खुद-इन पारिस्थितिकी प्रणालियों - पदार्थों के संचलन को अंजाम देने, बाहरी प्रभावों का विरोध करने, जैविक उत्पादों का उत्पादन करने की क्षमता। पारिस्थितिक तंत्र की सार्वभौमिक संपत्ति-उन्हें उद्भव, जिसमें यह तथ्य शामिल है कि संपूर्ण प्रणाली के गुण केवल इसके घटक भागों या तत्वों के गुणों का योग नहीं हैं। पारिस्थितिक तंत्र में ऊर्जा प्रक्रियाएंकोई भी पारिस्थितिकी तंत्र ऊर्जा की खपत करता है। ऊर्जा कहीं नहीं जा सकती, वह केवल दूसरी ऊर्जा में जा सकती है। और इसमें से कुछ नष्ट हो जाता है। अपरिवर्तनीय ऊर्जा अपव्यय का माप है एन्ट्रापी. एन्ट्रापी के विपरीत को नेगेंट्रॉपी कहा जाता है - यह एक प्रणाली की स्थिरता का एक उपाय है।

बायोगेसीनोसिस-सुकचेव के अनुसार, इसमें सभी ब्लॉक और लिंक शामिल हैं। इस अवधारणा का उपयोग भूमि प्रणालियों के लिए किया जाता है। बायोगेकेनोज में मुख्य कड़ी के रूप में एक उत्साही समुदाय की उपस्थिति अनिवार्य रूप से होती है। बायोगेकेनोज के उदाहरण घास के मैदान, सीढ़ियां, दलदल, वन क्षेत्र हैं। प्रत्येक बायोगेकेनोसिस को एक पारिस्थितिकी तंत्र कहा जा सकता है, लेकिन प्रत्येक पारिस्थितिकी तंत्र बायोगेकेनोसिस की श्रेणी से संबंधित नहीं है।

18. पारिस्थितिक तंत्र में ऊर्जा प्रवाह और खाद्य श्रृंखला।

सौर ऊर्जा जब जीवों द्वारा पहली प्रजाति, 1 खाद्य श्रृंखला, खाद्य श्रृंखला से दूसरी में उपभोग की जाती है।

ऊष्मप्रवैगिकी के सिद्धांत:

ऊर्जा को नए सिरे से नहीं बनाया और गायब किया जा सकता है, लेकिन केवल एक रूप से दूसरे रूप में स्थानांतरित किया जा सकता है।

ऊर्जा परिवर्तनों से जुड़ी प्रक्रियाएं अनायास ही आगे बढ़ सकती हैं यदि ऊर्जा एक केंद्रित रूप से एक विसरित रूप में जाती है।

किनारे की ऊर्जा से अंतर, एक बार शरीर द्वारा उपयोग किया जाता है, गर्मी में बदल जाता है और खो जाता है, पदार्थ जीवमंडल में प्रसारित होते हैं - जिसे जैव रासायनिक चक्र कहा जाता है।

ऊर्जा का बार-बार उपयोग नहीं किया जा सकता है। तत्वों के चक्र और स्व-विनियमन प्रक्रियाओं के कार्यान्वयन में जिसमें पारिस्थितिक तंत्र के सभी भाग भाग लेते हैं।

पावर सर्किट:किसी भी पारितंत्र में कई पोषी (भोजन) स्तर शामिल होते हैं। पहला स्तर - पौधे। उनके नाम। स्वपोषी। (यह खुद को खिलाता है) या निर्माता। दूसरे जानवर हेटरोट्रॉफ़ या उपभोक्ता हैं। अंतिम स्तर सूक्ष्मजीवों और कवक द्वारा दर्शाया गया है। उनके नाम। डीकंपोजर वे कार्बनिक पदार्थों को मूल खनिज तत्वों में विघटित करते हैं। खुद की बिजली श्रृंखला- चीजों के जैविक चक्र का कार्यान्वयन और कार्बनिक पदार्थों में संग्रहीत ऊर्जा की रिहाई।

पारिस्थितिकी तंत्र ऊर्जा

अपने अस्तित्व के लिए जीवित जीवों को लगातार ऊर्जा की भरपाई और व्यय करना चाहिए। प्रकाश संश्लेषण के दौरान पौधे रासायनिक बंधों में ऊर्जा का भंडारण करने में सक्षम होते हैं। प्रकाश संश्लेषण के दौरान पौधे सूर्य की ऊर्जा का केवल एक छोटा सा हिस्सा ही अवशोषित करते हैं। यह संयंत्र jav का केवल 1% है। खाद्य श्रृंखला में अन्य सभी जीवों के लिए प्राथमिक ऊर्जा आपूर्तिकर्ता। ऊर्जा का मुख्य हिस्सा जीवन समर्थन (आंदोलन, तापमान रखरखाव) पर खर्च किया जाता है, ऊर्जा का हिस्सा शरीर के शरीर में गुजरता है - उपभोक्ता, द्रव्यमान में वृद्धि के साथ।

19. पारिस्थितिकी तंत्र उत्पादकता और पारिस्थितिक पिरामिड।

विभिन्न पोषी स्तरों के जीवों का बायोमास समान नहीं होता है। स्थलीय पारितंत्रों में पोषी स्तर में वृद्धि के साथ यह घटता है, क्योंकि एक पोषी स्तर से दूसरे पोषी स्तर में जाने पर ऊर्जा का ह्रास होता है। विभिन्न पोषी स्तरों के जीवों के बायोमास के अनुपात को बायोमास पिरामिड के रूप में ग्राफिक रूप से दर्शाया गया है।

स्थलीय पारिस्थितिकी तंत्र। जलीय पारिस्थितिकी तंत्र।

प्रत्येक ट्राफिक स्तर के जीवों के बायोमास को एक आयत के रूप में दर्शाया जाता है, जिसकी लंबाई या क्षेत्र बायोमास की मात्रा के समानुपाती होता है।

स्थलीय पारितंत्रों में पोषी स्तर में वृद्धि के साथ बायोमास का भंडार घटता है, जबकि समुद्री पारितंत्रों में यह बढ़ता है। इन पारिस्थितिक तंत्रों में मुख्य उत्पादक फाइटोप्लांकटन है।

बायोमास के पिरामिडों के अलावा, संख्याओं के पिरामिड भी हैं। वे ऊर्जा पिरामिड भी बनाते हैं जो एक पोषी स्तर से दूसरे पोषी स्तर में इसके संक्रमण को दर्शाते हैं।

एकोपीरामिड। 10% नियम।

"सभी प्रजातियां जो खाद्य श्रृंखला बनाती हैं, पौधों द्वारा बनाई गई जैविक सामग्री के कारण मौजूद हैं।"

इस मामले में, ऊर्जा के उपयोग और रूपांतरण की दक्षता से जुड़ी एक महत्वपूर्ण नियमितता है।

"ऑटोट्रॉफ़्स (पौधों) द्वारा उत्पादन ऊर्जा को 10% की मात्रा में पारिस्थितिक पिरामिड के अगले चरण में स्थानांतरित किया जाता है"।

भविष्य में, कदम दर कदम, समान पैटर्न देखे जाते हैं।

21. पारिस्थितिक तंत्र की गतिशीलता: प्रगतिशील परिवर्तनों के प्रकार और तंत्र।

प्रगतिशील परिवर्तनलंबे समय तक और आमतौर पर एक बायोकेनोसिस को दूसरे द्वारा बदल दिया जाता है। उन्हें बुलाया जा सकता है:

पारिस्थितिक तंत्र जीवों की महत्वपूर्ण गतिविधि के प्रभाव में प्राकृतिक वातावरण में परिवर्तन;

जंगल की आग, जलवायु परिवर्तन, या मानव हस्तक्षेप जैसे व्यवधान के बाद प्रजातियों के बीच स्थिर संबंध स्थापित करना;

मानव प्रभाव।

प्रगतिशील परिवर्तनों को उत्तराधिकार कहा जाता है (अव्य। किसी और के स्थान पर लगातार प्रवेश, निरंतरता) - एक दूसरे के साथ और अजैविक वातावरण के साथ जीवों की बातचीत के परिणामस्वरूप पारिस्थितिकी तंत्र का आत्म-विकास। उत्तराधिकार के दौरान, एक अस्थिर बायोकेनोसिस को एक अधिक स्थिर बायोकेनोसिस से बदल दिया जाता है।

पारिस्थितिक तंत्र में रहने वाले जीवों की महत्वपूर्ण गतिविधि के कारण उत्तराधिकारियों पर विचार करें। जीवन की प्रक्रिया में जीव कुछ पदार्थों से पर्यावरण को संतृप्त करते हैं। पर्यावरण बदलता है और अन्य प्रजातियों के जीवन के लिए अधिक उपयुक्त हो जाता है, पूर्व की जगह।

एक नंगे चट्टानी क्षेत्र का उत्तराधिकार अजैविक कारकों - तापमान, आर्द्रता, सूर्य के प्रकाश के प्रभाव में चट्टानों के अपक्षय से शुरू होता है। चट्टानों का विनाश बैक्टीरिया, कवक, शैवाल, नीले-हरे, स्केल लाइकेन द्वारा जारी है। प्रारंभिक अवस्था में कार्बनिक पदार्थ के उत्पादक नीले-हरे, लाइकेन शैवाल और मुक्त रहने वाले शैवाल हैं। ब्लू-ग्रीन्स विशेष रूप से स्पष्ट हैं, वे वायुमंडल से नाइट्रोजन को स्वतंत्र रूप से अवशोषित करने में सक्षम हैं। खाद्य स्वतंत्रता नीले-सागों को निर्जन चट्टानों को विकसित करने की अनुमति देती है। उनके मरने वाले जीव पर्यावरण को नाइट्रोजन से समृद्ध करते हैं।

20. पारिस्थितिक तंत्र की गतिशीलता: चक्रीय परिवर्तनों के प्रकार और तंत्र।

चक्रीय परिवर्तनप्रकृति में आवधिक परिवर्तन के कारण - दैनिक, मौसमी और दीर्घकालिक। सूखे साल गीले लोगों के साथ वैकल्पिक होते हैं, और जीवों की आबादी की संख्या या तो सूखे या नमी के लिए अनुकूलित होती है।

दैनिक रूपांतरणबायोकेनोज आमतौर पर मजबूत व्यक्त किए जाते हैं, तापमान, आर्द्रता और अन्य पर्यावरणीय कारकों में दिन और रात में अंतर जितना अधिक होता है। तो मध्य एशिया के रेतीले रेगिस्तानों में गर्मियों में दोपहर के समय जीवन रुक जाता है। यहां तक ​​​​कि दिन के समय की गतिविधि वाली प्रजातियां भी गर्मी से सैक्सौल की छाया में या इसकी शाखाओं (अगम, छिपकलियों) में छिप जाती हैं। रेगिस्तान रात में जीवंत हो उठता है। यहाँ दैनिक जानवरों की तुलना में अधिक निशाचर और सांध्यकालीन जानवर हैं। कई दैनिक प्रजातियां गर्मियों में एक रात की जीवन शैली में बदल जाती हैं (अधिकांश सांप, मकड़ियों, अंधेरे)। कॉप्रोफैगस भृंग सक्रिय हैं, नाइटजर, हाउस उल्लू उड़ते हैं, जेरोबा, गेकोस फ़ीड, फालानक्स, बिच्छू, लोमड़ी, कोर्सैक, सांप शिकार करते हैं। उष्ण कटिबंध से लेकर टुंड्रा तक सभी क्षेत्रों के समुदायों में दैनिक लय का पता लगाया जा सकता है। टुंड्रा में गर्मियों में निरंतर रोशनी के साथ, पौधों में फूलों के खिलने, पक्षियों के भोजन, उड़ान में और कीड़ों के वितरण आदि में दैनिक लय का उल्लेख किया जाता है।

मौसमी परिवर्तनशीलताबायोकेनोज न केवल राज्य और गतिविधि में, बल्कि व्यक्तिगत प्रजातियों के मात्रात्मक अनुपात में, उनके प्रजनन चक्र, मौसमी प्रवास, वर्ष के दौरान व्यक्तिगत पीढ़ियों की मृत्यु आदि के आधार पर परिवर्तन में व्यक्त किया जाता है। वर्ष के कुछ निश्चित समय पर , कई प्रजातियों को समुदाय के जीवन से लगभग पूरी तरह से बाहर रखा जाता है, गहरी निष्क्रियता (मूर्खता, हाइबरनेशन, डायपॉज) की स्थिति में गुजरते हुए, अंडे और बीजों के चरण में एक प्रतिकूल अवधि का अनुभव करते हुए, अन्य बायोटोप्स या भौगोलिक क्षेत्रों में पलायन या उड़ान भरते हैं क्षेत्र।

मौसमी परिवर्तनशीलताबायोकेनोसिस की स्तरीय संरचना भी अक्सर प्रभावित होती है: पौधों के अलग-अलग स्तर वर्ष के संबंधित मौसमों में पूरी तरह से गायब हो सकते हैं, उदाहरण के लिए, वार्षिक से मिलकर एक जड़ी-बूटियों का स्तर।

समुदायों की मौसमी लय सबसे स्पष्ट रूप से जलवायु क्षेत्रों और गर्मी और सर्दियों की विपरीत परिस्थितियों वाले क्षेत्रों में व्यक्त की जाती है। कमजोर रूप में, हालांकि, उष्णकटिबंधीय वर्षावनों में भी उनका पता लगाया जा सकता है, जहां वर्ष के दौरान दिन की लंबाई, तापमान और आर्द्रता में बहुत कम परिवर्तन होता है।

दीर्घकालिक परिवर्तनशीलता- किसी भी बायोकेनोसिस के जीवन में एक सामान्य घटना। यह मौसम संबंधी स्थितियों (जलवायु उतार-चढ़ाव) या समुदाय को प्रभावित करने वाले अन्य बाहरी कारकों (उदाहरण के लिए, नदी बाढ़ की डिग्री) के वर्षों में परिवर्तन पर निर्भर करता है। इसके अलावा, लंबी अवधि की आवधिकता को पौधों के जीवन चक्र की विशेषताओं के साथ जोड़ा जा सकता है, जानवरों के बड़े पैमाने पर प्रजनन या पौधों के लिए रोगजनक सूक्ष्मजीवों आदि की पुनरावृत्ति के साथ।

22. जीवमंडल के जीवन और संरचना का वितरण।

बायोस्फीयर शब्द की शुरुआत 1875 में हुई थी। एडवर्ड सूस। प्रति बीओस्फिअउन्होंने वायुमंडल, जलमंडल और स्थलमंडल के उस सभी स्थान को जिम्मेदार ठहराया, जहां जीवित जीव मिलते हैं। वर्नाडस्की के अनुसार बीओस्फिअ- सभी अंतरिक्ष (पृथ्वी का खोल) जहां जीवन मौजूद है या अस्तित्व में है। जीवमंडल का वह भाग जहाँ वर्तमान में जीवित जीव पाए जाते हैं, कहलाते हैं। आधुनिक जीवमंडलया निओबायोस्फीयर, और प्राचीन जीवमंडल को संदर्भित किया जाता है पेलियोबायोस्फीयरया सफेद जीवमंडल(कोयला, तेल, चाक, अयस्क संरचनाओं के जमा)।

जीवमंडल की सीमाएँ:वायुमंडल में नियोबायोस्फीयर लगभग ओजोन परत (ध्रुवों पर 8-10 किमी, भूमध्य रेखा पर 17-18 किमी और शेष पृथ्वी की सतह से ऊपर - 20-25 किमी) तक फैला हुआ है। ओजोन परत के बाहर जीवन असंभव है। विनाशकारी पराबैंगनी किरणों के लिए। नीचे के तलछट, जहां जीवित जीवों का अस्तित्व संभव है, को भी नियोबायोस्फीयर के लिए जिम्मेदार ठहराया जा सकता है।

पैलियोबियोस्फीयर की सीमाएँवातावरण में मोटे तौर पर नियोबायोस्फीयर के साथ मेल खाता है; पानी के नीचे, तलछटी चट्टानों को भी पैलियोबियोस्फीयर के लिए जिम्मेदार ठहराया जा सकता है। यह मोटाई सैकड़ों मीटर से लेकर दसियों किलोमीटर तक होती है। आधुनिक, साथ ही सफेद जीवमंडल के भीतर, जीवन की संतृप्ति असमान है। जीवमंडल के किनारे पर, केवल संयोग से पेश किए गए जीव पाए जाते हैं। जीवमंडल के मुख्य भाग के भीतर, जीव लगातार मौजूद हैं, लेकिन समान रूप से नहीं।

जीवमंडल के मुख्य तत्व: 1) जीवमंडल एक केंद्रीकृत प्रणाली है. जीवित जीव इसकी केंद्रीय कड़ी हैं। 2) जीवमंडल एक खुला तंत्र है।बाहर से (सूर्य, अंतरिक्ष से) ऊर्जा प्राप्त किए बिना इसका अस्तित्व असंभव है। 3) जीवमंडल एक स्व-विनियमन प्रणाली है।अपनी मूल स्थिति में लौटने में सक्षम। ले चेटेलियर-ब्राउन के सिद्धांत:जब बल प्रणाली पर कार्य करते हैं, इसे स्थिर संतुलन की स्थिति से बाहर लाते हैं, तो बाद वाले को उस दिशा में विस्थापित कर दिया जाता है जिसमें इस प्रभाव का प्रभाव कमजोर होता है। 4) जीवमंडल एक ऐसी प्रणाली है जिसकी विशेषता महान विविधता है।जीवमंडल की स्थिरता के लिए विविधता को मुख्य शर्त माना जाता है।

जीवमंडल की एक महत्वपूर्ण संपत्ति हैइसमें उन तंत्रों की उपस्थिति होती है जो इन-इन के चक्र को सुनिश्चित करते हैं और व्यक्तिगत रासायनिक तत्वों और इसके साथ जुड़े उनके यौगिकों की अटूटता सुनिश्चित करते हैं।

23. ग्रहीय जैव-भू-रासायनिक चक्रों का सार और महत्व।

मूल ग्रह कार्यइसलिए, पृथ्वी पर जीवित पदार्थ सौर ऊर्जा के बंधन और भंडारण में निहित है, जो तब जीवमंडल में कई अन्य भू-रासायनिक प्रक्रियाओं का समर्थन करने के लिए जाता है।

पृथ्वी पर जीवन के अस्तित्व के दौरान, जीवित पदार्थ ने बड़ी मात्रा में सौर ऊर्जा को रासायनिक कार्य में परिवर्तित कर दिया है। भूवैज्ञानिक इतिहास के दौरान इसका एक महत्वपूर्ण हिस्सा एक बाध्य रूप में जमा हुआ है। आधुनिक जीवमंडल को पेलियोज़ोइक, मेसोज़ोइक और सेनोज़ोइक में गठित कोयले और अन्य कार्बनिक पदार्थों के जमाव की विशेषता है।

जीवमंडल में, सूक्ष्मजीवों की महत्वपूर्ण गतिविधि के परिणामस्वरूप, चर वैलेंस (नाइट्रोजन, सल्फर, लोहा, मैंगनीज, आदि) वाले तत्वों के ऑक्सीकरण और कमी जैसी रासायनिक प्रक्रियाएं बड़े पैमाने पर की जाती हैं। इन जीवों की गतिविधि के भूवैज्ञानिक परिणाम तलछटी सल्फर जमा के गठन, अवायवीय परिस्थितियों में धातु सल्फाइड के जमाव के गठन और एरोबिक स्थितियों में, उनके ऑक्सीकरण और घुलनशील अवस्था में स्थानांतरण, लोहे और फेरोमैंगनीज की घटना में प्रकट होते हैं। अयस्क

बड़ी संख्या में हेटरोट्रॉफ़्स, मुख्य रूप से कवक, जानवरों और सूक्ष्मजीवों की महत्वपूर्ण गतिविधि के कारण, पूरी पृथ्वी के पैमाने पर कार्बनिक अवशेषों को विघटित करने के लिए एक विशाल कार्य है। कार्बनिक द्रव्यमान के विनाश के दौरान, दो समानांतर प्रक्रियाएं होती हैं: खनिजकरण और एक महत्वपूर्ण ऊर्जा आरक्षित के साथ मिट्टी के धरण का निर्माण। ह्यूमस मिट्टी की उर्वरता का आधार है। इसका अपघटन भविष्य में बहुत धीमी गति से आगे बढ़ता है, एक निश्चित, ऑटोचथोनस मिट्टी माइक्रोफ्लोरा के प्रभाव में, जो पौधों को खनिज पोषण के तत्व प्रदान करने में स्थिरता सुनिश्चित करता है।

जीवित पदार्थ जीवमंडल में परमाणुओं का पुनर्वितरण करता है। पर्यावरण में अक्सर नगण्य सामग्री के बावजूद, कई जीवों में कुछ तत्वों को अपने आप में जमा और केंद्रित करने की क्षमता होती है। उदाहरण के लिए, लिथोटेमनियम शैवाल उनके शरीर में 10% तक मैग्नीशियम जमा करते हैं, ब्राचिओपोड के गोले में लगभग 20% फॉस्फोरस और सल्फर बैक्टीरिया 10% तक सल्फर होते हैं। कई जीव कैल्शियम, सिलिकॉन, सोडियम, एल्यूमीनियम, आयोडीन आदि को केंद्रित करते हैं। जब वे मर जाते हैं और द्रव्यमान में दब जाते हैं, तो वे इन पदार्थों का संचय करते हैं। चूना पत्थर, बॉक्साइट, फॉस्फोराइट, तलछटी लौह अयस्क आदि जैसे यौगिकों के भंडार हैं। मनुष्य उनमें से कई का उपयोग खनिजों के रूप में करता है।

24. मानव पर्यावरण के बुनियादी पारिस्थितिक गुण।

वायुमंडल

प्रदूषण के मुख्य स्रोत कार और औद्योगिक उद्यम हैं। हर साल 200 मिलियन टन कार्बन मोनोऑक्साइड और कार्बन डाइऑक्साइड, 150 मिलियन टन सल्फर ऑक्साइड और 50 मिलियन टन नाइट्रोजन ऑक्साइड वायुमंडल में उत्सर्जित होते हैं। इसके अलावा, बड़ी संख्या में सूक्ष्म कण वायुमंडल में उत्सर्जित होते हैं, जो तथाकथित वायुमंडलीय एरोसोल बनाते हैं। कोयले के दहन के कारण पारा, आर्सेनिक, सीसा, कैडमियम पदार्थों के संचलन में उनकी भागीदारी से अधिक मात्रा में वातावरण में प्रवेश करते हैं। पारिस्थितिक रूप से प्रदूषित क्षेत्रों में बड़ी मात्रा में धूल हवा में उगती है, जो 20-50% सूरज की रोशनी को रोक लेती है। वातावरण में कार्बन डाइऑक्साइड की सांद्रता में वृद्धि, जो पिछले 100 वर्षों में 10% की वृद्धि हुई है, बाहरी अंतरिक्ष में थर्मल विकिरण को रोकती है, जिससे ग्रीनहाउस प्रभाव होता है।

हीड्रास्फीयर

जल बेसिन के प्रदूषण का मुख्य कारण औद्योगिक और नगरपालिका उद्यमों के साथ-साथ कृषि भूमि से अनुपचारित अपशिष्ट जल का निर्वहन है। खनिज उर्वरकों और कीटनाशकों की नदियों में धोने से पीने के पानी की गुणवत्ता में गिरावट आती है और जलीय जानवरों की कई प्रजातियों की मृत्यु हो जाती है। विश्व महासागर के प्रदूषण का स्तर नदी अपवाह, वायुमंडलीय वर्षा और महासागरीय शेल्फ पर तेल उत्पादन के साथ बढ़ रहा है। बड़ी मात्रा में सीसा, तेल और तेल उत्पाद, घरेलू कचरा, कीटनाशक पानी में मिल जाते हैं।

स्थलमंडल

उपजाऊ मिट्टी की परत लंबे समय तक बनती है, और कृषि फसलों की खेती के लिए धन्यवाद, पौधों के पोषण के मुख्य तत्व, लाखों टन पोटेशियम, फास्फोरस और नाइट्रोजन सालाना मिट्टी से निकाले जाते हैं। यदि जैविक और खनिज उर्वरकों का प्रयोग किया जाता है तो मिट्टी का क्षरण नहीं होता है। यदि पौधों को नहीं खिलाया जाता है और फसल चक्र नहीं देखा जाता है, तो उपजाऊ परत कम से कम हो जाती है। मिट्टी की कृत्रिम सिंचाई का भी प्रतिकूल प्रभाव पड़ता है, क्योंकि मिट्टी की सतह परत का जल भराव या लवणीकरण सबसे अधिक बार होता है। मिट्टी में मानवजनित परिवर्तनों के बीच, अपरदन का बहुत महत्व है - ऊपरी उपजाऊ मिट्टी की परत का विनाश और विध्वंस। K-700 ट्रैक्टर एक मौसम में मिट्टी की एक परत को धूल में बदल देता है, जिसके बनने में 5 साल लगते हैं। हवा और पानी का कटाव हो रहा है। जल अपरदन सबसे विनाशकारी है, यह भूमि की अनुचित खेती से विकसित होता है।

पारिस्थितिक संकट

पारिस्थितिक संकट एक पारिस्थितिकी तंत्र के भीतर संबंधों का उल्लंघन है या मानव गतिविधि के कारण जीवमंडल में अपरिवर्तनीय घटना है। मानव जीवन और समाज के विकास के लिए खतरे की डिग्री के अनुसार, एक प्रतिकूल पारिस्थितिक स्थिति, एक पारिस्थितिक आपदा और एक पारिस्थितिक तबाही को प्रतिष्ठित किया जाता है।

25. मानव जाति की वैश्विक समस्याएं, उन्हें हल करने के तरीके।

1. मनुष्य, अन्य सभी जीवित प्रजातियों के विपरीत, न केवल सूर्य की ऊर्जा का उपयोग करता है, बल्कि पिछले भूवैज्ञानिक युगों में संचित कोयले, तेल और खनिजों का भी उपभोग करता है। पर्यावरण प्रदूषण जल, वायु, मिट्टी आदि की रासायनिक संरचना में परिवर्तन से प्रकट होता है। समस्या # 1: "ग्रीनहाउस प्रभाव" जीवाश्म ईंधन के जलने के कारण वातावरण में कार्बन डाइऑक्साइड में वृद्धि ओजोन परत का ह्रास। समस्या संख्या 2: पर्यावरण प्रदूषण लोगों के स्वास्थ्य का बिगड़ना आंखों और श्वसन पथ के श्लेष्म झिल्ली में जलन, घातक ट्यूमर।

मिट्टी की भौतिक और रासायनिक संरचना के बिगड़ने के मुख्य मानवजनित कारण।

मिट्टी की भौतिक और रासायनिक संरचना के बिगड़ने को प्रभावित करने वाले मुख्य मानवजनित कारक इस तरह की क्रियाएं हैं:

भूमि की जुताई, मिट्टी की खेती और खेती के लिए अनुपयुक्त भूमि

पर्याप्त वनीकरण के बिना वनों की कटाई और खुले गड्ढे खनन;

निर्माण, तीव्र विकास और उच्च जनसंख्या घनत्व;

चरागाहों का अत्यधिक गहन उपयोग;

मिट्टी और जल संसाधनों का कुप्रबंधन, जिससे मिट्टी का कटाव, लवणीकरण और जलभराव में वृद्धि हुई है;

मानव जाति ने विज्ञान और प्रौद्योगिकी के विकास में अभूतपूर्व सफलता हासिल की है, पृथ्वी के गुरुत्वाकर्षण की सीमाओं से बच गई है, लेकिन फिर भी इसका अस्तित्व भूमि के एक हिस्से को ढकने वाली एक पतली फिल्म पर निर्भर करता है - मिट्टी। इसकी मोटाई 2 मीटर से 1.5-2 सेमी तक होती है।

मिट्टी एक प्राकृतिक संरचना है जिसमें चेतन और निर्जीव प्रकृति में निहित कई गुण हैं। मिट्टी के गुणों में सबसे महत्वपूर्ण है इसमें ह्यूमस और पानी की उपस्थिति से जुड़ी उर्वरता।

पृथ्वी का प्रदूषण बहुत व्यापक स्तर पर हो गया है। कितनी बार भवन भागों के अवशेषों से ढके क्षेत्र होते हैं: पैनल, ब्लॉक, ईंटें, राख, लावा से अटे पड़े। टैंक फार्मों के क्षेत्रों में, भूमि ईंधन तेल, तेल और स्नेहक की एक परत से ढकी होती है।

शहर ही नहीं ग्रामीण अंचलों में भी गंदगी का अंबार बढ़ रहा है। इसके अलावा, यह मिट्टी और पानी का प्रदूषक हो सकता है। मशीन के पुर्जे जमीन पर पड़े रहते हैं, कृषि उपकरण खुले में पड़े रहते हैं, और बस इधर-उधर पड़े रहते हैं-सब जंग लग जाता है, जिससे लोहा और अन्य धातुएं मिट्टी में प्रवेश कर जाती हैं। इस प्रकार, आर्सेनिक, पारा, तांबा आदि के यौगिक मिट्टी में जमा हो जाते हैं।सल्फर डाइऑक्साइड मिट्टी में प्रवेश करती है और इसे काफी अम्लीकृत करती है, जो किसानों को अधिक चूना लगाने के लिए मजबूर करती है। 70 के दशक में, जर्मनी ने पेश किया

54 किग्रा / हेक्टेयर, और 1982-83 - 170 किग्रा / हेक्टेयर।

राजमार्गों के पास मिट्टी में सीसे की मात्रा बढ़ जाती है। तेल उत्पादों द्वारा मृदा प्रदूषण व्यापक है। कीटनाशक मिट्टी में मिल सकते हैं। अंत में, यहां तक ​​​​कि खनिज उर्वरक भी मिट्टी को नुकसान पहुंचा सकते हैं, खासकर जब उन्हें किसी दिए गए क्षेत्र की विशिष्ट विशेषताओं को ध्यान में रखे बिना लागू किया जाता है। पौधे हमेशा उर्वरकों में सभी पोषक तत्वों का उपयोग नहीं कर सकते हैं। ऑल-यूनियन एग्रीकल्चरल एकेडमी ऑफ एग्रीकल्चरल साइंसेज के शिक्षाविद वी। विनोग्रादोव के अनुसार, खनिज उर्वरकों की संरचना में मिट्टी में पेश किए गए सभी नाइट्रोजन में से 16-20% और कभी-कभी 50% अवशोषित नहीं होते हैं।

मृदा प्रदूषण का खतरा केवल इसके भौतिक और रासायनिक गुणों में परिवर्तन में ही नहीं है। विदेशी पदार्थ, मिट्टी में मिल रहे हैं, मिट्टी के बायोकेनोसिस के अलग-अलग समूहों के बीच मौजूदा लिंक को नष्ट कर देते हैं। स्थापित पोषी शृंखलाएं नष्ट हो जाती हैं। यह सब अंततः प्रजनन क्षमता को प्रभावित करता है। घरेलू और पशुधन अपशिष्ट जल रोगजनक बैक्टीरिया से मिट्टी को प्रदूषित करते हैं।

लवणीकरण पौधों की सामान्य वृद्धि और विकास के लिए अस्वीकार्य सांद्रता में ऊपरी मिट्टी की परत में सोडियम, कैल्शियम, मैग्नीशियम लवण के संचय की प्रक्रिया है। यह मिस्र, इराक, भारत, पाकिस्तान और अन्य शुष्क देशों में व्यापक हो गया है। पूर्व यूएसएसआर में मिट्टी का सबसे बड़ा लवणीकरण मध्य एशिया और ट्रांसकेशिया की सिंचित भूमि पर होता है।

कम लवणता होने पर भी कपास की उपज 20-30%, मक्का 40-50% और गेहूं 50-60% कम हो जाती है।

गैर-चेरनोज़म क्षेत्र के कई क्षेत्रों में, उरल्स में, बाल्टिक राज्यों और बेलारूस में जलभराव के परिणामस्वरूप, मिट्टी का जलभराव व्यापक हो गया है। यह देश के अन्य हिस्सों में नहरों और जलाशयों के पास भी देखा जाता है। आर्द्रभूमि को निकालने के लिए, स्लेटेड नालियों को जमीन में काट दिया जाता है।

प्राकृतिक संसाधनों की कमी और गैर-ब्लैक अर्थ क्षेत्र की प्रकृति पर अवांछनीय और नकारात्मक प्रभावों से सुरक्षा को ध्यान में रखते हुए आर्द्रभूमि का सुधार किया जाना चाहिए।

26. अंतर्राष्ट्रीय पर्यावरण संगठन और पर्यावरण कानून।

पर्यावरणीय मुद्दों पर अंतर्राष्ट्रीय सहयोग का नेतृत्व किसके द्वारा किया जाता है? यूनेस्को. 1972 में, उन्होंने अंतर सरकारी कार्यक्रम विकसित किया संयुक्त राष्ट्रपर्यावरण पर। यह पर्यावरण शिक्षा के विकास में सहायता करता है। रिकॉर्ड रखता है और विश्व विरासत के रूप में वर्गीकृत प्राकृतिक वस्तुओं के संरक्षण का आयोजन करता है।

प्रकृति और प्राकृतिक संसाधनों के संरक्षण के लिए अंतर्राष्ट्रीय संघ (आईयूसीएन)। उनकी गतिविधि का क्षेत्र प्राकृतिक पारिस्थितिक तंत्र का संरक्षण, पौधों और जानवरों की दुर्लभ और लुप्तप्राय प्रजातियों का संरक्षण, साथ ही साथ प्राकृतिक स्मारक, प्रकृति भंडार और राष्ट्रीय उद्यानों का संगठन है। पर्यावरण शिक्षा।

विश्व स्वास्थ्य संगठन (डब्ल्यूएचओ). गतिविधि का क्षेत्र - स्वच्छता का संगठन - पर्यावरण की महामारी विज्ञान निगरानी। स्वच्छता और स्वच्छ परीक्षा आयोजित करना और पर्यावरण की गुणवत्ता का आकलन करना।

अंतर्राष्ट्रीय परमाणु ऊर्जा एजेंसी (IAEA). गतिविधि का क्षेत्र - परमाणु ऊर्जा संयंत्रों के निर्माण और संचालन के लिए नियमों का विकास। विकिरण सुरक्षा मानकों की स्थापना। पर्यावरण पर रेडियोधर्मी पदार्थों के प्रभाव का आकलन।

अंतर्राष्ट्रीय समुद्री संगठन (आईएमओ). प्रदूषण से समुद्र के संरक्षण पर अंतर्राष्ट्रीय सम्मेलनों का विकास।

संयुक्त राष्ट्र के खाद्य और कृषि संगठन (एफएओ)।गतिविधि का क्षेत्र कृषि में पर्यावरणीय समस्याओं का समाधान है। समुद्र की भूमि, जल संसाधन, वन, वन्य जीवन, जैविक संसाधनों का संरक्षण और उपयोग।


इसी तरह की जानकारी।


11.1. आबादी के लिए मानवजनित जोखिम कारक

आबादी पर मानव प्रभाव अलग-अलग तीव्रता का हो सकता है और कई चैनलों के माध्यम से होता है, जो कई आबादी के अस्तित्व के लिए जोखिम पैदा करता है। मुख्य जोखिम कारक निम्नलिखित हैं।
स्थान बरबादी। स्टेपी पौधों और जानवरों की आबादी प्राकृतिक कदमों के क्षेत्र में तेज कमी और कृषि योग्य भूमि में उनके विकास से पीड़ित है। उत्तरी जानवरों की आबादी का प्राकृतिक प्रवास रैखिक संरचनाओं (सड़कों, पाइपलाइनों) से प्रतिकूल रूप से प्रभावित होता है जो तेल और गैस उत्पादन के क्षेत्रों में निर्मित होते हैं। वोल्गा और अन्य नदियों पर जलाशयों के कैस्केड के निर्माण ने स्टर्जन और अन्य मूल्यवान मछली प्रजातियों के लिए स्पॉनिंग क्षेत्रों में प्रवास करना असंभव बना दिया। नदियों और झीलों के जल संरक्षण क्षेत्रों में दचा निर्माण पक्षियों के लिए अशांति पैदा करता है और उनके जीवों की दरिद्रता की ओर जाता है।
पर्यावरण का रासायनिक प्रदूषण। यह कारक मुख्य रूप से जलीय पारिस्थितिक तंत्र के निवासियों पर प्रतिकूल प्रभाव डालता है। इस प्रकार, अम्लीय वर्षा ने मध्य यूरोप की झीलों के जलीय जीवों की कई प्रजातियों की मृत्यु का कारण बना। वोल्गा और उरल्स में जल प्रदूषण से स्टर्जन मछली के रोग होते हैं। 1950 और 1960 के दशक में राइन का औद्योगिक प्रदूषण नदी के मछली जीवों को लगभग पूरी तरह से नष्ट कर दिया। सौभाग्य से, पिछले 30 वर्षों में, जब प्रदूषण के स्तर में नाटकीय रूप से गिरावट आई है, जीव-जंतु ठीक हो गए हैं। समुद्र में, तेल प्रदूषण से तटीय और महाद्वीपीय शेल्फ के निवासियों की आबादी को बहुत नुकसान होता है, विशेष रूप से टैंकर दुर्घटनाओं में, जब हजारों टन तेल पानी में गिर जाता है। हाल ही में मेक्सिको की खाड़ी में एक तेल मंच पर एक दुर्घटना के परिणामस्वरूप जीवों की सामूहिक मृत्यु हुई।
जनसंख्या का अत्यधिक दोहन। आबादी की वसूली क्षमता से अधिक के उपयोग से जनसंख्या के आकार में कमी और उनकी मृत्यु हो जाती है। रूस में, गहन शिकार (विशेषकर अवैध शिकार) के प्रभाव में, एल्क और साइगा आबादी की संख्या में तेजी से कमी आई है। अवैध शिकार ने कैस्पियन बेसिन में स्टर्जन की आबादी को कमजोर कर दिया है। सुदूर पूर्व में, गहन कटाई के परिणामस्वरूप, टूटे फर्न की आबादी समाप्त हो गई है। दक्षिणी Urals में, औषधीय पौधे Rhodiola Iremelskaya की प्राकृतिक आबादी व्यावहारिक रूप से नष्ट हो जाती है। रूस के यूरोपीय भाग के कई क्षेत्रों में, क्रैनबेरी और ब्लूबेरी की आबादी कम हो गई है।
गहन चराई का प्रभाव। पशुधन द्वारा प्राकृतिक फाइटोफेज के प्रतिस्थापन, जो कि स्टेपी के पौधों के समुदायों के साथ पारिस्थितिक संतुलन में थे, ने स्टेपी पारिस्थितिक तंत्र की प्रजातियों की संरचना में कमी (34 गुना) की। इसका एक उदाहरण दक्षिणी यूराल के मैदानों में पंख घास की आबादी है। अतीत में, पंख घास प्रमुख थे, लेकिन पिछले 30 वर्षों में वे भारी चराई से इतने कम हो गए हैं कि वे लाल किताबों के पन्नों पर दिखाई दिए हैं। कुंवारी और परती भूमि के विकास का पंख घास के भाग्य पर हानिकारक प्रभाव पड़ा, जिससे प्राकृतिक स्टेपी समुदायों का क्षेत्र कम हो गया और चरागाह भार बढ़ गया। साहसी प्रजातियों का प्रभाव। कई मामलों में, प्राकृतिक रूप से शुरू की गई प्रजातियां मूल निवासियों की तुलना में बेहतर स्थिति में हैं, क्योंकि नए क्षेत्रों में उनके पास कोई नियंत्रण प्रजाति नहीं है, देशी वनस्पतियों और जीवों की प्रजातियों को बाहर कर रही है। ऑस्ट्रेलिया में कंगारुओं की जगह खरगोश ले रहे हैं। न्यूजीलैंड में, पेश किए गए काले हंस और सिका हिरण ने कई देशी प्रजातियों की आबादी को तबाह कर दिया है। देशी प्रजातियों का विस्थापन पौधों के लिए परागणकों सहित किसी भी संसाधन के अवरोधन के कारण हो सकता है, जैसा कि यूरोप में लोहे के प्रसार के साथ हुआ था (6.2.1 देखें)।

परीक्षण प्रश्न

1. जनसंख्या के लिए मुख्य मानवजनित जोखिम कारकों की सूची बनाएं।
2. जनसंख्या के अत्यधिक दोहन के उदाहरण दीजिए।
3. विदेशी प्रजातियां देशी प्रजातियों की आबादी को कैसे प्रभावित कर सकती हैं?

जीनस कोविल (स्टिपा) परिवार ब्लूग्रास

(स्टिपा पेन्नाटा एल।>।) बेलगोरोड स्टेप्स की बात करें तो, मुख्य प्रमुख - पंख वाली पंख वाली घास का उल्लेख करने में विफल नहीं हो सकता है। पंख घास चाक और चूना पत्थर की बाहरी सतह पर भी पाई जाती है। फेदर ग्रास 30-100 सेंटीमीटर ऊँचा एक बारहमासी घनी घास का पौधा है। तने नंगे होते हैं, जल्द ही नोड्स के नीचे प्यूब्सेंट होते हैं। पत्तियां लंबाई में लुढ़कती हैं या, शायद ही कभी, सपाट, 0.52 मिमी चौड़ी, शीघ्र ही शीर्ष पर इंगित की जाती हैं।

पंख घास अप्रैल-मई में खिलती है, मई-जून में फलती है। पुष्पक्रम - पुष्पगुच्छ, 3-5 सेमी लंबा, 6-20 स्पाइकलेट्स से। पंख घास का फूल अनाज के लिए विशिष्ट है, लेकिन एक चीज में उनसे काफी अलग है - एक विशेष निचला लेम्मा। यह घनी फिल्म शीर्ष पर एक पतली और बेहद लंबी फिल्मी उपांग में गुजरती है - 40 सेमी तक एक awn. 189 awn बीच में व्यक्त किया जाता है, निचले मुड़ वाले हिस्से में नग्न, ऊपर पिननेट, लगभग 5 मिमी लंबे बालों के साथ। पंख वाली घास के दानों के कारण हवा लंबी दूरी तय करती है।

गुरुत्वाकर्षण का केंद्र दाने के नीचे स्थित होता है, इसलिए इसका नुकीला निचला हिस्सा जमीन में चिपक जाता है। सिरे के पास पीछे की ओर मुख वाले बालों का कोरोला होता है। जैसे ही कैरियोप्सिस जमीन में गहरा हुआ है, बाल छोटे लंगर की तरह उसे पकड़ लेते हैं। अनाज का आत्म-दफन शुरू होता है। वह, एक कॉर्कस्क्रू की तरह, जमीन में खराब हो गई है। यदि यह सूखा है, तो यह एक पेचदार आकार में मुड़ जाता है; यदि बारिश होती है, तो यह खुल जाती है। लेकिन फल एक ही समय में जमीन में गहरे और गहरे दब जाते हैं।

क्या यह महत्वपूर्ण है! पंख घासस्टेपी में तेजी से दुर्लभ पाया जाता है। इसे मवेशी चरागाहों में खाते हैं। इन्फ्लोरेसेंस का उपयोग अक्सर सूखे गुलदस्ते में किया जाता है, और पंख वाले awns को चमकीले रंगों में चित्रित किया जाता है। सूखी घास के वसंत में जलने और स्टेपी क्षेत्रों की जुताई से पंख घास को बहुत नुकसान होता है। पंख घास की संख्या में कमी के कारण, यह संरक्षण के अधीन है और रूस में शामिल है और (दुर्लभ स्थिति II की श्रेणी - एक प्रजाति जो संख्या में घट रही है)।

बालों वाली पंख घास(tyrsa) (Stipa capillata L.j एक बारहमासी, टर्फ पौधा 40-80 सेमी ऊँचा है। यह चाक के साथ पंक्तिबद्ध स्टेप्स में, चाक आउटक्रॉप्स, पुराने जमा पर बढ़ता है। यह अन्य प्रकार की पंख घास की तुलना में अधिक बार होता है। इसका मुख्य अंतर नंगे है। बालों की तरह 15 लंबे -25 सेमी (जिसके परिणामस्वरूप पौधे को इसका नाम मिला) और बाद में फूल आने का समय - जुलाई।

क्या यह महत्वपूर्ण है!शीर्षासन करने से पहले, बालों वाली पंख वाली घास (टाइरसा) को पशुओं द्वारा अच्छी तरह से खाया जाता है, इसे डेयरी घोड़ी और भेड़ के लिए सबसे अच्छा भोजन माना जाता है। फलने के दौरान, पंख घास के चरागाहों पर चरना खतरनाक हो जाता है: तेज कैरियोप्स जानवर के बालों से चिपक जाते हैं, फिर (हाइग्रोस्कोपिसिटी के कारण) वे चलते हैं और उसके शरीर में घुस जाते हैं। नतीजतन, पंख घास के फल न केवल जानवर को पीड़ित करते हैं, बल्कि अक्सर मौत का कारण बनते हैं, इसके आंतरिक अंगों में प्रवेश करते हैं, जिससे फोड़े और रक्तस्राव होता है।

पहले, पशु चिकित्सा पर पाठ्यपुस्तकों में, इस पंख घास की बीमारी को "शेके-क्रुत" कहा जाता था, जिसका अर्थ कज़ाख में "अस्थायी कीड़ा" होता है। यह खूबसूरत अनाज घोड़ों और मवेशियों में "हेजहोग बियर्ड" नामक पंख-घास की बीमारी का कारण भी बन सकता है। बालों वाली पंख वाली घास की पत्तियाँ बालों से मिलती-जुलती हैं, जिनकी सतह ऊपर की ओर निर्देशित नुकीले कांटों से घनी होती है। जानवर के अंदर घुसने पर ये रीढ़ की हड्डी में जान आ जाती है। एक चादर, फिर दूसरी - उनमें से एक पूरा गुच्छा इकट्ठा किया जाता है, जो रीढ़ की मदद से आगे बढ़ता है, जिससे जानवरों की हार होती है।

पंख कम करना(स्टिपा लेसिंगियाना ट्रिन। एट रुपर.जे) स्टेप्स में, चाक और चूना पत्थर के बहिर्वाह पर बढ़ता है। इसकी ऊंचाई 30-70 सेमी है। इसके आंवले पिननेट होते हैं, अपेक्षाकृत छोटे बालों से ढके होते हैं। एवन की लंबाई 15-25 सेमी है। लेम्मा छोटे होते हैं, बल्कि छोटे होते हैं (एवन के बिना लंबाई 8-11 मिमी होती है)। अप्रैल-मई में खिलते हैं, मई-जून में फल लगते हैं। बीज द्वारा प्रचारित।

क्या यह महत्वपूर्ण है! पंख कम करनाचारे की दृष्टि से पंख वाली घासों में सर्वश्रेष्ठ मानी जाती है। हालांकि, यह याद रखना चाहिए कि लेसिंग की पंख घास सूचीबद्ध है बेलगोरोड क्षेत्र की लाल किताब(दुर्लभ स्थिति श्रेणी III - दुर्लभ प्रजाति)।

स्टेपीज़ (कामेन्या पथ) में पंख घास की दो और दुर्लभ प्रजातियाँ हैं, जो कम मात्रा में रूस और हमारे क्षेत्र की लाल किताबों में शामिल हैं।

पंख घास यौवन(Stipa dasyphylla (लिंडेम।) Trautv।) में प्यूब्सेंट लीफ ब्लेड्स होते हैं। यह इसे अन्य प्रकार की पंख घास से अलग करता है। इसकी दुर्लभ स्थिति श्रेणी I है - एक लुप्तप्राय प्रजाति।

सबसे सुंदर पंख वाली घास(Stipa pulcherrima C.Coch) पत्तियों में पंख वाली यौवन पत्तियों से अलग है या बाहर की तरफ विरल ब्रिसल्स के साथ, लेकिन लंबे मुलायम बालों के बिना। पंख घास से - निचले लेम्मा पर बालों की एक पट्टी, 2.5 मिमी awn के आधार तक नहीं पहुंचती। इसकी दुर्लभ स्थिति श्रेणी III है - एक दुर्लभ प्रजाति। वानस्पतिक अवस्था में इन पंख वाली घासों को पशुधन द्वारा खाया जाता है। घने यौवन और पत्तियों के मजबूत खुरदरेपन की उपस्थिति के कारण पंख घास यौवन बहुत खराब खाया जाता है।

यह दिलचस्प है! रूसी नाम कोविलीस्लाव शब्द फोर्ज से आया है - बीट, चॉप। पंख का अर्थ है घास जो काट दी जाती है। शायद यह शब्द तुर्क भाषा से आया है - कोवालिक, जिसका अर्थ है पत्ती रहित ईख। जीनस स्टिपा का लैटिन नाम ग्रीक स्टिप - टो, टो (पंख घास की अधिकांश प्रजातियों के awns के मजबूत यौवन के कारण) पर वापस जाता है।

स्टेपी में उगने वाले पंख नमी की निरंतर कमी की स्थिति में जीवन के अनुकूल हो गए हैं। उदाहरण के लिए, उनके रंध्र पत्ती की सतह के साथ चलने वाले खांचे के अंदर रखे जाते हैं। जैसे ही सूरज कड़ी मेहनत करना शुरू करता है, पत्ती एक ट्यूब में लुढ़क जाती है। अब रंध्र बंद खांचे की गहराई में सुरक्षित रूप से छिपे होते हैं, जो स्वयं पत्ती के बंद हिस्से में स्थित होते हैं। यह दोहरी सुरक्षा शीट को सूखने से बचाती है।

पंख स्टेपी के प्रतीक हैं। अब, जब ये अद्वितीय समुदाय कम और कम होते जा रहे हैं, तो प्रत्येक जीवित साइट को बचाना आवश्यक है। इन प्रजातियों के सभी आवासों को वनस्पति भंडार या विशेष रूप से संरक्षित प्राकृतिक क्षेत्रों के अन्य रूपों में शामिल किया जाना चाहिए। और फिर हर वसंत में हम बार-बार चांदी के लहराते स्टेपी समुद्र की प्रशंसा कर पाएंगे!

लिट।: / चेर्न्यावस्किख वी.आई., डिग्टियर ओ.वी., डिग्टियर ए.वी., दुमचेवा ई.वी. - बेलगोरोड।


ऊपर